■ちょっとした疑問や質問はここに書いてね104■

このエントリーをはてなブックマークに追加
1ご冗談でしょう?名無しさん
■ちょっとした疑問や質問はここに書いてね103■
http://science6.2ch.net/test/read.cgi/sci/1229687020/

質問する前に
教科書や参考書をよく読もう
http://www.google.com/ などの検索サイトを利用し、各自で調べること
宿題を聞くときは、どこまでやってみてどこが分からないのかを書くこと。
丸投げはダメだからね
(丸投げ君は完全無視。答えるだけ無駄。)

質問に対する返答には、何かしらの返答ちょうだいね
★書き込む前に>>2の注意事項を読んでね
★数式の書き方(参考)はこちら>>3
(予備リンク:>>2-10
荒らし厳禁、煽りは黙殺、忘れないうちに定期age
単発質問スレを発見したらこのスレッドへの誘導をよろしくね。

定番FAQ
英語最新版
http://math.ucr.edu/home/baez/physics/
旧版日本語訳
http://research.kek.jp/people/morita/phys-faq/
他にも参考にあるサイトなどあればぜひ。
例)http://ja.wikipedia.org/
※wikipedia内の説明はすべてが正確なわけではありません。
このスレでの受け答えもそうですが。相互に補完しつつ精度を高めましょう。
2ご冗談でしょう?名無しさん:2009/01/01(木) 22:53:31 ID:??? BE:454421186-PLT(44180)
書き込む際の注意

1.)
板の性格上、UNIX端末からの閲覧も多いと推察されます。
機種依存文字(ローマ数字、丸数字等)は避けて下さい。

2.)
以下のような質問に物理板住人は飽き飽きしているので、たぶん無視されます。
しないで下さい。
「相対性理論は間違っています」「量子力学は間違っています」
「宇宙論は間違っています」「シュレディンガーの猫は変です」
「永久機関を作りました」「タイムマシンについて教えて」
「どうして〜?」関連(→「どのようにして〜?」と質問すること)
「なぜ〜?」関連(たいてい、物理学の対象ではないため)

「どうして・なぜ」:http://academy6.2ch.net/philo/
(哲学板・雑談板のほうがむいている場合が多いです。)
雑談は雑談スレで:http://science6.2ch.net/test/read.cgi/sci/1181386663/

3.)
宿題を聞くときは、どこまでやってみてどこが分からないのかを書くこと。
丸投げはダメよ。丸投げに答えるのもダメよ。 丸投げを相手にする人はお馬鹿さん。
せめてポインタを示す程度に留めましょう。

4.)
厨房を放置できない奴も厨房
3ご冗談でしょう?名無しさん:2009/01/01(木) 22:53:46 ID:??? BE:302948148-PLT(44180)
数式の書き方(参考)
●スカラー:a,b,...,z, A,...,Z, α,β,...,ω, Α,Β,...,Ω,...(「ぎりしゃ」「あるふぁ〜おめが」で変換)
●ベクトル:V=[v1,v2,...], |V>>,V↑,vector(V) (混同しないならスカラーの記号でいい。通常は縦ベクトル)
●テンソル:T^[i,j,k...]_[p,q,r,...], T[i,j,k,...;p,q,r,...]  (上下付き1成分表示)
●行列  M[i,j], I[i,j]=δ_[i,j]  M=[[M[1,1],M[2,1],...],[M[1,2],M[2,2],...],...], I=[[1,0,0,...],[0,1,0,...],...]
(右は全成分表示。行または列ごとに表示する。例:M=[[1,-1],[3,2]])
●転置行列・随伴行列:M ',tM, M†("†"は「きごう」で変換可) ●行列式・トレース:|A|=det(A), tr(A)
●複号:a±b("±"は「きごう」で変換可)
●内積・外積・3重積:a・b, a×b, a・(b×c)=(a×b)・c=det([a,b,c]), a×(b×c)
●関数・数列:f(x), f[x] a(n), a[n], a_n
●平方根:√(a+b)=(a+b)^(1/2)=sqrt(a+b) ("√"は「るーと」で変換可)
●指数関数・対数関数:exp(x+y)=e^(x+y) ln(x/2)=log[e](x/2)(exp(x)はeのx乗、lnは自然対数)
●三角比:sin(a), cos(x+y), tan(x/2)
●絶対値:|x|  ●共役複素数:z~ ●ガウス記号:[x] (関数の変数表示と混同しないよう注意)
●階乗:n!=n*(n-1)*(n-2)*...*2*1, n!!=n*(n-2)*(n-4)*...
●順列・組合せ:P[n,k]=nPk, C[n,k]=nCk, Π[n,k]=nΠk, H[n,k]=nHk ("Π"は「ぱい」で変換可)
4ご冗談でしょう?名無しさん:2009/01/01(木) 22:54:00 ID:??? BE:681631889-PLT(44180)
数式の書き方続き(参考)
●微分・偏微分:dy/dx=y', ∂y/∂x=y_x ("∂"は「きごう」で変換可)
●ベクトル微分:∇f=grad(f), ∇・A=div(A),∇xA=rot(A), (∇^2)f=Δf ("∇"は「きごう」,"Δ"は「でるた」で変換可.)
●積分:∫[0,1]f(x)dx=F(x)|_[x=0,1], ∫[y=0,x]f(x,y)dy, ∬[D]f(x,y)dxdy, ∬[C]f(r)dl ("∫"は「いんてぐらる」,"∬"は「きごう」で変換可)
●数列和・数列積:Σ[k=1,n]a(k), Π[k=1,n]a(k) ("Σ"は「しぐま」,"Π"は「ぱい」で変換可)
●極限:lim[x→∞]f(x) ("∞"は「むげんだい」で変換可)
●図形:"△"は「さんかく」 "∠"は「かく」 "⊥"は「すいちょく」 "≡"は「ごうどう」 "∽"は「きごう」
●論理・集合:"⇔⇒∀∃∧∨¬∈∋⊆⊇⊂⊃∪∩"は「きごう」で変換
●等号・不等号:"≠≒<>≦≧≪≫"は「きごう」で変換
5にょにょ ◆yxpks8XH5Y :2009/01/01(木) 22:54:13 ID:???
  ∧_∧
 ( ´∀`)  <ヨン様
6ご冗談でしょう?名無しさん:2009/01/02(金) 00:31:25 ID:???
ttp://kissho.xii.jp/1/src/1jyou59942.png
内部抵抗r1,r2 起電力E1,E2の電池を並列に接続
このとき抵抗Rに流れる電流Iはどうなる?


E1 = IR + i1r1
E2 = IR + i2r2
I = i1 + i2
この3式を立ててやろうとしたんですが、Iが出てきませんでした。
どうしたらよいのでしょうか
7ご冗談でしょう?名無しさん:2009/01/02(金) 00:35:28 ID:???
月に行った宇宙飛行士がロズウェル事件の真相を再び語ったけど
実際にUFOが墜落して宇宙人を回収したの?
8ご冗談でしょう?名無しさん:2009/01/02(金) 00:37:07 ID:WLgPsRpe
出ます。
未知数がi1とi2とIの三つで、条件式が三つあるのでIは出ます。
9ご冗談でしょう?名無しさん:2009/01/02(金) 00:44:29 ID:Te8m3WMp
逆行列求めればOKじゃね?
10ご冗談でしょう?名無しさん:2009/01/02(金) 00:49:25 ID:???
>>8
落ち着いてやったらでました。
ありがとうございました。
11ご冗談でしょう?名無しさん:2009/01/02(金) 01:03:04 ID:???
出たなら計算式を書いてくれないかな・・・
12ご冗談でしょう?名無しさん:2009/01/02(金) 01:19:13 ID:Te8m3WMp
へー、君は解けなかったの?
13ご冗談でしょう?名無しさん:2009/01/02(金) 01:48:02 ID:vjRft/04
速さ0.5cで走っている列車に乗っている観測者が、列車の前方から速さ0.6cで走ってくる物体を観測した。
地上の観測者に対する物体の速さを求めよ。

お願いします。
14ご冗談でしょう?名無しさん:2009/01/02(金) 02:03:10 ID:???
>>13
>>1
>宿題を聞くときは、どこまでやってみてどこが分からないのかを書くこと。
>丸投げはダメだからね
15ご冗談でしょう?名無しさん:2009/01/02(金) 03:22:20 ID:V5PUxwld
ビッグバン理論の出発点
(どの理論から導出された理論なのか?)
16ご冗談でしょう?名無しさん:2009/01/02(金) 03:24:44 ID:???
>>12
ただ「でました」じゃ本当に解けたのかどうか分からない。

>>13
1.地上の観測者は列車と物体に対してどの位置にいるんだい?
2.列車は地上の観測者からどの方向に進んでるんだい?

それによって計算も答えも違ってくるんだが。
17ご冗談でしょう?名無しさん:2009/01/02(金) 03:31:42 ID:???
>>15
「現在、全ての天体同士が互いに遠ざかっているという観測結果」から。

その、いわゆる「現在、広がっている最中である」という事実を
「じゃぁ○○年前は?」「そのもっと前は?」と考えれば
どんどん全ての天体が1点に集中してくる様が思い浮ぶだろう。

もっと分かり易く言うと、
「現在広がっているのならば、過去に遡るほど縮んでゆくはずだ。」
ということ。

18ご冗談でしょう?名無しさん:2009/01/02(金) 04:07:12 ID:???
例えば
φ : x → 2x
という写像を考えた時、
φ^(-1)( (-∞,∞) ) = (-∞,∞)
だよ

現在の宇宙が有界でなければ、膨張しているからといって過去に1点であるとは限らない
(つまり定常宇宙論が一定の説得力を持つわけ)

ビッグバンが指示されているのは背景輻射等の実験事実をもっともよく説明しているから
理論的なバックボーンとしては、一般相対論や素粒子・核物理とある程度整合していること
(もちろんダークマターとか未解決問題はあるけど)
19ご冗談でしょう?名無しさん:2009/01/02(金) 04:45:46 ID:V5PUxwld
すいません。
wikiで調べてしましました。
つまりは>>17さんのおっしゃる通り
相対論のフリードマンモデルによってですよね。
20ご冗談でしょう?名無しさん:2009/01/02(金) 05:00:41 ID:V5PUxwld
重力ってなんでしょうかね?
(地球や火星のように物体として存在している天体の重力は感覚的に分かるのですが、
木星やBHのように物体として存在していない天体の重力は感覚的にわかりかねるので、
もしおわかりでしたらお願いいたします)
21ご冗談でしょう?名無しさん:2009/01/02(金) 05:11:40 ID:???
>>20
木星が物体ではないとは、何か物体の定義が普通と違うような気がするぞ。
22ご冗談でしょう?名無しさん:2009/01/02(金) 06:49:11 ID:V5PUxwld
>>21
自分のなかでは「物体=固体の形を持つもの」で「物質=分子構造をもつもの」
として区別しやすいように分けて考えているだけ
23ご冗談でしょう?名無しさん:2009/01/02(金) 08:15:21 ID:???
>>22
キミ小学生?
24ご冗談でしょう?名無しさん:2009/01/02(金) 12:45:46 ID:???
>>22

ということは、
液体、気体には重力が生じないという方が感覚的に納得いくということ?

だが、液体、気体も原子で出来ているわけだし、
その結合状態が変化するだけで重力が生じたり消えたりする宇宙方が
気持ち悪くない?
25ご冗談でしょう?名無しさん:2009/01/02(金) 14:38:14 ID:???
質問
物体の運動で量子効果ってどういうとき考えなきゃいけないの?
いまオレの側にある消しゴムはどういうときにシュレディンガー方程式で記述されるの?
26ご冗談でしょう?名無しさん:2009/01/02(金) 14:49:09 ID:Te8m3WMp
>>25
人間の手に負える時だけシュレディンガー方程式で記述されます。
27ご冗談でしょう?名無しさん:2009/01/02(金) 15:05:05 ID:???
光の屈折がいまいち分からないんだが
外部電磁場で媒質が分極し対称性が破れる→ゴルドストンボソン場が生じ、光子が影響を受ける→速度が落ちる
ってプロセスと考えていいの?
屈折してる光子はヒッグス場で速度が落ちた各種質量持ち粒子みたいに、静止系を導入できるの?
28ご冗談でしょう?名無しさん:2009/01/02(金) 16:11:21 ID:???
宇宙線だと重い原子核だからLHCとの比較にならないと聞きましたが、なぜでしょうか?
29ご冗談でしょう?名無しさん:2009/01/02(金) 16:13:29 ID:???
また来たぞ
30ご冗談でしょう?名無しさん:2009/01/02(金) 16:15:03 ID:???
>>29
なぜなのか聞きたいだけです。
31ご冗談でしょう?名無しさん:2009/01/02(金) 16:25:23 ID:???
32ご冗談でしょう?名無しさん:2009/01/02(金) 16:31:56 ID:V5PUxwld
>>23,24
そんなことはないですよ。
ただ実証として木星やBHには物体としての土壌が形成されていないのに重力が存在している事実を
どう認識されているかを聞きたいだけ。
33ご冗談でしょう?名無しさん:2009/01/02(金) 16:35:35 ID:???
320 名前:冗談です名無しさん[] 投稿日:2009/01/02(金) 03:16:00 ID:V5PUxwld
光の質量が0なのは空間を構成するヒッグス粒子からエネルギーを受け取りながら進むから(笑)


やっぱり小学生だな
34ご冗談でしょう?名無しさん:2009/01/02(金) 16:38:57 ID:???
>>25
どんなときでもシュレディンガー方程式で記述される。(揚げ足取りじゃなく、まじめな話ね。
量子力学が基礎法則で古典力学は量子力学からの近似だから。)

古典力学からのズレが現れるのは、物質波としてのドブロイ波長が運動のスケールと
同程度な場合とか、作用がプランク定数と同程度な場合。
というのが普通の教科書的な答えだけど、実際はもっといろいろあって、

http://www.amazon.co.jp/Decoherence-Appearance-Classical-Quantum-Theory/dp/3540003908/ref=sr_1_1?ie=UTF8&s=english-books&qid=1230881264&sr=1-1

てな本があるほど大きなトピックになってる。

>>32
物質の状態に関係なく、質量が重力を決める。(正確にはストレステンソルてなものが
重力を決めるけど、普通の天体では質量の効果が断然大きい)
35ご冗談でしょう?名無しさん:2009/01/02(金) 16:47:05 ID:V5PUxwld
>>33
はいはい。
どうせお遊びだし。

>>34
ありがとうございます。
てゆうか、そういう専門的な分かりにくいことはどうでもよくて
感覚的に宇宙空間という無重力空間では液体は当方均一的な外圧と液体自身が持つ表面張力との関係によって
球体をなし、その外圧と内部圧力によって中心に重力が発生する。そんな感じでいいんですよ。
36ご冗談でしょう?名無しさん:2009/01/02(金) 16:48:19 ID:???
>>31
487 名前:ご冗談でしょう?名無しさん[sage] 投稿日:2009/01/02(金) 16:45:53 ID:???
悪いが、ここには、アホしかいない。
ガッコの先生に聞いた方が確かだと思うぞ。

と言われた…
37ご冗談でしょう?名無しさん:2009/01/02(金) 16:51:23 ID:???
>>36
もう諦めろ、二度と来るな
38ご冗談でしょう?名無しさん:2009/01/02(金) 16:52:19 ID:???
誰かわかる人いませんかね?
>>37
諦めきれない
39ご冗談でしょう?名無しさん:2009/01/02(金) 16:53:21 ID:???
アホしかいないと思われ。
40ご冗談でしょう?名無しさん:2009/01/02(金) 16:54:27 ID:???
41ご冗談でしょう?名無しさん:2009/01/02(金) 16:55:13 ID:???
>>35
物質の状態に関係なく、質量が重力を決める。
42ご冗談でしょう?名無しさん:2009/01/02(金) 17:01:18 ID:V5PUxwld
>>41
そんなことは相対論や万有引力でわかっているから。
だけど感覚的に地に足が着いていない状態で重力が発生している状態をなぜって考えないかな?
43ご冗談でしょう?名無しさん:2009/01/02(金) 17:04:21 ID:V5PUxwld
>>41
だからそれは相対論や万有引力でわかっているから。
だけど感覚的に地に足が着いていない状態で重力が発生している状態をなぜって考えないかな?
44ご冗談でしょう?名無しさん:2009/01/02(金) 17:04:43 ID:???
>>42
お前は万有引力を何だと思ってる?
45ご冗談でしょう?名無しさん:2009/01/02(金) 17:05:37 ID:???
wikiによると宇宙線は陽子らしいですぜい。
あと原子核の結合エネルギーはMeVスケールだから無問題じゃないんですかい?

知らないけど。
46ご冗談でしょう?名無しさん:2009/01/02(金) 17:05:40 ID:V5PUxwld
二重投稿すんません。
パソコンの調子悪いさかい、堪忍な。
47ご冗談でしょう?名無しさん:2009/01/02(金) 17:06:54 ID:???
>>35
これだからまじめに答える気が失せる
48ご冗談でしょう?名無しさん:2009/01/02(金) 17:10:05 ID:V5PUxwld
>>47
でも本の受け売りしか言えないんでしょ。
49ご冗談でしょう?名無しさん:2009/01/02(金) 17:12:05 ID:V5PUxwld
>>44
そういうあなたはどうお考えですか?
50ご冗談でしょう?名無しさん:2009/01/02(金) 17:12:31 ID:???
>>48
はいはい。
どうせお遊びだし。
51ご冗談でしょう?名無しさん:2009/01/02(金) 17:14:35 ID:V5PUxwld
>>50
負けを認めるんですね。
52ご冗談でしょう?名無しさん:2009/01/02(金) 17:16:17 ID:V5PUxwld
>>49
言ってもいいですけど、
あなたのような人種は誹謗・中傷しか言わないですから
あなたからお願いします。
53ご冗談でしょう?名無しさん:2009/01/02(金) 17:18:27 ID:V5PUxwld
結局ここは自己の見解を持ってなく、
ただ誹謗・中傷するだけの場みたいですね。
54ご冗談でしょう?名無しさん:2009/01/02(金) 17:20:56 ID:V5PUxwld
結局はここは自己の見解を持たず誹謗や中傷をする場ですか?
55ご冗談でしょう?名無しさん:2009/01/02(金) 17:21:19 ID:???
質問者がひどいからこうなる。
56ご冗談でしょう?名無しさん:2009/01/02(金) 17:23:41 ID:???
はいはい。
57ご冗談でしょう?名無しさん:2009/01/02(金) 17:24:25 ID:V5PUxwld
結局ここは自己の見解を持たず誹謗・中傷をする場ですか?
58ご冗談でしょう?名無しさん:2009/01/02(金) 17:24:56 ID:???
物理やってるものからすれば、
系のマクロな状態によってミクロな物理法則が影響を受ける方が感覚的におかしいけどね

ID:V5PUxwldみてて、kikulogにもこんなやついたなあと思い出した
59ご冗談でしょう?名無しさん:2009/01/02(金) 17:25:18 ID:V5PUxwld
>>55
ですがその質問にも答えられないでいる
60ご冗談でしょう?名無しさん:2009/01/02(金) 17:26:35 ID:V5PUxwld
結局は専門的なことを知ったかぶっているだけでしょ
61ご冗談でしょう?名無しさん:2009/01/02(金) 17:27:12 ID:???
>>59
答えなら散々出てるだろうが文盲
62ご冗談でしょう?名無しさん:2009/01/02(金) 17:28:40 ID:V5PUxwld
>>58
でも量子力学ではヒッグス粒子というマクロ的な場を埋め尽くす粒子を既定しているみたいだけど
それはどう説明します?
63ご冗談でしょう?名無しさん:2009/01/02(金) 17:30:20 ID:V5PUxwld
>>61
はいはい。
でも答えられないでいて誹謗・中傷するほうがどっちがそうでしょうかねえ。
64ご冗談でしょう?名無しさん:2009/01/02(金) 17:34:08 ID:???
>>62
Higgs粒子がどう物理法則に関係してくるの?
「万有引力」の起源はエネルギーと運動量だから、
質量の有無と万有引力には直接の関係はないけど
65ご冗談でしょう?名無しさん:2009/01/02(金) 17:36:20 ID:V5PUxwld
>>64
宇宙空間という特殊な場の外圧としての効果?とか(笑)
66ご冗談でしょう?名無しさん:2009/01/02(金) 17:37:54 ID:???
質問は日本語でお願いします
67ご冗談でしょう?名無しさん:2009/01/02(金) 17:38:26 ID:V5PUxwld
疲れたんで寝ます。おやすみなさい。
68ご冗談でしょう?名無しさん:2009/01/02(金) 17:42:35 ID:???
>>64
>質量の有無と万有引力には直接の関係はないけど
物理の全歴史を否定されるのですか?
69ご冗談でしょう?名無しさん:2009/01/02(金) 17:50:02 ID:???
>>68
全否定するつもりがないことは「直接」という言葉に込めたんだけど

Higgs粒子が出てくるような話で、Newtonの重力理論を用いるのは不適切だろう
70ご冗談でしょう?名無しさん:2009/01/02(金) 18:39:34 ID:VKtT5Brj
自分の回答が分かりづらいのを棚に上げて質問者のせいに
する人間がいるんだ。

そんな人このスレには百害あって一利なしだから無理して
答えなくていいよ。
71ご冗談でしょう?名無しさん:2009/01/02(金) 18:50:54 ID:???
ここは1から10まで回答するスレじゃないから、そこは勘違いしないようにな
どちらかといえばヒントを与える趣旨
72ご冗談でしょう?名無しさん:2009/01/02(金) 18:52:24 ID:???
>>70
まあそりゃそうだが、今回のは質問者が悪いかと
73ご冗談でしょう?名無しさん:2009/01/02(金) 18:54:31 ID:???
どうみても質問者じゃないだろう。

単に人に難癖つけるための荒らし行為に質問の体裁とってるだけでしょ
だから皆もそんな奴は相手にしなきゃいいと思うよ。いくつかのキーワードとかIDとかをNGにするだけで
気持ちよく正月を過ごせると思うよ。
74ご冗談でしょう?名無しさん:2009/01/02(金) 18:54:35 ID:???
本日のお言葉。



       質問者が悪い



7573:2009/01/02(金) 19:01:33 ID:???
書くタイミングが悪くて>>72の「質問者が悪い」に>>73で「どう見ても質問者じゃない」と返答しているように
見えなくもないかもしれないが、これは「悪いのは質問者の方じゃない」の意味ではないので、>>72 がもし
気をわるくしてたらごめん。

違うことは続きの内容を読めばわかるとは思うが。念のため。
76ご冗談でしょう?名無しさん:2009/01/02(金) 19:03:09 ID:???
日本語OK
77ご冗談でしょう?名無しさん:2009/01/02(金) 19:36:59 ID:???
>>32
>実証として木星やBHには物体としての土壌が形成されていない

木星に関して質問。
物体としての土壌とはなにか?
何が実証なのか?
78ご冗談でしょう?名無しさん:2009/01/02(金) 19:57:48 ID:wuDxU3bZ
某歌聞いて思ったんだが、核融合炉ってメルトダウン起きるの?
起きるとしたら状況例を教えて欲しいんだけど
79ご冗談でしょう?名無しさん:2009/01/02(金) 20:45:53 ID:iMJRAHaN
あいす
80ご冗談でしょう?名無しさん:2009/01/02(金) 21:28:53 ID:qpv1Oz8q
熱を一方向にだけ通す物質はあり得ますか?
(熱伝導率に方向性を持たせることは出来ますか?)
81ご冗談でしょう?名無しさん:2009/01/02(金) 21:42:15 ID:???
>>80
マクスウェルの悪魔か?
82ご冗談でしょう?名無しさん:2009/01/02(金) 21:44:48 ID:???
>>81
それとは違うんじゃないか?ダイオードみたいなもんだろ。
83ご冗談でしょう?名無しさん:2009/01/02(金) 22:37:14 ID:???
新年のお言葉は「質問者が悪い」w
84ご冗談でしょう?名無しさん:2009/01/02(金) 23:42:11 ID:???
>>80
熱力学的には、条件に合わせて断熱壁と透熱壁を入れ替えればよいから、
特に禁止されてはないと思う

ミクロにはランダム運動を一方向に伝える物質というわけだから、
いろいろと考えてみたけどなさそうな気がするなあ
85ご冗談でしょう?名無しさん:2009/01/03(土) 00:18:07 ID:VdPQpsyS
断熱壁とバネつきの弁を使えば一方向だけの分子の流れを作り出せる
86ご冗談でしょう?名無しさん:2009/01/03(土) 00:37:05 ID:???
>>85
どうやって?
減衰するようなバネなら散逸のエネルギー考えないといかんが
87ご冗談でしょう?名無しさん:2009/01/03(土) 00:40:34 ID:???
ダンピングの「放熱」は高温側だけにフィードバックすればOK.
8873:2009/01/03(土) 00:43:12 ID:???
まあ頑張って第二種永久機関作ってくれ。
一生を棒に振るだろうけど、10^(-10^10) くらいの確率(適当)で救世主になれるかもしれないぞ。
89ご冗談でしょう?名無しさん:2009/01/03(土) 00:51:00 ID:???
>>85
それまさにマックスウェルの悪魔だから。
90ご冗談でしょう?名無しさん:2009/01/03(土) 00:58:19 ID:???
バネとダンパーは高温側に設置すれば、断熱壁で隔てられた低温側には熱が散逸することはない。
91ご冗談でしょう?名無しさん:2009/01/03(土) 01:06:17 ID:???
ブラウン運動を取り出すことができるスケールの弁で、
どうやって減衰するものを作るの?
92ご冗談でしょう?名無しさん:2009/01/03(土) 01:12:23 ID:???
電磁力を利用したダンピング機構を利用します。
93ご冗談でしょう?名無しさん:2009/01/03(土) 01:21:22 ID:???
それを具体的にどうすんのよ
電気抵抗使うにしても熱雑音無視できないでしょ
94ご冗談でしょう?名無しさん:2009/01/03(土) 01:23:51 ID:???
電磁気学で放射ゲージの存在の証明についてお伺いしたいのですが
φ_r、φ_Lをそれぞれローレンツゲージ、放射ゲージでのスカラーポテンシャルとして

φ_r = φ_L - ∂χ_0/∂t = 0

かつ

(1/c^2 ∂^2/∂t^2 - Δ)χ_0 = 0

を同時に満たすようなχ_0が存在することをどのように証明したらよいでしょうか?
95ご冗談でしょう?名無しさん:2009/01/03(土) 01:26:45 ID:???
連投すみません
放射ゲージなので真空中、ρ=0、j=0の仮定のもとでです
96ご冗談でしょう?名無しさん:2009/01/03(土) 01:27:33 ID:???
ゲージ変換について勉強すればすぐわかります。
97ご冗談でしょう?名無しさん:2009/01/03(土) 03:17:23 ID:???
>>80

広い意味でいいなら、ペルチエ素子とか。
98ご冗談でしょう?名無しさん:2009/01/03(土) 04:23:57 ID:???
映画を見ていたら 磁力は400年で効果が消える! 旨のセリフがあったのですが、本当ですか?
99ご冗談でしょう?名無しさん:2009/01/03(土) 04:44:05 ID:jzbqNrLV
細かいことやってますね。
100ご冗談でしょう?名無しさん:2009/01/03(土) 08:12:15 ID:???
難しいだろうけど、参考書読め
101ご冗談でしょう?名無しさん:2009/01/03(土) 08:13:57 ID:???
粒子 動
102ご冗談でしょう?名無しさん:2009/01/03(土) 08:59:59 ID:BrBtOXxJ
導体、半導体、絶縁体について質問です。
ふつう物質はそのバンド構造に則してこれらの場合分けがなされますが、
金属でない物質(共有結合、イオン結合)のバンド構造なんてどうやって
扱うのでしょうか。水素原子の正エネルギー軌道(参考ランダウ量子)の
様な物を使って波動関数を構成する結果なのでしょうか。それとも強引に
金属として計算してしまうのでしょうか。もし金属として計算するのなら
ばその波動関数は共有の項と金属の項の一次結合になっているのでしょう
か。よろしくお願いします。
103ご冗談でしょう?名無しさん:2009/01/03(土) 10:19:34 ID:DMm+WvzV
つぎのポテンシャル・エネルギーの場の中で、質量mの質点が運動しているとき
その振動の周期をエネルギーEの関数として表せ

U=ーU。/cosh^2 αx     −U。<E<0

cosh αx=tとおいて積分したらarcsinが出てきました
積分範囲は0から∞だと思うので解が求まりません
ランダウの力学の第3章§11の問題です
よろしくお願いします
104ご冗談でしょう?名無しさん:2009/01/03(土) 11:26:41 ID:???
ブラックホールって一瞬ですべてを吸い込むものじゃないんですか?教えてエライ人
105ご冗談でしょう?名無しさん:2009/01/03(土) 12:43:44 ID:???
一瞬じゃないよ。
BHの周囲の物質はBHの周囲を回りながらBHに近づいていく。
BHの周りには降着円盤といってBHに落ち込もうとする物体が
ものすごい密度で密集している。

ただ、一般相対論の教えによれば事象の地平線に近づくにつれ
時間が進むのが遅れてき、結局BH外の人には物体が事象の
地平線を割るのは永遠に見ることができない。

もちろん物体自身にとっては時間はふつーに流れて知らぬ間に
事象の地平線を割ってるんだけど、外部の人からは永遠に事象
の地平線に近づいているようにしか見えない。

結局外部の人がBHに物体が落ち込むのは観測できない。
106ご冗談でしょう?名無しさん:2009/01/03(土) 12:52:02 ID:UMOHcLcG
山内・杉浦の連続群論入門を読んでいる学生です。
表現の同値性について質問があります。

『V,Uを表現空間、群GからGL(V),GL(U)への表現を(ρ,V), (σ,U)とするとき
VからUの上への一対一の一次変換γが存在して、すべてのGの元gに対して
σ(g)γ=γρ(g)・・・(*)
を満たすとき、ρとσは同値であるという』

とあるのですが、(*)の部分がどうしても引っかかります。(*)の代わりに
σ(g)=γρ(g)・・・(**)
なるγが存在すればγは同型写像になるのではないですか?

(**)ではなく(*)が定義になっている所以をご教示ください。
107ご冗談でしょう?名無しさん:2009/01/03(土) 13:13:34 ID:tJoQlRkD
>>102 共有結合を作っている原子軌道をうまいこと重ね合わせて計算する
方法もあれば、「強引に金属として計算してしまう」方法(自由運動の
重ね合わせで考える)おある。
コンピューターの能力が低かったころは非金属に対しては前者の方が
よく用いられたが今では後者の方が優勢の様子。
もし金属として計算するのなら
>ばその波動関数は共有の項と金属の項の一次結合になっているのでしょう
>か。よろしくお願いします。
上で「金属的」に扱う、とはexp[ikx]の重ね合わせを用いて計算する、という
意味合いで使っている。フーリエ展開の話と同じで共有結合軌道だって
「金属的」な波動関数の重ね合わせで表現できるので君の質問は
ナンセンスになる。ちゃんと教科書を読み進めるのがいいと思う。
>103 −U。<E<0の時 E=-U。/cosh^2 αxを満たす有限のxがあると思うが?
ランダウの教科書にあるとおりやれば(今手元になし)積分範囲は0からこのx
までになるはず。
>106 γ V -> U、ρ V -> V、σ U -> Uなので、君の書いた
γρ V -> Uとなってσと一致しようがない。では教科書の方は
どうか?自分で書いてみて納得しよう。

108ご冗談でしょう?名無しさん:2009/01/03(土) 13:27:49 ID:???
>>105
もし地球上に小さなブラックホールができたらどうなりますか?
109ご冗談でしょう?名無しさん:2009/01/03(土) 13:28:47 ID:???
また来たぞ
110ご冗談でしょう?名無しさん:2009/01/03(土) 13:29:45 ID:???
>>109
前スレで結論出ましたっけ?
111ご冗談でしょう?名無しさん:2009/01/03(土) 13:34:27 ID:???
>>106
γ^{-1}σ(g)γ=ρ(g)
と書いたら分かりませんかね?
ρ(g1g2)=ρ(g1)ρ(g2)とconsistentであることも
チェックすればいいかと。
112ご冗談でしょう?名無しさん:2009/01/03(土) 13:34:57 ID:???
超高エネルギー宇宙線の成分ってなにからできてるのでしょうか?
113ご冗談でしょう?名無しさん:2009/01/03(土) 13:37:21 ID:k1qJ7ZaO
また来たぞ
114ご冗談でしょう?名無しさん:2009/01/03(土) 13:39:02 ID:???
>>105に科学的な反論もできないまま放置するなんてこのスレの回答者の
レベルも知れてますね。
115ご冗談でしょう?名無しさん:2009/01/03(土) 13:45:40 ID:???
>>114
放置じゃなく、思考停止てのが正しい。
要するに、解答しようにも分からないってこと。
116ご冗談でしょう?名無しさん:2009/01/03(土) 13:55:39 ID:???
反論?どこを?
117ご冗談でしょう?名無しさん:2009/01/03(土) 13:58:05 ID:???
>>113
また来たぞとしか言えないんですか?質問に答えてください。
118ご冗談でしょう?名無しさん:2009/01/03(土) 14:00:09 ID:???
下のスレを見るといいですよ。
ttp://science6.2ch.net/test/read.cgi/sci/1222489174/l50
119ご冗談でしょう?名無しさん:2009/01/03(土) 14:01:04 ID:???
>>118
ありがとうございました。
120ご冗談でしょう?名無しさん:2009/01/03(土) 14:08:33 ID:???
>>103
数学公式見たら、1/cosh^2 x の不定積分は tanh x になってたぞ。
それから、積分範囲は図6のX1、X2 だろ。
121ご冗談でしょう?名無しさん:2009/01/03(土) 14:18:18 ID:DMm+WvzV
>>120
そんな公式があったのですか
知りませんでした

Uは偶関数ですから0からある値xまでが積分範囲でいいんですよね?
時間はそれの2倍だから(11-5)の式がそのまま使えると思ったのですが
122ご冗談でしょう?名無しさん:2009/01/03(土) 14:28:27 ID:???
>>121
いや公式というより本だ。岩波全書の数学公式1。
いま自分でやってみる時間がないので勘で書いたのだが、0から無限大だと周期も
無限大になってしまいそうだから、まずあり得ないんじゃないかと。
1/cosh^2 x のグラフは指定された範囲で下に凸になってる?それを確認しないと。

あとでやってみて何かわかったらまた書く。レスがなかったらギブアップと思ってくれw
123ご冗談でしょう?名無しさん:2009/01/03(土) 14:38:23 ID:DMm+WvzV
>>122
わかりました
私もグラフの形が良く分からないまま(ざっとエクセルで計算しましたが)です
∞のこともその通りだと思います
ありがとうございました


124ご冗談でしょう?名無しさん:2009/01/03(土) 14:47:58 ID:???
素粒子物理では何で群論を使うの?
125ご冗談でしょう?名無しさん:2009/01/03(土) 14:51:10 ID:???
>>124
対称性を扱うのに便利だから
固体物性でも結晶構造の分類に使ったりする
126ご冗談でしょう?名無しさん:2009/01/03(土) 15:18:31 ID:???
じゃあ素粒子の運動は波動関数で記述して、分類に群論を使うのね。
127ご冗談でしょう?名無しさん:2009/01/03(土) 15:35:44 ID:BrBtOXxJ
>>107 どうもありがとうございます。しかし今ひとつ解らないことがあるのです。
ただ私の整理がついていないだけなのかもしれませんが今一度お付き合い下さい。
共有結合的なるものと金属的なるものを分けたのはそのエネルギーの正負を意識
してのことです。この観点から知りたいことは、共有結合を自由電子の波動関数
で表現できるかではなく(気づいてなかったけど)、波動関数を構成する時に正
のエネルギーと負のエネルギーが入り交じるけれども感覚的にそれでいいのかと
いうことなのです。よろしくお願いします。
128ご冗談でしょう?名無しさん:2009/01/03(土) 15:59:36 ID:???
東大でブラックホール実験やったから、尋ねたらいい
129ご冗談でしょう?名無しさん:2009/01/03(土) 16:53:15 ID:BrBtOXxJ
>>127補足
感覚的にそれでいいのかと言われても困ると思うので少し補足しておきます。
一般の分子理論の原子価結合法では近似を良くするためにイオン項を手で取
り除きますよね。こんな風に正エネルギーと負エネルギーが入り交じっても
近似が悪くなることはないのかと言う意味です。
参考:ガシオロウィッツ量子力学
130ご冗談でしょう?名無しさん:2009/01/03(土) 18:26:12 ID:???
子供のころからの疑問です。教えて下さい。
人差し指を立てて床にゆっくり近ずいていきます。この時、床と指の間には
無限に小さくなっていく距離があるわけでしょ?あと1mm あと0.1mm あと
0.0000001mm 0.000000000000000000000000001mm 0.0000000000・・・
たしかに小さくなっていく距離を通過しているのに、指が床に触れた瞬間
0になる。無限に小さくなっていくのに0になるのはなぜ?
アキレスと亀っぽいけど、こちらのほうが疑問です。
131ご冗談でしょう?名無しさん:2009/01/03(土) 18:38:59 ID:???
虫眼鏡で観察しよう。考えるのその後だな。
13273:2009/01/03(土) 18:40:32 ID:???
>>130
アキレスと亀はゼノンのパラドクスの一つだけど、それもゼノンのパラドクスの一つ (矢は的に当たらない) だな。

指と床の距離が 0 になるまでの間に、あと1mm、あと0.1mm、あと0.01mm…が無数にある、というだけの話。
133ご冗談でしょう?名無しさん:2009/01/03(土) 18:41:13 ID:???
ああ、なんか意味もなく73が残ってるな。
134120 122:2009/01/03(土) 18:56:00 ID:???
>>123
すまん、ぼけてた。必要なのは 1/cosh^2 x の不定積分じゃないよな。
グラフの形状は確認した。今から計算できるかどうかやってみる。
135ご冗談でしょう?名無しさん:2009/01/03(土) 19:11:04 ID:???
ガンマ線バーストが発生する確率はどれくらい低いのでしょうか?
136ご冗談でしょう?名無しさん:2009/01/03(土) 19:14:59 ID:J6EjWjDD
おいおい…。
朝からずっと何やってんだよ。
1/cosh^2xは解析的に解ける数少ないポテンシャルだぞ…。
137ご冗談でしょう?名無しさん:2009/01/03(土) 19:19:44 ID:???
>>131
無限に拡大できる虫眼鏡なら床に触れねーな。
138ご冗談でしょう?名無しさん:2009/01/03(土) 19:45:41 ID:???
まず、高性能の虫眼鏡の開発から始めるのがベストかね。
139120,122,134:2009/01/03(土) 20:01:19 ID:???
>>136
それじゃあとは頼む。よろしく。
140121:2009/01/03(土) 20:53:51 ID:???
ちょww
助けて下さいよ

>>36
∫{dx/√(E-cosh^2 αx)} を求めたとき現れる関数形だけでも
教えてもらえませんか
arcsinは間違いでしょうか?
141ご冗談でしょう?名無しさん:2009/01/03(土) 20:55:04 ID:???
間違えた
>>36ではなく>>136のかたへ
142ご冗談でしょう?名無しさん:2009/01/03(土) 21:42:55 ID:???
フォトンベルトってあるの?
143ご冗談でしょう?名無しさん:2009/01/03(土) 21:48:31 ID:???
>>142 お前の心の中にな
144ご冗談でしょう?名無しさん:2009/01/03(土) 21:50:24 ID:???
>>143
実際には存在しないの?科学的に否定できるの?
145ご冗談でしょう?名無しさん:2009/01/03(土) 22:06:45 ID:???
146ご冗談でしょう?名無しさん:2009/01/03(土) 22:08:10 ID:???
>>145
なるほど。存在しないのか
147ご冗談でしょう?名無しさん:2009/01/03(土) 22:31:00 ID:???
>>145
見た限り否定も肯定もできんな。実際どうなんだろうか?
148ご冗談でしょう?名無しさん:2009/01/03(土) 22:44:24 ID:???
>>147 君がそう思いたいのであればそう思っていたまえ。
149ご冗談でしょう?名無しさん:2009/01/03(土) 22:45:33 ID:???
>>147
肯定的な証拠は何一つ無いな。
150ご冗談でしょう?名無しさん:2009/01/03(土) 22:47:00 ID:???
>>149
そりゃそうだが…
151ご冗談でしょう?名無しさん:2009/01/03(土) 22:51:54 ID:???
フォトン・ベルトか。
2012年末ね。
初めて知った。覚えておこう。
152ご冗談でしょう?名無しさん:2009/01/03(土) 23:19:04 ID:???
波束が収束するって具体的にどういうケースに起こるんですか?
どんな例でもいいんで教えてもらえないでしょうか
153ご冗談でしょう?名無しさん:2009/01/04(日) 00:22:46 ID:???
宇宙には巨大な汲み取り便所があってな。それが銀河を周回するあいだに垂れ流された糞便が
帯状に分布しているわけよ。これをポットンベルトという。なんか名前を間違って流布されてるみ
たいだけどな。

2012には地球がこのポットンベルトに突入することになる。人類は糞にまみれて絶滅だな。

…とまあ、こんな思い付きの出鱈目でも「科学では絶対はない」以上、絶対にない、と否定する
ことはできないわけよ。フォトンベルトもその程度の話だと思っとけ。
154ご冗談でしょう?名無しさん:2009/01/04(日) 03:18:44 ID:???
>>153科学界の最重要機密を簡単に漏らすもんじゃないぞ。
一般人が絶望するじゃないか!!君も知っているように
糞除去装置はもうほとんど出来かけているのだから、完成後
公表すればいいのだ。
155ご冗談でしょう?名無しさん:2009/01/04(日) 04:59:47 ID:ejpj4HqI
156ご冗談でしょう?名無しさん:2009/01/04(日) 05:18:12 ID:GDSehU71
円運動の話なんだけど、なんで向心加速度は速度と角速度の積になるのか教えてくさい
なんか先生が内積を利用すればわかるとかいってたけど、いまいちわかりませんでした
157ななし:2009/01/04(日) 06:54:44 ID:C7QJxUv8
フォトンベルトって・・・誰じゃこのなまえつけたひとw
158ご冗談でしょう?名無しさん:2009/01/04(日) 07:56:19 ID:c4X9xGcL
>>156心配するな、君の先生の言う事が理解できる人間は
殆どいないよ。で、君が外積と内積を聞き違えた可能性は
ないのかな?
 本題に戻ると素直に等速円運動における加速度を計算してそれで
納得するのが一番だと思う。それがvωに比例するのは次元解析ですぐに
でるけど無次元比例係数が1になるのはまともに計算するのが一番。
>>127
返事が遅くなりました(酒飲んで寝て、また今飲んでたりする)。で、
君の書いた文章見ると本来区別すべき事柄がごっちゃになっているように
見える。正負のエネルギー云々とイオン項云々の所。その前のレスから
察するに正負のエネルギー云々って束縛状態かそうでないか、の区別を
言っているように見えるがバンド計算においてはそのような事は考慮に
入れません。それでいいのか?表面近くならともかく物質内部において
は電子が外部の影響を受ける事はあまり無いだろう、という考えを認めれば
束縛状態とそれより上の 状態を構成する波動関数の区別をする必要は
無くなる。そして現実的にはバンド計算と言うものはフェルミレベルから
上下1、2のバンド部分だけをきちんと捉えよう、とするものなのでそれ
より大きな励起状態についてはそもそも考えていない、と思う。(計算
すれば形式的には答え出て、論文なんかでもグラフがでてるが。なお当方は
バンド計算は非専門)
159ご冗談でしょう?名無しさん:2009/01/04(日) 12:09:40 ID:GDSehU71
>>158
>>156心配するな、君の先生の言う事が理解できる人間は
殆どいないよ。で、君が外>外積
>次元解析
>無次元比例係数
わかりません('A`)
160ご冗談でしょう?名無しさん:2009/01/04(日) 12:38:04 ID:???
>>156
とりあえず、向心加速度の大きさだけなら、こう考えればよい。

半径r, 角速度ω等速円運動の「速度ベクトル」を表す矢印を、始点を固定して書けば、矢
印の先端は半径 |v| = rω (等速円運動の速さ) の円を描くことがわかるだろう。そしてそ
の角速度 (回転の速さ) はもとの円運動の角速度 ω と同じだ (回転の位相は90°ずれ
てるが)。

つまり、「速度の変化」は半径 |v| = rω で、角速度ω の等速円運動なわけだ。その
「速度の変化の速さ」は当然 |v|ω = rω^2 になる。「速度の変化の速さ」は加速度の大
きさに他ならない。

あと、加速度の方向は、

  速度ベクトルは位置ベクトルと 90°ずれてる

ことが、「速度ベクトルの回転」についても同じことが言えるので

  加速度ベクトルは速度ベクトルと 90°ずれてる
   = 加速度ベクトルは位置ベクトルと 180°ずれてる

ことになり、加速度の方向は位置ベクトルの反対、つまり中心方向であることがわかる。

ややこしい事を書いているように思えるかもしれないが、図を描いてみればすぐにわかるだろう。
161106:2009/01/04(日) 12:39:57 ID:NcvH53Zv
>>111
ありがとうございます。頑張ってみます
162159:2009/01/04(日) 13:02:36 ID:GDSehU71
ミスです
正しくは
>>158
>次元解析
>無次元比例係数
わかりません('A`)
でした

>>160
なるほど!!!
理解できました
ありがとう!!
163ご冗談でしょう?名無しさん:2009/01/04(日) 13:03:57 ID:???
>>152
君が観測した時に収束する。
君がじーっと睨んでたら毎瞬間に収束しつづける。
164ご冗談でしょう?名無しさん:2009/01/04(日) 13:13:18 ID:XB73qf12
163
不確定性原理とかいうのと関係あるの?
165ご冗談でしょう?名無しさん:2009/01/04(日) 13:47:54 ID:4z2mxLtX
>>158
ありがとうございました。
あれこれ勉強します。
166ご冗談でしょう?名無しさん:2009/01/04(日) 13:49:02 ID:???
頼む教えてくれ

電場があると地場が発生するらしいが、電磁波にならない波なんてあるのか?
167ご冗談でしょう?名無しさん:2009/01/04(日) 13:51:47 ID:???
媒質中の電磁気を考えるときに真空中と違って
二つのmaxwell反対称四元テンソル(EBのとDHの)が必要になることの物理的意味がわかりません
ある種の双対?新しいゲージ対称性が出るわけではないよね・・
168ご冗談でしょう?名無しさん:2009/01/04(日) 14:39:02 ID:???
>>166
地場?磁場じゃないの?
電場がある場合に、必ず磁場も発生するわけじゃないけど
169ご冗談でしょう?名無しさん:2009/01/04(日) 15:33:15 ID:???
>>167
物質中では微視的Maxwell方程式と全荷電粒子のLorentz力の連立方程式解けたとすれば
(変数はE, Bと各荷電粒子の位置)それで十分だけど、
そんなの不可能だから、背景として扱いたい荷電粒子の寄与をPとMにまとめてしまうわけ
つまり、E, B, P, Mの4変数を扱う巨視的Maxwell方程式を解けばいいんだが、
PとMを生で扱うよりEとHにしてしまった方が簡単だし、思考の経済化にもなる
物理的意味はそういうこと
170ご冗談でしょう?名無しさん:2009/01/04(日) 16:53:31 ID:???
>>168
すまん磁場だ。
では電場の波だけが存在するの?
171ご冗談でしょう?名無しさん:2009/01/04(日) 16:57:01 ID:???
点光源に依って出来る影を消すために色々考えてみたんだが

1、点光源と光を遮る物体を、内面が鏡で出来た球で覆う
2、点光源からの光を霧など微細な氷粒で乱反射させ、光を遮る物体まで届かせない

この2つは正解?
現実的でなくとも構わないから、他に方法あれば教えて下しあ
172ご冗談でしょう?名無しさん:2009/01/05(月) 08:00:31 ID:???
量子力学における角運動量演算子Jについての質問です。

J±が状態|α, β>に作用するとαは変えずβのみを±1変化させるとのことですが、
どうしてそう言えるのかわかりません。
http://www-nuclth.phys.sci.osaka-u.ac.jp/wakamatu/quantum_mechanics2/Lecture_Note/lectqm2_02.pdf

このPDFファイル(重くて申し訳ない)の4ページでは、観測1と観測2
より導けるというように書いてありますが、この2式からだと、
J^2とJzの固有値がhα、h(β-1)ということしかわからない気がするのですが・・・・
173ご冗談でしょう?名無しさん:2009/01/05(月) 17:13:25 ID:???
>>172
> J^2とJzの固有値がhα、h(β-1)ということ
どの状態が↑こうなるということがわかったの?
174ご冗談でしょう?名無しさん:2009/01/05(月) 19:21:51 ID:EpJP/LLH
空間リズムを変えて、進むことはできますか?アルゴリズムでもいいけど。
175ご冗談でしょう?名無しさん:2009/01/05(月) 19:25:17 ID:EpJP/LLH
,過去レスのは、ゲージを使えばできるよ。
176ご冗談でしょう?名無しさん:2009/01/05(月) 20:18:05 ID:???
>>170
波と言うのが動的に伝播する電磁場を指すなら、電場のみのものは存在しない。
静電場で磁場が無い状態は有り得る。
177ご冗談でしょう?名無しさん:2009/01/05(月) 20:39:51 ID:???
いかな静電場でもローレンツ変換を施せば磁場が出てくる
んじゃないの?
178ご冗談でしょう?名無しさん:2009/01/05(月) 20:58:01 ID:???
縦波をああ描くのはなんか違和感がある
179ご冗談でしょう?名無しさん:2009/01/05(月) 21:02:51 ID:???
>>177
まぁそれはそうだ。
あらゆる系から見て、という条件ならば、静的な場合も含めて電場のみの状態は存在しない。
180ご冗談でしょう?名無しさん:2009/01/05(月) 21:07:23 ID:???
>>177
「点電荷が静止している状態」と「点電荷が等速直線運動している状態」を同じ状態とみなす、という分類をするならそもそも「静」電場が定義できなさそうだが、そういう意味?
181ご冗談でしょう?名無しさん:2009/01/05(月) 21:13:11 ID:???
世の中に静電場がなくなったら何か困るか?
182ご冗談でしょう?名無しさん:2009/01/05(月) 21:17:39 ID:???
別に…
183ご冗談でしょう?名無しさん:2009/01/05(月) 21:19:57 ID:???
実害がないならいいじゃん。
184ご冗談でしょう?名無しさん:2009/01/05(月) 21:53:36 ID:EpJP/LLH
精電場なくなったら、電波塔建たないじゃん。
185ご冗談でしょう?名無しさん:2009/01/05(月) 21:56:13 ID:EpJP/LLH
携帯にごくわずかだが、静的な場合も含めて電場のみの状態は存在するよ。じゃないと、携帯から電波発信しないじゃん
186ご冗談でしょう?名無しさん:2009/01/05(月) 22:01:57 ID:EpJP/LLH
それより、電磁波をどう抑えるかが大変なんだよ。
187ご冗談でしょう?名無しさん:2009/01/05(月) 22:11:14 ID:???
だれか納得ゆく説明できるエロい人いないかね
188ご冗談でしょう?名無しさん:2009/01/05(月) 22:15:44 ID:EpJP/LLH
企業の参考書読め!!
189ご冗談でしょう?名無しさん:2009/01/05(月) 22:24:18 ID:???
>>184-185
全く意味不明。

ちなみにプラズマ中には磁場を伴わない波 (静電波) があるそうな。
といってもプラズマの振動を伴う波だから電場だけの波とは言えないがな。
190ご冗談でしょう?名無しさん:2009/01/05(月) 22:38:09 ID:???
禅問答だなまるで。
191ご冗談でしょう?名無しさん:2009/01/05(月) 22:56:31 ID:???
電場のみ、磁場のみの波は存在しないでFA?
192ご冗談でしょう?名無しさん:2009/01/05(月) 23:43:36 ID:???
「波」と言ってる時点で電場は時々刻々と揺らいでいるので、マクスウェルの
方程式に従えば、どうしても磁場が発生してしまう。

磁場も「波」うつ時点で場が時々刻々と揺らいでいるので、マクスウェルの方程式
に従って、電場の発生は避けられない。

ということは、分かるな?
193ご冗談でしょう?名無しさん:2009/01/05(月) 23:52:26 ID:???
どうして?
194ご冗談でしょう?名無しさん:2009/01/06(火) 07:58:36 ID:???
>>191 >>192
そうだな。
ガウス単位系では電場と磁場がおなじ単位で表されるが、それだと電磁波では電場の振幅と
磁場の振幅が同じ大きさだということがわかる。
ローレンツ変換してもこの関係と偏光の向きは変わらない。
195ご冗談でしょう?名無しさん:2009/01/06(火) 08:53:50 ID:???
円錐台のz軸方向に関する慣性モーメントがわかりません。
公式などがあれば助かります。
わかる人がいたら教えてくれませんか?
196ご冗談でしょう?名無しさん:2009/01/06(火) 08:56:44 ID:???
>>194
つまり、電場と磁場は同じ物ということですか?
197194:2009/01/06(火) 09:07:32 ID:???
>>196
そんなことは一言も書いてないが。
198ご冗談でしょう?名無しさん:2009/01/06(火) 09:15:28 ID:???
>>195 素直に積分しろよ
199ご冗談でしょう?名無しさん:2009/01/06(火) 09:18:48 ID:???
>>195
>>198 に補足すると、円盤の慣性モーメントがわかってればあとは簡単。
200ご冗談でしょう?名無しさん:2009/01/06(火) 09:24:30 ID:???
>>199 ありがとうございます。積分は苦手なのですが
    もう少しがんばってみます。
201ご冗談でしょう?名無しさん:2009/01/06(火) 12:20:07 ID:???
充電
というものを分かり易く説明して下さい
電気は貯めておくことは出来ないと聞いたことがありますが
じゃあ電池やバッテリーはなんなのかと…
簡単な理屈が分かれば良いので誰か様お願いします
202ご冗談でしょう?名無しさん:2009/01/06(火) 12:23:11 ID:???
>>201
貯めておくことができないというのは、そのままの形でという意味。
違うエネルギーに変換すれば貯めておける。
203ご冗談でしょう?名無しさん:2009/01/06(火) 12:23:34 ID:???
まずググれ。
204ご冗談でしょう?名無しさん:2009/01/06(火) 12:24:02 ID:???
化学反応をためてるとでも思えば良いが、小学生には難しいかな?
205ご冗談でしょう?名無しさん:2009/01/06(火) 12:42:47 ID:MMwtOnOn
よく量子エンタングルの論文に出てくるcoincident countて同時測定って意味ですか?
そう訳すとあまりピンとこないのですが‥‥
206ご冗談でしょう?名無しさん:2009/01/06(火) 13:41:54 ID:???
>>103
ランダウの力学、§11の問題。
ポテンシャルの形が左右対称なので周期は
T = 4√(m/2)∫1/√(U0/cosh^2(αx) - |E|)dx、積分区間は0からU(x)=Eの解となるxまで。
sinh αx = t とおくと、a = √(U0/|E| - 1)として
T = 4√(m/2)/(α√|E|)∫[0,a]1/√(a^2 -t^2)dt
= 4√(m/2)/(α√|E|) [arcsin t/a] (t = 0, a)
= 4√(m/2)/(α√|E|) π/2
= (π/α)√(2m/|E|)
207ご冗談でしょう?名無しさん:2009/01/06(火) 14:25:57 ID:???
>>205
俺の翻訳マシンでは計算と出たな
208ご冗談でしょう?名無しさん:2009/01/06(火) 14:28:24 ID:???
ひどいね。
209ご冗談でしょう?名無しさん:2009/01/06(火) 16:53:27 ID:???
>>205
同時測定でおk。Quantum Eraserなどの論文で出てくるな。
同時測定というか、同所測定というか。詳しくはPhy Rev Letteなどで量子消去を漁ってみるとたくさん出てくるぞ。
210ご冗談でしょう?名無しさん:2009/01/06(火) 17:33:24 ID:FwlioN6O
ホーキング虚時間の宇宙 (竹内薫)を読むとホーキングって振動宇宙論が
すきそうなのだけれども、振動宇宙って解はアインシュタイン方程式や
宇宙方程式の解として存在してたっけ?学生時代の記憶ではそのような
解を見た記憶ないんですけども。
211ご冗談でしょう?名無しさん:2009/01/06(火) 17:37:37 ID:???
宇宙モデルなんて、状態方程式次第でどうにでもなる。
212ご冗談でしょう?名無しさん:2009/01/06(火) 17:52:45 ID:???
μを透磁率、Bを磁束密度として

μBはエネルギーの次元をもちますか?
213ご冗談でしょう?名無しさん:2009/01/06(火) 18:11:46 ID:???
>>206
ありがとうございます!!
積分範囲の間違いは気づいてました
やっぱりarcsinで良かったんだ
今夜やろうと思っていたのですが、すっきりしました
お世話になりました
214ご冗談でしょう?名無しさん:2009/01/06(火) 18:14:58 ID:FwlioN6O
>>211 宇宙モデルが状態方程式次第でどうにでもなるものならば、困ったことが起こります。
岩波新書の宇宙論入門(佐藤勝彦)に振動宇宙は特異点定理によって禁止される旨が書かれています。
特異点定理が一般相対論の一つの帰結ならば、振動宇宙も相対論の結果の一つです。
佐藤勝彦が正しいならばこれらは互いに矛盾しあいます。
これの意味するところは一般相対性理論の内部矛盾です。
これは困った。
215ご冗談でしょう?名無しさん:2009/01/06(火) 18:21:48 ID:???
>>214
特異点定理は、状態方程式にある条件を課して導かれるものに過ぎない。
216ご冗談でしょう?名無しさん:2009/01/06(火) 18:31:37 ID:synQnNvm
高いところにいる人は低いところにいる人より地球の自転によって
速く動いているから早く歳をとるって本当ですか?
217ご冗談でしょう?名無しさん:2009/01/06(火) 18:45:05 ID:???
>>212
持たない。μが磁気モーメントなら話は別だが。
218ご冗談でしょう?名無しさん:2009/01/06(火) 18:45:15 ID:ZL2cdFTq
今のところ、ビッグバンは無から生じた、ということでいいのでしょうか?

219ご冗談でしょう?名無しさん:2009/01/06(火) 18:45:56 ID:???
何をもって無とするかによるが・・・。
220ご冗談でしょう?名無しさん:2009/01/06(火) 18:48:12 ID:???
いんや、何も分かっていない。
221ご冗談でしょう?名無しさん:2009/01/06(火) 18:48:45 ID:???
>>216
高いところの方が速く年を取る(正確には時間経過が速くなる)のは本当だが、
理由は動いているからではなく、重力ポテンシャルの高いところにいるから。

動いているせいならむしろ、時間経過は遅い。
222ご冗談でしょう?名無しさん:2009/01/06(火) 19:35:05 ID:???
>>217
ありがとうございます
223ご冗談でしょう?名無しさん:2009/01/06(火) 19:38:45 ID:???
ブラックホールの中心は無重力?
224ご冗談でしょう?名無しさん:2009/01/06(火) 19:56:30 ID:FwlioN6O
>>215つまり付加条件どうしが矛盾してるにすぎないのですか。
ありがとうございました。
225ご冗談でしょう?名無しさん:2009/01/06(火) 20:02:59 ID:???
構成要素が何個以上集まれば、熱・統計力学的な性質が現れるのですか?
226ご冗談でしょう?名無しさん:2009/01/06(火) 20:08:26 ID:???
ブラックホールの近くにブラックホールが発生したらどうなるの?
227ご冗談でしょう?名無しさん:2009/01/06(火) 20:12:24 ID:???
LHCの実験はやってみないとわからないのが真実ですか?
228ご冗談でしょう?名無しさん:2009/01/06(火) 20:20:37 ID:???
力学の初歩に出てくる
v_1^2 - v_0^2 = 2α(x_1-x_0)
という式がありますが、
この式って物理的な意味はどういうことなんでしょうか
形式的な意味合いしかないのですか?
229ご冗談でしょう?名無しさん:2009/01/06(火) 20:24:33 ID:???
>>228
式に出てくるそれぞれの量の物理的な意味はどういうことなんでしょうか
230ご冗談でしょう?名無しさん:2009/01/06(火) 20:24:41 ID:???
m/2を掛けて見れ。

mv_1^2/2-mv_0^2/2=mα(x_1-x_0)=F(x_1-x_0)
つまり1次元運動では一定の力を加えた時の仕事が運動エネルギーの差だという式に過ぎない。
231ご冗談でしょう?名無しさん:2009/01/06(火) 20:25:32 ID:???
>>229
mαなら力とか、1/2 mv^2ならエネルギーとかありますが、
速度の自乗って何か意味あるんですか?
232ご冗談でしょう?名無しさん:2009/01/06(火) 20:26:20 ID:???
>>230
なるほどー
速度の自乗自体に意味はないんですね
233ご冗談でしょう?名無しさん:2009/01/06(火) 20:26:58 ID:???
>>231
m/2はm/2を無次元化して単位の方に放り込んだと考えれば、
単なるエネルギー保存即になる。

単位の事は分かるよね?
234ご冗談でしょう?名無しさん:2009/01/06(火) 20:27:29 ID:???
>>232
どういう意味で意味は無いって言ってるの?
235ご冗談でしょう?名無しさん:2009/01/06(火) 20:28:49 ID:???
>>233
把握しました

力を加えたぶんがエネルギーの差額として現れると理解しましたが
それでいいんでしょうか
236ご冗談でしょう?名無しさん:2009/01/06(火) 20:29:40 ID:???
>>229
お前みたいなカスって初学者いじめて何が楽しいの
237ご冗談でしょう?名無しさん:2009/01/06(火) 20:29:45 ID:???
>>235
良いと思うよ。
別にこの式をそれ以上深く考察しても構わないけど、
それは好きにすれば良いと思う。
238ご冗談でしょう?名無しさん:2009/01/06(火) 20:30:34 ID:???
>>237
ありがとうございました!すっきりしました!
239ご冗談でしょう?名無しさん:2009/01/06(火) 20:38:53 ID:???
電子1個では超伝導は起こらないのでしょうか?
240ご冗談でしょう?名無しさん:2009/01/06(火) 20:54:51 ID:???
ONEPIECEの月歩は実現可能?
241ご冗談でしょう?名無しさん:2009/01/06(火) 21:01:38 ID:???
>>239
ごめん意味が分からないので詳しく
242213:2009/01/06(火) 21:30:53 ID:???
>>206
>>103の問題を教えていただいたおかげで
次の問題は簡単にできました
あらためて、ありがとうございましたっ(^o^)
243206:2009/01/06(火) 21:35:06 ID:???
>>242
俺も勉強になった。こんな計算やるの数十年ぶり。ボケ防止だよw
244ご冗談でしょう?名無しさん:2009/01/06(火) 21:47:28 ID:???
>T = 4√(m/2)∫1/√(U0/cosh^2(αx) - |E|)dx
この積分はシュレディンガー方程式からダイレクトに出てくるの?
245ご冗談でしょう?名無しさん:2009/01/06(火) 21:55:04 ID:???
>>244
これは古典力学だよ。E=(m/2)(dx/dt)^2 + V(x) から直ちに出る。
246ご冗談でしょう?名無しさん:2009/01/06(火) 22:09:03 ID:???
>>241
伝導電子が1っ子しかない物質(勿論損なのは頭の中にしかないけど)
ではクパーペアは組めないので絶対に超伝導にならないのかなと・・・
247ご冗談でしょう?名無しさん:2009/01/06(火) 22:27:54 ID:???
原子・分子の中の電子は永久運動してるから、
ミニミニ超伝導じゃないの?
248ご冗談でしょう?名無しさん:2009/01/06(火) 22:35:08 ID:???
>>247
んなアホな。
249ご冗談でしょう?名無しさん:2009/01/06(火) 22:46:38 ID:???
どこが?
250ご冗談でしょう?名無しさん:2009/01/06(火) 23:37:58 ID:???
>>248
まだこんな馬鹿がいるのか
251ご冗談でしょう?名無しさん:2009/01/06(火) 23:40:58 ID:YgSzL36G
気になって仕方ないので教えてください。
文章力に乏しいので、ご容赦願います。

光を100%反射する物質(あるかどうかはわかりませんが)で
中身が空洞な正六面体を作ったとします。
その空洞の中に、光を照射して光を外部に逃さないまま
六面体を閉じることができたとしたら、光は六面体の中で
永久に反射し続けることができるのでしょうか?

上手く説明できなくてすみません。
どなたか教えてください。

252ご冗談でしょう?名無しさん:2009/01/06(火) 23:43:07 ID:???
>>225
熱力学的ということに関して統計力学での一つの観点は、統計値として算出する熱力学量の
偏差が十分小さいかどうかだろうな。
これは一般には注目している系、物理量と、偏差がどの程度なら十分小さいと見るかによる。

例えばカノニカルアンサンブルでの平均エネルギー(内部エネルギー)の場合、その偏差は
ΔE/<E> 〜 O(1/√N)   ( ΔE=√<(E-<E>)^2>、N:粒子数 )
だから、偏差1%以下を許容するなら大体10^4個以上。
253ご冗談でしょう?名無しさん:2009/01/06(火) 23:49:06 ID:???
>>251
光を100%反射する物質があるとする仮定ならその通りだが、
光を100%反射する物質なんて無い。
254ご冗談でしょう?名無しさん:2009/01/06(火) 23:52:32 ID:???
>>251
そんなのが昔ドラえもんにあったな
255ご冗談でしょう?名無しさん:2009/01/07(水) 00:02:24 ID:???
251です。返答をいただき、ありがとうございます。

では、現在あるような光を反射する材料(鏡とか)で
前述したような実験を実施できたとしたら、どのような結果になるのでしょうか?
完全に反射できないのだから、徐々に明るさが減衰するのか、それとも
立体を閉鎖した瞬間に暗くなるのか。

どなたか教えてください。
256ご冗談でしょう?名無しさん:2009/01/07(水) 00:07:16 ID:???
>>250
超伝導について一から勉強しなおせよ
257ご冗談でしょう?名無しさん:2009/01/07(水) 00:14:03 ID:???
>>255
古典的な電磁波なら指数的に減衰するわな。…と言っても、たとえば反射率
99.9%で鏡間の平均距離が 1m としたら0.1ms で 0.0000000000001 倍の明る
さになってしまう

実際の光は量子だからどっかの時点で最後の光子が吸収されたら終わりだな。
その前に黒体輻射と区別付かなくなるだろうけど。
258ご冗談でしょう?名無しさん:2009/01/07(水) 01:04:00 ID:LxplZ+Gj
2つの質量3[g]、電荷量q[C]の物体の長さ20[cm]の糸で吊り下げられている。
両者は反発しあい、糸が垂直と30℃の角度をなしてつりあっている。
電荷量q[C]の大きさはいくらか、という問題について教えてください。
Tsinθ=kq^2/r^2
Tcosθ=mg
r=lsinθ
r^2tanθ=kq^2/mg

q^2=l^2(sinθ)^2mg/(tanθk)
q=7.52×10^-6 C
という答えであってますでしょうか?
詳しい解説よろしくお願いします…。
259ご冗談でしょう?名無しさん:2009/01/07(水) 01:23:37 ID:???
>>253
ちょっとまって
光は真空中でも減衰して熱エネルギーになるんじゃないか
260ご冗談でしょう?名無しさん:2009/01/07(水) 01:35:04 ID:???
ごめん。
お前の発想は突飛すぎててついていけないや。
261ご冗談でしょう?名無しさん:2009/01/07(水) 01:38:40 ID:???
光は赤方偏移しながらエネルギーを失うから閉じ込めるのは無理だな
262ご冗談でしょう?名無しさん:2009/01/07(水) 01:41:01 ID:???
>>258
問題の設定と記号の定義がはっきり分からんが、
> r=lsinθ
これはあやしい感じがする。
263ご冗談でしょう?名無しさん:2009/01/07(水) 01:43:50 ID:???
>>261
その前にトンネル効果で外に逃げていっちゃうよ。
264225:2009/01/07(水) 02:04:40 ID:???
>>252
こういう風に定量的に評価できるのは知りませんでした。
でも最後は主観的な判定が不可避なのが面白いと思いました。
統計的に安定するかしないかのギリギリの系を研究すれば面白い発見が出来そう。
265ご冗談でしょう?名無しさん:2009/01/07(水) 06:24:57 ID:???
俺たぶん勘違いしてると思うんで教えて頂きたいんですが、
量子力学的には人間が壁をすりぬけたり、突然ワープしたりすることが
天文学的以上のめちゃくちゃ小さい確率で起きるわけですよね?

一方で、熱力学では外部から仕事を加えなければ
熱はひとりでに低温から高温に移動しませんが、
逆の言い方をすれば仕事を加えれば低温から高温に移動するということです

このように人間が壁をすりぬけるような現象、
つまり量子力学的なミクロでは起こりうる現象を
仕事を与えることでマクロなレベルで人為的に起こすことが出来るものなのでしょうか?
よろしくお願い致します。
266ご冗談でしょう?名無しさん:2009/01/07(水) 08:08:35 ID:m/iAtsgL
時 場合 条件
267ご冗談でしょう?名無しさん:2009/01/07(水) 12:39:03 ID:???
例えば超流動はマクロな量子現象なんだけど
そういうことが言いたいわけではないんだろうね。
268ご冗談でしょう?名無しさん:2009/01/07(水) 14:12:34 ID:???
バネを速く縮めて放した時と、ゆっくり縮めた時では、元に戻ろうとする力の差ってあるんでしょうか。
269ご冗談でしょう?名無しさん:2009/01/07(水) 14:14:45 ID:???
厳密にはあるだろうね。
270ご冗談でしょう?名無しさん:2009/01/07(水) 14:19:45 ID:???
現実の物質にはある。
ゴムを統計力学で考えても良いし、
弾性が何から来てるか考えても良い。
271ななし:2009/01/07(水) 15:24:42 ID:9fp0RqdT
がっちり円で交互に手をつないでいるから、すりぬけはむり。
272ご冗談でしょう?名無しさん:2009/01/07(水) 15:50:17 ID:???
コーラみたいな糖分たっぷりの炭酸飲料に紙を浸して
その紙を乾かしたら紙になにか変化はありますか?
273ご冗談でしょう?名無しさん:2009/01/07(水) 15:57:32 ID:7xMOEOOE
カピカピになる
274ご冗談でしょう?名無しさん:2009/01/07(水) 16:07:30 ID:???
>>267
いや、いい例じゃないの
275ご冗談でしょう?名無しさん:2009/01/07(水) 17:55:40 ID:???
井々の陣を張って一般の議論をする、ってどういう意味ですかね
テラカンに出てきたんすけど
276265:2009/01/07(水) 21:07:04 ID:???
>>267
あー確かに超流動はマクロな量子効果ですね
低温とか条件が揃えば、他にもマクロなレベルで
量子効果を人為的に起こす可能性もあると言うことでしょうか
277ご冗談でしょう?名無しさん:2009/01/07(水) 21:11:55 ID:???
>>225
アボガドロ数
278ご冗談でしょう?名無しさん:2009/01/07(水) 21:14:57 ID:???
>>276

ここで訊いて来い

  ○平成21年1月26日(月) 16:30-18:00 [理学部1号館 小柴ホール]
    Anthony J. Leggett 氏(イリノイ大学) 2003年ノーベル物理学賞
   ‘Cuprate Superconductivity:The Current State of Play’
279ご冗談でしょう?名無しさん:2009/01/07(水) 21:17:35 ID:???
>>278
ありがとうございます
でも専門家じゃないんで、英語はむりです
280268:2009/01/07(水) 22:13:58 ID:???
>>269-270

ありがとうございます
281ご冗談でしょう?名無しさん:2009/01/07(水) 23:09:56 ID:UYHxQGTR
COの分子振動数は2145cm^−1である。
2000Kの場合のν=0、ν=1における分布数の比を求めよ。
という問題で躓いてます。

ボルツマン分布を使うのだろうという予測はできていますが、
ボルツマン分布ni=a・gi・exp(-εi/kT)をどのように使うのか
サッパリわかりません。
当方高校物理程度の知識しかなく、この問題について10時間ほど
ググったのですが、全くわかりませんでした。
282ご冗談でしょう?名無しさん:2009/01/07(水) 23:26:35 ID:???
>>281
> ν=0、ν=1
νの定義は?
283ご冗談でしょう?名無しさん:2009/01/07(水) 23:31:50 ID:UYHxQGTR
>>282
それが書いてないんですよ…
多分エネルギー準位的やアレだと思うんですが…
284ご冗談でしょう?名無しさん:2009/01/07(水) 23:33:52 ID:6a0HlP6h
>>221 ありがとうございます。たいした説明もなしに言われたことだったのですっきりしました。
僕みたいな人にとってここは本当に良い場所ですね
285ご冗談でしょう?名無しさん:2009/01/07(水) 23:44:45 ID:???
>>281
振動数なのに単位がcm^-1なの?
286ご冗談でしょう?名無しさん:2009/01/07(水) 23:47:54 ID:???
>>283
指数関数の比を取ると、エネルギー準位の差が出てくるべ。
これ以上のヒントはやらん。自分で悩め。
287ご冗談でしょう?名無しさん:2009/01/07(水) 23:52:20 ID:UYHxQGTR
>>285
波数と言ってもいいかもしれません。

>>286
おふ…手厳しいお言葉…
288ご冗談でしょう?名無しさん:2009/01/07(水) 23:53:40 ID:1kBktD3f
375mb60℃の宇宙船に1010mb20℃の外気を注入すると
宇宙船の室温は何℃になるか計算してみよう。単純に考え
たほうが正解が出せます。
289ご冗談でしょう?名無しさん:2009/01/08(木) 00:00:51 ID:???
ふーん、そうなんだ。
290ご冗談でしょう?名無しさん:2009/01/08(木) 00:11:51 ID:???
>>288
マルチ氏ね
291ご冗談でしょう?名無しさん:2009/01/08(木) 00:41:37 ID:Xq0cdxV+
直径1mm、長さ1mの銅、アルミニウムの針金に100kgの加重をかけた時の伸びの量、ひずみの大きさはいくらか?
銅、アルミニウムのヤング率は探し出さなければならない。
という問題について教えてください。
調べたら銅のヤング率:13×10^10 アルミニウムのヤング率:7.0×10^10と出ましたが、この先の計算をどうしたらいいのかよくわかりません・・・。
どなたか教えてください。
292ご冗談でしょう?名無しさん:2009/01/08(木) 00:46:25 ID:???
ヤング率は圧力と同じ次元を持つはずだけど、なんで無次元なの?
293ご冗談でしょう?名無しさん:2009/01/08(木) 00:48:38 ID:???
>>292
あなたが勘違いしているからです。
294ご冗談でしょう?名無しさん:2009/01/08(木) 00:54:33 ID:???
ふーん。じゃあ勘違いしているらしい俺じゃ力になれんから一人で頑張ってくれ。
295ご冗談でしょう?名無しさん:2009/01/08(木) 00:57:13 ID:Xq0cdxV+
>>292
291です。
ヤング率の単位は調べたらPaとありました・・・。
296ご冗談でしょう?名無しさん:2009/01/08(木) 01:00:09 ID:???
ソイツハヨカッタネ \(^O^)/
297ご冗談でしょう?名無しさん:2009/01/08(木) 01:00:17 ID:???
その問題はなんかの講義とか授業の課題なんだろうけど、
ヤング率の意味とかそれを使った計算例とか授業でならわなかったの?
298ご冗談でしょう?名無しさん:2009/01/08(木) 01:11:52 ID:Xq0cdxV+
>>297
習いはしたんですがさっぱりわかりませんでした…。高校で物理とっておけばもう少し理解できてたんだと思いますがorz
あと書き忘れたのですが、ポアッソン比は銅:0.34 アルミニウム:0.35です。
299のぞきや:2009/01/08(木) 01:18:32 ID:e+97mp/I
何者かが天文気象に投稿した問題をいくつものスレッドに
書き込んでいるので288番は撤回します
解は34.85℃です


300ご冗談でしょう?名無しさん:2009/01/08(木) 03:45:51 ID:???
ポアソン比はその問題には関係ないと思う。

ヤング率をぐぐると Wikipedia が一番最初に出てくるが、それを読んでもわからないわけ?
ほぼ似たような例まで書かれてるんだが

> たとえば、ヤング率が約100GPaである銅では、断面積1mm^2、長さ1mのワイヤに10kg
> のオモリをぶら下げると、0.1%のひずみが生じる、すなわち約1mm伸びることなどを推
> 定することに使う値である。

 ・この問題での荷重は? (100kg とは 100kgf のことだろうけど、SI単位に直さないとな)
 ・針金の断面積は?
・針金にかかる引っ張り応力は?
 ・伸び方向のひずみは?
 ・伸びは?

と順番に考えたら?
301ご冗談でしょう?名無しさん:2009/01/08(木) 07:06:22 ID:Wxv6CGnj
302ご冗談でしょう?名無しさん:2009/01/08(木) 09:15:04 ID:h+MvKf4S
地球の周りを光速で移動したらどうなる?
303ご冗談でしょう?名無しさん:2009/01/08(木) 09:48:35 ID:XxJwGDFA
何が移動するかによる
304ご冗談でしょう?名無しさん:2009/01/08(木) 10:03:44 ID:???
スーパーマンの映画のワンシーンを思い浮かべてるなら>>302はチンカス以下の存在。
305ご冗談でしょう?名無しさん:2009/01/08(木) 10:57:50 ID:h+MvKf4S
スーパーマン見たことない。


宇宙船で地球の自転方向に光速で移動したら
未来に行けるんじゃないの?
306ご冗談でしょう?名無しさん:2009/01/08(木) 11:14:20 ID:1olCYici
[5階のテンソルは正の直交変換に関して等方であることを示せ]

っていう問題がわかりません

誰か教えてくださいm(TT)m
307ご冗談でしょう?名無しさん:2009/01/08(木) 11:58:12 ID:???
>>305
じゃぁ・・・
静止してる宇宙船のそばで地球のような丸い球(?)が自転方向(?)に光速で回転したら、
宇宙船は未来に行けるのかい?
308ご冗談でしょう?名無しさん:2009/01/08(木) 12:15:24 ID:???
>>306

丸投げですね(笑)
309ご冗談でしょう?名無しさん:2009/01/08(木) 12:44:18 ID:Wxv6CGnj
>>253
100%なら、反物質にあるんじゃない?
310ご冗談でしょう?名無しさん:2009/01/08(木) 12:52:10 ID:???
>>309
は?

>>251
トンネル効果でエネルギーが浸透していって減衰することは?
311ご冗談でしょう?名無しさん:2009/01/08(木) 14:03:38 ID:???
http://up2.viploader.net/pic/src/viploader898056.jpg
↑水に浮かべた船に扇風機を置いてスイッチオン
右に進むの?それとも動かない?

http://up2.viploader.net/pic3/src/vl2_092033.jpg
↑木の棒に対して絵のように紐を引っ掛ける
片方に足場を設置して、その上に乗っかり、紐を引っ張ると・・・

何だが動かない気がするけど、どう?
312ご冗談でしょう?名無しさん:2009/01/08(木) 15:13:14 ID:???
>>311
・初歩的には内力になり動かないと言う答えでもいいけど、
実際には前に動いたり後ろに動いたり横に動いたりする。
・普通に上がる。
313ご冗談でしょう?名無しさん:2009/01/08(木) 15:23:58 ID:???
>>311
小学校を卒業されてない方ですよね?
314ご冗談でしょう?名無しさん:2009/01/08(木) 15:37:46 ID:???
>>311
船の方は左方向に進むだろう。
315ご冗談でしょう?名無しさん:2009/01/08(木) 15:50:11 ID:???
>>313
そういう煽りはいいから。
>>314
どの方向に動くかは帆次第だな。
316ご冗談でしょう?名無しさん:2009/01/08(木) 16:22:32 ID:???
誰かテンソルの問題頼むよ
317ご冗談でしょう?名無しさん:2009/01/08(木) 19:25:48 ID:???
>>306
意味不明。おそらく問題文の写し間違い。
318ご冗談でしょう?名無しさん:2009/01/08(木) 20:15:31 ID:???
>>311
普通に右に進む
319ご冗談でしょう?名無しさん:2009/01/08(木) 20:26:49 ID:???
いや、>>315 のほうが正しい。帆が船と平行になってれば左に進むのはわかるだろ?
帆によってうまい具合に風が左側に流れ出すようにできれば右に進む。
320ご冗談でしょう?名無しさん:2009/01/08(木) 20:41:07 ID:???
>>311 の後半の問題
台が体重より軽いという条件は一応付けておくべきだろうな。
321ご冗談でしょう?名無しさん:2009/01/08(木) 22:03:56 ID:???
右に進む帆の形を見せてくれ
322ご冗談でしょう?名無しさん:2009/01/08(木) 22:29:08 ID:L/fX6s/6
「音」って質量を持たないの?
323ご冗談でしょう?名無しさん:2009/01/08(木) 22:32:03 ID:El2bR2dZ
PDFファイルで申し訳ありません。
相対性理論の教科書の問題の解答を載せているサイトがあったのですが、
ttp://yamakatsusan.hp.infoseek.co.jp/Schutz0706.pdf
7問目の(a)おのおののメトリックに対して、自由落下粒子の4元運動量の保存する成分を見つけよ。
という問題の解答がわかりません。
シュワルツシルトメトリックについてなのですが
PDFファイルの解答では、13ページ目にp_[0]、p_[i]が保存される。とあります。

自分でやってみると
m(dp_[α]/dτ)=(1/2)(∂g_[μν]/∂x^[α])p^[μ]p^[ν]
の式から、左辺のメトリック微分が0になるようなものが保存量になることが分かります。
シュワルツシルトメトリックは、rとθに依存することが12ページの表記からわかるので
保存量は、p_[0]とp_[φ]なのではないかと考えました。

面倒な質問ですが、よろしくおねがいします。
324ご冗談でしょう?名無しさん:2009/01/08(木) 22:49:25 ID:???
>>322
「音とは何か?」をちょっと調べればすぐに答えが出るよ。
325319:2009/01/08(木) 23:08:29 ID:???
>>321
俺は知らないよ。
326ご冗談でしょう?名無しさん:2009/01/08(木) 23:10:19 ID:???
物体を空中浮揚させる方法を教えて下さい。
327ご冗談でしょう?名無しさん:2009/01/08(木) 23:10:32 ID:L/fX6s/6
322 名前: ご冗談でしょう?名無しさん 投稿日: 2009/01/08(木) 22:29:08 ID:L/fX6s/6
「音」って質量を持たないの?

>>324
ちょっと調べればって、ちょっと調べる気も無いから「ちょっとした疑問や質問はここに書いてね104」で聞いたんだけど。
まー、ちょっと調べてみたけどわからん。

物体の振動と大気中の成分が関係してるって事は質量がある。
でOKなん?
もしそうなら、解説つきでちゃんとした答えを出してくれや。

答える気ないのか、わからないのなら変な試し方したり、ちゃちゃ入れるなや。
328ご冗談でしょう?名無しさん:2009/01/08(木) 23:16:56 ID:???
丸投げ野郎が何言ってんだww
329ご冗談でしょう?名無しさん:2009/01/08(木) 23:18:47 ID:???
330ご冗談でしょう?名無しさん:2009/01/08(木) 23:23:30 ID:???
>>327
普通、そこまで分かってるなら自分で分かるだろうが・・・
声や音っていうのは空気が振動する現象の結果だから、
音には質量なんか無いよ。
質量があるのは振動する空気。

まぁアレだ。
プールの波だと思えばいい。
波の大きさが変っても(音量が大きくなっても小さくなっても)
そのプールにある水の量は変らないということだ。
波に質量があるわけじゃなくて、水に質量があるんだよ。

分かるかい?小学生は早く寝な。
331ご冗談でしょう?名無しさん:2009/01/08(木) 23:24:00 ID:???
近頃は冬休みが終わりに近いからあせってるやつが多いのか?
332ご冗談でしょう?名無しさん:2009/01/08(木) 23:28:17 ID:???
うちの娘は昨日で冬休み終わってるけどね。
333ご冗談でしょう?名無しさん:2009/01/08(木) 23:31:30 ID:L/fX6s/6
偉そうに。
あんま調子に乗ってるとぶっ飛ばすぞ!
334ご冗談でしょう?名無しさん:2009/01/08(木) 23:31:41 ID:???
>>330
音のエネルギーの分だけ空気の質量が増えるよ。だから音には質量があるとも言える。
まあ空気の質量に比べたら無いに等しいから、無いとも言えるな。
335ご冗談でしょう?名無しさん:2009/01/08(木) 23:32:12 ID:???
>>333
偉そうなのはお前。死ねばいいと思うよ
336ご冗談でしょう?名無しさん:2009/01/08(木) 23:33:49 ID:L/fX6s/6
>>334
だろ?
量子論ではフォノンとか、あるしな。

光ではタキオンとか。

言い様によっては全く否定できない、とも言えなくともなくとも無い様な気がスルでもない気がした。
337ご冗談でしょう?名無しさん:2009/01/08(木) 23:35:23 ID:???
>>336
>量子論ではフォノンとか、あるしな。
空気を伝わる音って、量子化できるのか?
338ご冗談でしょう?名無しさん:2009/01/08(木) 23:38:00 ID:???
タキオンで思い出したけど、タキオンって虚数の質量を持つんだよね

ってことは、逆に言えば虚数の質量を持てば光速を超えられる、でおk?
339ご冗談でしょう?名無しさん:2009/01/08(木) 23:38:38 ID:L/fX6s/6
>>337
一つの論理でだ。
振動を量子化する、これが過程の話だと思ったらそう捉えてもらってかまわない。
ボソン(Boson)で論文読んでみ。
340ご冗談でしょう?名無しさん:2009/01/08(木) 23:39:51 ID:???
日本語にょろ。
341ご冗談でしょう?名無しさん:2009/01/08(木) 23:43:31 ID:L/fX6s/6
お前らの考え方について聞きたかっただけなのに。
>>338
エーテルの風。
342ご冗談でしょう?名無しさん:2009/01/08(木) 23:48:40 ID:???
最近の子供は自分が恥ずかしい立場になるとすぐキレるのな。
こういう奴は自分の考えが否定されたり知識の無さを認識させられたりすると
急に荒らし始めるんだよな。

>>334
そんなことで「音に質量がある」なんて言えないよ。
あと「無いに等しいから、無いとも言える」っていうのも間違い。
無いものは無い。
また、もし空気の質量が増えるとしても、それは空気の質量であって
音の質量ではないんだよね。
音っていうのは質量を持つかどうかという議論さえする必要も無いものだから。
まぁ、「痛み」に質量が無いのと同じようなもんだよ。
343ご冗談でしょう?名無しさん:2009/01/08(木) 23:57:53 ID:???
> また、もし空気の質量が増えるとしても、それは空気の質量であって
> 音の質量ではないんだよね。

それは音にはエネルギーが無いと言ってるのと同じだな。
「空気のエネルギーであって音のエネルギーじゃない」とどう違うのか?
それとも「音にはエネルギーは無い」でいいのか?
344ご冗談でしょう?名無しさん:2009/01/09(金) 00:00:51 ID:???
エネルギーは分子のものだろ。
音は形に過ぎないから、物質ではなく、エネルギーはない。
345ご冗談でしょう?名無しさん:2009/01/09(金) 00:06:17 ID:???
>>341
詳しく
346ご冗談でしょう?名無しさん:2009/01/09(金) 00:08:41 ID:Vd8fRtFx
>>323おねがいします
347ご冗談でしょう?名無しさん:2009/01/09(金) 00:18:17 ID:???
いやです
348ご冗談でしょう?名無しさん:2009/01/09(金) 00:49:36 ID:???
教えて下さい。
相対性理論から言えば、我々も光速で移動してるんじゃないんでしょうか?
349ご冗談でしょう?名無しさん:2009/01/09(金) 01:26:21 ID:???
遠くの宇宙から見れば光速を越えてるよ。
350ご冗談でしょう?名無しさん:2009/01/09(金) 01:32:16 ID:???
なんで分かるの?
351ご冗談でしょう?名無しさん:2009/01/09(金) 01:34:15 ID:???
相対性理論から言えば、我々は超光速で移動してるんじゃないんでしょうか?
352ご冗談でしょう?名無しさん:2009/01/09(金) 01:36:04 ID:???
イミフ
353ご冗談でしょう?名無しさん:2009/01/09(金) 03:00:12 ID:yh16qRF2
yはホログラム座標、θは角度、Aは定数で、
物体の波面e^(jφ)を記録したホログラムを角周波数ω、
波数kの光で再生したところ、ホログラムから
u=A*e^(-jφ)*e^(jky*sinθ)*e^(jωt) の光が射出した。
e^(-jφ)とe^(jky*sinθ)は、 それぞれ何を表しているのですか?
と言う質問をかなり(1ヵ月ぐらい)前にしたのですが、波動の勉強を
しても未だに理解できません。
馬鹿相手に説明しているぐらいのレベルで説明して頂けると嬉しいです。
354ご冗談でしょう?名無しさん:2009/01/09(金) 03:15:05 ID:ECE9PQOM
>>306 5階のテンソルで等方でないやつなんて掃いて捨てるほどある、
よってそもそもそれうそ。ま、>>316が正しいのだろう。
>>323一般相対論は教科書読んだ事がある、って程度だけれどもそれで
正しい、と俺は思う。一般にある連続対称性に対して保存量がでてくる
わけで、この場合は時間推進とz軸周りの回転なので。
>>337空気を伝わる音波は量子化できないなあ。>>339は通常の音波と
第2音波、縮退フェルミ系の第1音波あたりを混同している様子。
355ご冗談でしょう?名無しさん:2009/01/09(金) 03:25:37 ID:Vd8fRtFx
>>354
正しいってのは、私の解答と、サイトの解答とどちらのことですか?
356ご冗談でしょう?名無しさん:2009/01/09(金) 04:54:45 ID:ECE9PQOM
>>355 今指摘のサイト見たけど、これはミスプリじゃないか?
君ので合ってそうではないか、文を読む限り。
357ご冗談でしょう?名無しさん:2009/01/09(金) 07:13:35 ID:0CyprKXe
>>334
,量子の学問では、そうだな
358ご冗談でしょう?名無しさん:2009/01/09(金) 07:15:44 ID:0CyprKXe
波が発生するときに、エネルギーが加算されるから、そう考えても、無理はない。
359ご冗談でしょう?名無しさん:2009/01/09(金) 07:24:36 ID:???
うぷぷ

はぃい!?
360ご冗談でしょう?名無しさん:2009/01/09(金) 07:25:35 ID:???
エネルギーレベルの状態密度D(ε)をエネルギーレベルの粒子数の平均値にかけて全エネルギーで積分すると、どうして粒子数を表すことができるのですか?
しょーもない質問ですみません。。
361ご冗談でしょう?名無しさん:2009/01/09(金) 07:40:21 ID:???
丸投げですね♪
362360:2009/01/09(金) 08:11:01 ID:???
確かに丸投げ気味なのでもう少しわからないところを詳しく書きます。

私は状態密度をかけなくてもエネルギーレベルの粒子数の平均値を全エネルギーで積分すれば粒子数をそれだけで表せることができるのでは?

と疑問に思いました。なぜ状態密度D(ε)をかける必要があるのでしょうか。
363ご冗談でしょう?名無しさん:2009/01/09(金) 09:01:06 ID:???
「エネルギーレベルの粒子数」って何のこといってるの?
364ご冗談でしょう?名無しさん:2009/01/09(金) 09:02:57 ID:rQNjhQzs
数学で解くとだろ
365ご冗談でしょう?名無しさん:2009/01/09(金) 10:41:24 ID:???
理系全般板に顔出してみたんだがもしかしてあそこって単なる崩れの最終処分場なん?
煽りだらけでまともな議論やってねえんだが
366ご冗談でしょう?名無しさん:2009/01/09(金) 13:07:00 ID:???
誰か>>275
分かる人いませんか
367ご冗談でしょう?名無しさん:2009/01/09(金) 13:38:20 ID:???
井井=整整
368ご冗談でしょう?名無しさん:2009/01/09(金) 13:40:17 ID:???
正正の旗、堂堂の陣
369ご冗談でしょう?名無しさん:2009/01/09(金) 15:01:08 ID:???
>>367-368
ありがとうございました
孫子でしたか
さすが寺澤寛一大先生
370ご冗談でしょう?名無しさん:2009/01/09(金) 15:20:27 ID:???
光の速さで回転寿司が回ったらどうなるの?
http://takeshima.2ch.net/test/read.cgi/news4vip/1231480639/

お前らの知恵を貸してください
371をっさん ◆upJFuRhnJA :2009/01/09(金) 15:22:57 ID:???
ちゅどーーーーーーーーん!
372ご冗談でしょう?名無しさん:2009/01/09(金) 16:18:17 ID:???
>>371 新年明けまして死ね
373ご冗談でしょう?名無しさん:2009/01/09(金) 18:06:43 ID:WTrctpZM
「光」って質量を持たないの?
374ご冗談でしょう?名無しさん:2009/01/09(金) 18:07:04 ID:???
>>371
羽田野直道なつかしいw
375ご冗談でしょう?名無しさん:2009/01/09(金) 18:09:09 ID:Tl02D8Eo
>>356
遅くなってすみません。ありがとうございました。
376ご冗談でしょう?名無しさん:2009/01/09(金) 18:12:57 ID:???
>>373
>「光」って質量を持たないの?
慣性質量も重力質量もある。
しかし、静止質量はないかきわめて小さくがあるかは分かっていない。
377ご冗談でしょう?名無しさん:2009/01/09(金) 18:26:34 ID:WTrctpZM
>>376
と言う事は、地球やそれよりも強大・極小な重力がある天体(恒星)付近では、光の速度が変わると言う事でいいですか?
また、静止質量を仮定的な文言ではなく、「きわめて小さい」可能性もあるのでしょうか?

静止質量と重力質量の理論(論理?)に整合性がない(若しくは無いかもしれない)と言うのは、等価原理にそぐわない。
ゆえに、相対正論を覆す論理と捉えてよいですか?
378ご冗談でしょう?名無しさん:2009/01/09(金) 18:41:28 ID:???
>>377
> 光の速度が変わると言う事でいいですか?
よくない。

> 可能性もあるのでしょうか?
ない。
真空中での光の速度は一定、静止することはない故に。
379ご冗談でしょう?名無しさん:2009/01/09(金) 20:16:37 ID:giKknZoB
時間は無限に分割できるの?
380ご冗談でしょう?名無しさん:2009/01/09(金) 20:20:42 ID:???
>>378
>真空中での光の速度は一定、静止することはない故に。
机上の理論では、簡単の為にそうしてるに過ぎない。
光(子)が静止質量があるかどうかは分かっていないし、
人知を超える謎であり、永遠に分からないかもしれない。
381ご冗談でしょう?名無しさん:2009/01/09(金) 20:26:36 ID:???
はいはい。で?
382ご冗談でしょう?名無しさん:2009/01/09(金) 20:48:05 ID:???
そんなこと言ったら何でも言える
383ご冗談でしょう?名無しさん:2009/01/09(金) 20:49:57 ID:???
何でも言ってね。たとえば?
384ご冗談でしょう?名無しさん:2009/01/09(金) 20:52:01 ID:???
380に言ったのだが…
380の人?
385ご冗談でしょう?名無しさん:2009/01/09(金) 20:58:13 ID:???
> ゆえに、相対正論を覆す論理と捉えてよいですか?

ただの相間ですか
386ご冗談でしょう?名無しさん:2009/01/09(金) 21:04:06 ID:???
>>380
>机上の理論では、簡単の為にそうしてるに過ぎない。

せまい範囲に閉じ込められるのではなく、
本当に静止するならば、電磁波が伝搬しないということ、それは
マックスウェル方程式が成り立たない系があるということでは?
387ご冗談でしょう?名無しさん:2009/01/09(金) 21:51:08 ID:???
>>380がどういうつもりで言ったのかはわからんが(単なる意地悪
じいさんなだけなのかもしれんが)、光子に質量が実はありました、
ってことになる可能性が絶無というわけじゃない。実際しばらく
前までニュートリノに質量はないことになってたしな。ただ、通常
のエネルギー領域ではニュートリノはほぼ光速で走っているから
質量がないのとほとんど違いないというだけのこと。光子も同じで
我々が出会うことがあるエネルギー領域が光子の質量に比べて
圧倒的に大きいから気がつかないだけ、という可能性はもちろん、
ある。

 ただ、そのことは>>377を擁護するもんではまったくない。

388ご冗談でしょう?名無しさん:2009/01/09(金) 21:51:23 ID:KK7AvriD
光子の振動数または波長は光子の質量ではないのですか?
389ご冗談でしょう?名無しさん:2009/01/09(金) 21:52:32 ID:???
>>388
じぇんじぇん、違う。なんで振動数が波長で質量なんだ。お前の身長は体重か?

390ご冗談でしょう?名無しさん:2009/01/09(金) 21:57:00 ID:???
私の身長=体重です。
391ご冗談でしょう?名無しさん:2009/01/09(金) 22:06:28 ID:???
>>387
光に質量を持たせるとゲージ対称性が破れるから理論的にとても難しい。
ニュートリノはそういうことはない。
392ご冗談でしょう?名無しさん:2009/01/09(金) 22:09:57 ID:???
>>391
>ゲージ対称性が破れる
つ 小林-益川理論
393ご冗談でしょう?名無しさん:2009/01/09(金) 22:11:22 ID:???
>>392
それはCPだ
話が違う
394ご冗談でしょう?名無しさん:2009/01/09(金) 22:13:42 ID:???
やぶれると何がこまるの? 光子が質量をもつからだめなん?
395ご冗談でしょう?名無しさん:2009/01/09(金) 22:16:48 ID:KK7AvriD
>>389
だって質量とエネルギーは等価でしょ。
E=mc^2

光子のエネルギーは波長または振動数できまるでしょ。
E=hν=hc/λ

物質波の波長は物体の運動量できまるでしょ。
λ=h/p

物体の運動量は質量と速度の積でしょ。
p=mv

光子のエネルギーは光子の運動量と光速度でもきまるでしょ。
E=pc

上記の関係から光子の波長と質量、または、振動数と質量の等価性が導きだされるんじゃない。
m=h/(cλ)=hν/c^2

つまり波長が伸びるほど光子質量は小さくなって、波長が短くなると光子質量が増えるとか
光子は波長によってエネルギーがきまっているんだから現実とちっとも矛盾してないよね。
396ご冗談でしょう?名無しさん:2009/01/09(金) 22:18:30 ID:???
>>>389
>だって質量とエネルギーは等価でしょ。
>E=mc^2

>p=mv
はいダウト
397ご冗談でしょう?名無しさん:2009/01/09(金) 22:21:11 ID:???
>>395
たしかに。
398ご冗談でしょう?名無しさん:2009/01/09(金) 22:25:35 ID:???
>>395
E=mc^2
って何のエネルギーかわかる?
おばかちゃん♪
399ご冗談でしょう?名無しさん:2009/01/09(金) 22:27:47 ID:KK7AvriD
>>396
p=mv+h/λ=mv+hv/c

て言いたいんでしょ。
運動量が足し算できているんだんから質量と波長は等価と言っているような関係だと思うんだけど。
400ご冗談でしょう?名無しさん:2009/01/09(金) 22:28:33 ID:???
>て言いたいんでしょ。
違います。
401ご冗談でしょう?名無しさん:2009/01/09(金) 22:28:38 ID:???
>>395
この範囲では、m=E/c^2 で定義された m が E の「別名」にしかなっていない
ので、矛盾はないが冗長。現在は「質量」という用語はここでの m とは異なる
量の名前として使われている。
402ご冗談でしょう?名無しさん:2009/01/09(金) 22:30:22 ID:???
>>399
そんなのどうでも良いから、
正しい運動量の表示を書いてみてよ。
403ご冗談でしょう?名無しさん:2009/01/09(金) 22:30:48 ID:???
>>399
いい加減諦めろよ、頭冷やしてさっさと寝ろ
404ご冗談でしょう?名無しさん:2009/01/09(金) 22:56:29 ID:???
循環論法
405ご冗談でしょう?名無しさん:2009/01/09(金) 22:59:03 ID:KK7AvriD
運動量はこうかってことですね。
p=mv/(1-v^2/c^2)^1/2

光子に質量があったら光子の運動量は、無限大になるって言うのなら
速度も量子と考えれば光速度の漸近線速度があって、この漸近線速度より1小さい速度が
実際の光速度で上の式のcは漸近線速度と考えると、たとえ光子でも漸近線速度を超えられないとなれば
光子に質量があってそのときの運動量がh/λに等しいって言えるんじゃないかなあ。
406ご冗談でしょう?名無しさん:2009/01/09(金) 23:10:33 ID:???
>>394
今、知られている力はすべてゲージ理論で記述されてるからね。
標準模型も。で、実験的にも合ってると。
まあ、可能性はゼロじゃないけどね。だから「とても難しい」と書いた。
407ご冗談でしょう?名無しさん:2009/01/09(金) 23:12:58 ID:???
>>405
>だって質量とエネルギーは等価でしょ。
>E=mc^2

ここからおかしい。
408ご冗談でしょう?名無しさん:2009/01/09(金) 23:13:30 ID:???
>>406
>ゲージ理論
で、ゲージ理論って何? ていうか、ゲージ対称性って何?
良く聞くけど、全然分からないヨーーーー
409ご冗談でしょう?名無しさん:2009/01/09(金) 23:16:13 ID:qci9Se9W
白熱灯が、熱を持つのは、どうして?
410ご冗談でしょう?名無しさん:2009/01/09(金) 23:19:26 ID:???
電気には愛が詰まっているからだよ。
411ご冗談でしょう?名無しさん:2009/01/09(金) 23:24:27 ID:tQTaHEXu

問1、ay"+2by'+cy=f(x)の解を求めよ。
問2、x"+2bx'+cx=f(t)の解を求めよ。

412ご冗談でしょう?名無しさん:2009/01/09(金) 23:26:18 ID:???
413ご冗談でしょう?名無しさん:2009/01/09(金) 23:28:28 ID:???
>>408
ゲージとはものさしの意。
ものさしを変えても物理法則自体は変わりませんよ、というのがゲージ
不変の意味。

理論がゲージ不変であるということは、おまいが好きなゲージ(例えば
座標系)を選んでも物理が正しく記述できることを保証している。
414ご冗談でしょう?名無しさん:2009/01/09(金) 23:30:56 ID:???
>>391
> 光に質量を持たせる
超伝導体中では実際そうなっている。真空でそれが起こることもあり得るというのが
南部、Brout-Englert、Higgs、Guralnik-Hagen-Kibble の理論だが。
415ご冗談でしょう?名無しさん:2009/01/09(金) 23:33:44 ID:???
独学で量子力学勉強しようと思うんですがお勧めの本ありますか?
(どのように勉強していけばいいですか?)

ちなみに高校数学(VCまでやった)は得意だったが今は文系です。
416ご冗談でしょう?名無しさん:2009/01/09(金) 23:35:58 ID:???
>>415
お勧めの本があるわけじゃないんだが
やっぱり先に古典力学と電磁気学はやっておいたほうがいいと思う
417ご冗談でしょう?名無しさん:2009/01/09(金) 23:41:45 ID:???
>>416
あ、古典力学と電磁気学は高校の時突っ込んで勉強したんで大丈夫だと思います。
418ご冗談でしょう?名無しさん:2009/01/09(金) 23:41:59 ID:???
一応、清水は文系でも読めるのが売り
ただ、>>416同様古典力学・電磁気学はやっといた方がいいと思うけど
個人的には熱力学も
419ご冗談でしょう?名無しさん:2009/01/09(金) 23:42:58 ID:???
文系ならブルーバックスの適当な量子力学本を読むのが一番いいと思う。
http://shop.kodansha.jp/bc2_bc/search_list.jsp?n=20&type=t&a=&t=&word=%97%CA%8Eq%97%CD%8Aw&search=%8C%9F%8D%F5

本当に真剣に量子力学を勉強したいのなら、シッフやらメシアやらディラックやらの本物の理学書に挑戦して挫折するのも良い経験。
420ご冗談でしょう?名無しさん:2009/01/09(金) 23:43:29 ID:???
E^2 = (mc^2
421ご冗談でしょう?名無しさん:2009/01/09(金) 23:43:45 ID:???
>>417
突っ込んでっていうと、古典力学だと解析力学とか、電磁気だと電磁波の導出とかまで?

あと>>416に書き忘れたけど、量子やるなら前もって振動・波動もやっといてほしい
422420:2009/01/09(金) 23:44:05 ID:???
あ、途中で書き込んじまった orz
なんかやる気なくしたので続きは後で…
423420:2009/01/09(金) 23:49:42 ID:???
気を取り直して…

E = mc^2 は静止してるとき (運動量 p が 0 のとき) の式で、一般には

 E^2 = (mc^2)^2 + (pc)^2

となる。

 光子のエネルギーは E = hν
 光子の運動量は    p = h/λ = h/(c/ν) = hν/c = E/c

だから

  E^2 = (mc^2)^2 + (E/c・c)^2 = (mc^2)^2 + E^2

つまり m = 0。何か問題が?
424ご冗談でしょう?名無しさん:2009/01/09(金) 23:51:23 ID:???
>>415
>独学で量子力学勉強しようと思うんですがお勧めの本ありますか?

別に細かい計算はいらんのだろ?

ディラクもいいけど、J. J. サクライとかも面白かったよ。
ttp://www3.ocn.ne.jp/~yoshioka/phys/ISBN4-8427-0222-2.html
425ご冗談でしょう?名無しさん:2009/01/09(金) 23:54:23 ID:???
文系にサクライを薦める鬼畜がいると聞いてやってきました
426ご冗談でしょう?名無しさん:2009/01/09(金) 23:57:48 ID:KK7AvriD
>>407
エネルギーと運動量の関係はこうだから
-E^2=-m^2*c^4+p^2

そもそもエネルギーと質量の等価性は物体が静止しているときにしかなりたたないって言うんですよね

でも光子の静止質量運動量に見えている考えても
E=pc
-E^2=-m^2*c^4+E^2
-2*E^2=-m^2*c^4
2*E^2=m^2*c^4
E=mc^2/√2

と定数項がc^2以外に1/√2がつくだけで等価性はくずれてませんよね。
427ご冗談でしょう?名無しさん:2009/01/10(土) 00:09:38 ID:???
>>426
あるものについては E=ma、別のものについては E=mb、a≠b のとき E と m は等価っていうのかね?
428ご冗談でしょう?名無しさん:2009/01/10(土) 00:12:19 ID:???
初学者なら和田先生の量子力学のききどころあたりがよいかと。
理系もみんなそういう読みやすい本から入ってる。
429ご冗談でしょう?名無しさん:2009/01/10(土) 00:16:44 ID:???
>>415
もし大学の物理専攻でやるように数式レベルで追いたいなら、数学は線形代数、多変数の微積分(偏微分・重積分)、
ベクトル解析、テイラー展開、フーリエ変換、微分方程式くらいをまず抑えた方がいい。
高校レベルの数学だけではちょっと無理です。
430ご冗談でしょう?名無しさん:2009/01/10(土) 00:22:37 ID:???
力に速度を掛けたものが運動エネルギーであると習ったのですが、
Fv[ kg m/s^2 ・m/s = kg m^2 / s^3]となって、
kg m^2 / s^2と合わないのですが・・・
431ご冗談でしょう?名無しさん:2009/01/10(土) 00:27:38 ID:???
>>414
自発的対称性の破れも書こうとしたのですけどね。
あまり自信がないのでやめましたw

真空中で起こるのは光ではないのでは?
432ご冗談でしょう?名無しさん:2009/01/10(土) 00:30:46 ID:???
>>430
あなたが聞き間違えたのか先生がいい間違えたのか知らないけど、力に速度をかけたものは運動エネルギーの時間変化率であって運動エネルギーそのものではない。
433ご冗談でしょう?名無しさん:2009/01/10(土) 00:30:55 ID:???
>>430
そりゃ、おまえが先生が運動エネルギーについて話している時に、居眠り
してたってだけのことだろう。力に測度かけても運動エネルギーにはならん。
434ご冗談でしょう?名無しさん:2009/01/10(土) 00:33:29 ID:???
ゲージ不変性が実は少しだけ破れてました、とあと10年の間に
発見されないと断言できる人間はいないはずだ。

物理の世界で言えることには常に「現在の実験精度のもとでは」
という枕詞がつく。
435ご冗談でしょう?名無しさん:2009/01/10(土) 00:38:36 ID:???
>>432
たぶん時間変化率のことです(´・ω・`)
ありがとうございました

>>433
測度とは数学者のかたですね!
436433:2009/01/10(土) 00:42:48 ID:???
>>435
うんにゃ、さっきまで経路積分してた物理屋だ。
437ご冗談でしょう?名無しさん:2009/01/10(土) 00:45:50 ID:???
脳筋ですら旬殺の問題とか・・・
438ご冗談でしょう?名無しさん:2009/01/10(土) 00:47:56 ID:???
>>430
つ [仕事率]
439ご冗談でしょう?名無しさん:2009/01/10(土) 01:18:26 ID:???
>>417
いまの高校の数学だとIIICまでやっても偏微分でてこねぇだろ
IIIC以上の数学独学してるならともかく、高校数学だけだとしてどうやって電磁気学勉強したんだ?
440ご冗談でしょう?名無しさん:2009/01/10(土) 01:28:37 ID:???
昔の高校数学には出てきたのか?と素朴な疑問が・・・・・
441ご冗談でしょう?名無しさん:2009/01/10(土) 01:39:20 ID:???
単純に昔のことは知らんというだけだが?
442ご冗談でしょう?名無しさん:2009/01/10(土) 01:43:49 ID:D91KXTe5
シュレーディンガーの猫に関して質問です。
当方日本文学専攻の大学生で、物理学に関しては全くの門外漢なのですが、読み物として書かれた物理の本には以前から魅力を感じており、レポートで「シュレーディンガーの猫」を題材にしたいと考えています。
wikipediaで「シュレーディンガーの猫」を参照すると
「現在ではシュレーディンガーの猫のように巨視的に量子力学の効果が現れる実験系が知られている」
という表記があります。これはシュレーディンガーの猫がパラドックスとして機能しない場合があると言うことでしょうか?また具体的にその「実験系」とはどのような物でしょうか?
大学の図書館でいくつか文献を当たりましたがお手上げ状態です。よろしくお願いします。
443ご冗談でしょう?名無しさん:2009/01/10(土) 02:00:00 ID:???
>>442
シュレディンガーの猫は思考実験だよ
444ご冗談でしょう?名無しさん:2009/01/10(土) 02:09:52 ID:???
最初からパラドクスじゃないんじゃない?
445ご冗談でしょう?名無しさん:2009/01/10(土) 02:16:38 ID:???
別に論理矛盾はない。
常識的に理解不能というだけ。
446ご冗談でしょう?名無しさん:2009/01/10(土) 02:49:35 ID:1xwvyhk4
ナンチャラ対称性がどうのこうので、粒子は残って反粒子が消えて宇宙は粒子だけになったってことですけど。
なら、現在も宇宙では粒子と反粒子は生まれ続けているんだから、
その過程で粒子は残って反粒子が消えると言う現象起こりつづけていて、
始まり以来たえず宇宙にある粒子は増え続けているというこてでしょうか?
447ご冗談でしょう?名無しさん:2009/01/10(土) 03:25:01 ID:???
>>442
>「現在ではシュレーディンガーの猫のように巨視的に量子力学の効果が現れる実験系が知られている」
書いてある通りの意味だよ。
さすがに生き物の生死について量子力学の効果が現れた実験系はないが、
数年前の実験で超電導物質の輪っかを作ったら、右回りに電流が流れていると同時に
左回りに電流が流れているという状況が作り出された。
448ご冗談でしょう?名無しさん:2009/01/10(土) 04:40:40 ID:bJfDjP5M
座標演算子xがエルミートであること(φ,xψ)=(xφ,φ)の証明ってどうすればいいのでしょうか?
運動量演算子-ihd/dxなんかなら微分があるので部分積分が使えるんですが、微分がない場合はどうすればいいのかわかりません。
(φ,xψ)の複素共役をとったり色々やってみましたがわかりません。
ご教授ください。
449ご冗談でしょう?名無しさん:2009/01/10(土) 05:12:24 ID:???
>>448
両辺の各々を定義に従って計算して比べる、ではだめ?
450ご冗談でしょう?名無しさん:2009/01/10(土) 05:19:43 ID:???
>>446
宇宙初期のある時期に生成されたと考えるシナリオが主流。
観測や実験の結果から、現在の宇宙に存在する量のバリオンが現在の宇宙の状態で
十分生成されたとは考えにくい。
451ご冗談でしょう?名無しさん:2009/01/10(土) 10:21:52 ID:???
>>449
内積を積分でということでしょうか?
それすらもわからない状態です…
左辺を積分形に直して共役とったり色々いじっても、結局xはψにくっついたままで右辺の積分にはならなくて…
452ご冗談でしょう?名無しさん:2009/01/10(土) 10:23:54 ID:pHaKuvBJ
>>451 固有値として捉えないと無理。
453ご冗談でしょう?名無しさん:2009/01/10(土) 10:46:11 ID:pHOUb8jM
>>451、君は刀iなぜか周回積分記号しか出てこないので放置)φ^*(x)xψ(x)dxにおいて
xの位置が変わると値が変わる、と認識してるのか??
454ご冗談でしょう?名無しさん:2009/01/10(土) 11:04:26 ID:bJfDjP5M
演算子xの固有値xみたいな形でいくということでしょうか?
455ご冗談でしょう?名無しさん:2009/01/10(土) 11:07:03 ID:bJfDjP5M
>>454>>452さんへのレスです

>>453
演算子ならいきなり波動関数の前や後ろにくっつけたらどうかなと思ってしまいます^^;
なにも作用しない演算子なら前後に移動してもいいんでしょうか?
456ご冗談でしょう?名無しさん:2009/01/10(土) 11:21:36 ID:rY8FzRjb
457ご冗談でしょう?名無しさん:2009/01/10(土) 12:05:39 ID:???
>>455
[x]という演算子は波動関数にxをかけるという操作を表す演算子
何も作用しないわけではない。
458ご冗談でしょう?名無しさん:2009/01/10(土) 12:39:37 ID:0o0gJHSG
 ビッグバンの時に粒子と反粒子が生成されたと言われていますが
そもそも、現在は反粒子が自然には存在しないのだから、粒子のみ
が生成されたのではないのでしょうか?
459ご冗談でしょう?名無しさん:2009/01/10(土) 13:59:13 ID:???
宇宙にあるブラックホールってどこかの文明が滅びたときにできたんじゃないでしょうか?ブラックホールの実験とかで。
460ご冗談でしょう?名無しさん:2009/01/10(土) 14:11:54 ID:rY8FzRjb
超新星爆発とかだと、ブラックホールできるよ。
461ご冗談でしょう?名無しさん:2009/01/10(土) 14:18:47 ID:YNSXJk+O
>>458
現代に恐竜がいないのだから過去にもいないという論理にはならない。
462ご冗談でしょう?名無しさん:2009/01/10(土) 14:28:56 ID:rY8FzRjb
客観でも、変化してしまうものってあると思う?
463ご冗談でしょう?名無しさん:2009/01/10(土) 14:29:25 ID:0o0gJHSG
>>461
 恐竜が居たという痕跡は残っている。しかし反粒子は実験
で作られていない。つまり自然には存在しない。

464ご冗談でしょう?名無しさん:2009/01/10(土) 14:33:10 ID:???
反粒子はコライダー実験のたびに作られてますが、何か。
465ご冗談でしょう?名無しさん:2009/01/10(土) 14:33:15 ID:???
>>463
> 反粒子は実験で作られていない。

あなたの脳内宇宙では知りませんが、私たちの宇宙では作られてますし自然でも観測もされてますが何か?
466ご冗談でしょう?名無しさん:2009/01/10(土) 14:41:17 ID:0o0gJHSG
>>464
スマン実験で・・・は実験でしか、の間違いです。
467ご冗談でしょう?名無しさん:2009/01/10(土) 14:41:52 ID:0o0gJHSG
>>465
 >自然でも観測もされてますが何か?
 自然観測で、対消滅反応がみられたの?
468ご冗談でしょう?名無しさん:2009/01/10(土) 14:47:38 ID:rY8FzRjb
主観で変化するものはありますね。
469ご冗談でしょう?名無しさん:2009/01/10(土) 14:47:54 ID:???
>>467
つ コンプトンガンマ線観測衛星
470ご冗談でしょう?名無しさん:2009/01/10(土) 14:48:07 ID:???
まだだよ。
471ご冗談でしょう?名無しさん:2009/01/10(土) 14:49:02 ID:???
>>468
>主観で変化するものはありますね。
波動関数の収束とか。
472ご冗談でしょう?名無しさん:2009/01/10(土) 14:51:03 ID:???
473ご冗談でしょう?名無しさん:2009/01/10(土) 15:12:59 ID:???
>>466
恐竜は実験ですら再現できない。ジュラシックパークならできたが。
実験でいつでも生成できる反粒子より恐竜の方が実在性は危うい。
化石のたぐいは宇宙人が仕込んだイタズラとも言えるしね。
474ご冗談でしょう?名無しさん:2009/01/10(土) 15:14:30 ID:???
反物質だって宇宙人が計器を操作して仕込んだ結果だヨ
475ご冗談でしょう?名無しさん:2009/01/10(土) 15:20:47 ID:???
ならお前が生活してる世界は全て宇宙人がこしらえた擬似現実なんだろう。

お前がゆくところ宇宙人が先回りして他人が生活してるように演技してるんだよ。
476ご冗談でしょう?名無しさん:2009/01/10(土) 15:29:50 ID:???
+がクワスかプラスかは判別不可能
477ご冗談でしょう?名無しさん:2009/01/10(土) 15:31:42 ID:rY8FzRjb
特異点
478ご冗談でしょう?名無しさん:2009/01/10(土) 15:34:43 ID:rY8FzRjb
これができたら、学士卒業おめでとう。

卒業のときは聞いてね。
http://www.nicovideo.jp/watch/sm4317695
479ご冗談でしょう?名無しさん:2009/01/10(土) 15:41:32 ID:???
>>467
そもそも反粒子の最初の発見って人工的に生成したものではなくて自然由来のものだよ
480ご冗談でしょう?名無しさん:2009/01/10(土) 17:55:14 ID:???
ローレンツ条件を用いた場合の電磁ポテンシャル(φ、vector(A))が満たすべき微分方程式を求めよ。
ローレンツ条件でしらべてみたのですが微分方程式の求め方がわかりません。おねがいします。
481ご冗談でしょう?名無しさん:2009/01/10(土) 17:56:24 ID:???
>>479、ベータプラス崩壊は放射能除去装置を売り込みたい宇宙人が自分の
商品デモの為に地球近辺で行っている、って知らないのかっ!!
滑ったらごめん。
482448:2009/01/10(土) 17:56:45 ID:XWtc9Ytl
(xφ,ψ)=x*・(φ,ψ)=x・(φ,ψ)=(φ,xψ)

この変形ならエルミートだと言えますか?
共役の算数に自信がないのですが^^;
483ご冗談でしょう?名無しさん:2009/01/10(土) 18:05:58 ID:???
>>481
最後の一行がなければなぁwwww
484ご冗談でしょう?名無しさん:2009/01/10(土) 18:22:08 ID:???
当方、文型人間です。

光の速さは一定ですよね。
音波の伝わる速さも一定だよね?

ところで水面の波は、速さは一定なの?
485ご冗談でしょう?名無しさん:2009/01/10(土) 18:26:38 ID:???
>>484
> 音波の伝わる速さも一定だよね?

風が吹けば音速も変わるッス
486ご冗談でしょう?名無しさん:2009/01/10(土) 18:27:31 ID:???
光の速さは慣性系において一定。
音波の速さは媒質によっていろいろ変わる。
水面の波の速さも媒質の状態によって変わる。
487ご冗談でしょう?名無しさん:2009/01/10(土) 18:34:05 ID:gzJXQpLS
「素粒子」って質量を持たないの?
488ご冗談でしょう?名無しさん:2009/01/10(土) 18:36:20 ID:???
>>486
>水面の波の速さも媒質の状態によって変わる。
深さ、波長、重力の強さかな。水の密度は関係ないらしい。
489ご冗談でしょう?名無しさん:2009/01/10(土) 18:37:10 ID:???
連続質問でスマン

「マッハ」の単位って音の速さって定義だと思ったけど、
音の速さが一定じゃないとしたら、じゃあ「マッハ」の定義ってどうなってるの?
490ご冗談でしょう?名無しさん:2009/01/10(土) 18:41:06 ID:???
ゴメン 調べたらわかった。
 
491420:2009/01/10(土) 19:04:55 ID:???
水の波は波長によっても振幅によっても変わるね。
492ご冗談でしょう?名無しさん:2009/01/10(土) 19:28:51 ID:???
地球はずっと地球上や周囲の物体に重力という力を
及ぼし続けていますが、なにかエネルギー(?)のようなものを消費しないのでしょうか?
そのうちエネルギー(?)を使い果たして重力が弱くなるようなことは起きないのですか?
493ご冗談でしょう?名無しさん:2009/01/10(土) 19:30:38 ID:???
>>492
もち、エネルギー消費してるよ。
494ご冗談でしょう?名無しさん:2009/01/10(土) 19:31:32 ID:???
>>493
ということは、徐々に地球は小さくなってるということになりますか?
495ご冗談でしょう?名無しさん:2009/01/10(土) 19:35:19 ID:???
地球の重力波として放出されるEなんてごく微量だから、
他のエネルギー収支を考慮するとかき消されるから何とも。

中性子星の連星系だと観測できる程度だけど。
496ご冗談でしょう?名無しさん:2009/01/10(土) 19:36:48 ID:???
あーなるほど
重力子という粒子が存在すると仮定して、
それが重力波として放出されていると考えればよいのですか・・・
重力子の質量とかって予言されていたりするのですか?
497ご冗談でしょう?名無しさん:2009/01/10(土) 20:06:18 ID:???
地球は重力子を放出すると、その質量=エネルギー分軽くなる。
しかし、他の天体からの重力子を吸収してい打ち消し合い、
結局、質量は変化しない。
もし、宇宙に地球しかないと、地球は重力子を放出し尽くし、
生命力を失い、重力的活性を失い死んでしまう。
498ご冗談でしょう?名無しさん:2009/01/10(土) 20:18:20 ID:???
>>482だめ。演算子xは波動関数に作用する。ところが君のx(φ,ψ)という書き方
だと単なる複素数(φ,ψ)(有限次元でもそうだろ、内積とった後はスカラー
になるよ)にxを作用させる、ということになってしまう。厳密に書きたいなら
x^を演算子としてのxとして(φ,x^ψ)=塔モ*(x) x ψ(x)dx、(x^*φ,ψ)=
(共役の定義からくる式)、(x^φ,ψ)=怒xφ(x)}*ψ(x)dx、あとは自分で
考えて。
499ご冗談でしょう?名無しさん:2009/01/10(土) 20:55:41 ID:h7ucMzpL
>>492
なんで力が働くだけでエネルギーを消費すると思うの?
まず高校物理の復習を
500ご冗談でしょう?名無しさん:2009/01/10(土) 21:16:23 ID:???
力を出して仕事すると、エネルギー失って疲れるじゃん。
501ご冗談でしょう?名無しさん:2009/01/10(土) 21:27:53 ID:???
495は制動放射のことを書いているが、
質問者はその精度の話を理解するレベルに達していないので
それで納得されても困る>>496
493は死ね
502ご冗談でしょう?名無しさん:2009/01/10(土) 21:33:11 ID:???
>>500
筋肉は力を出すと(力学的な意味での)仕事をするしないに関係なくエネルギーを
消費するような仕組みになってる。
503ご冗談でしょう?名無しさん:2009/01/10(土) 21:59:56 ID:meT0Mvu7
ほんとは化学なんですが・・
向こうの板はこっちがまじめな質問をしても、
程度が低いと(実際低くもないと思うんですが)ふざけてるとみなされ
スルーされてしまうので、こちらで聞かせて頂きます。
(こちらはよく解答が貰える。)
4つビーカーの中に液体を入れて、ABCDとして並べ、
互いに導線をつないで、DはAと導線をつなぎます。
導線の先端には、極板がついています。
この場合、溶液によって、ADが電池となり、BCが電気分解の実験となる。
また、BCが電池となりADが電気分解の実験となるんですよね?
つまり、電池と電気分解の区別の仕方を教えて頂きたいのですが…。
どちらも結局同じような反応で還元、酸化が起きて電子の移動が起こるので、
区別がつけられないような…
504ご冗談でしょう?名無しさん:2009/01/10(土) 22:04:38 ID:???
>>503
イオン化傾向やらで起電力を生じたりしなかったりで分からないかね?
505ご冗談でしょう?名無しさん:2009/01/10(土) 22:20:29 ID:???
>>503

具体的にどんな反応になるか知らんけど、
その電気分解が、電池として働く時の逆反応ならそれを「充電」と呼ぶんだろ。

電池と呼ぶときには、自発的にに反応がすすんで外に仕事するが(電池内のエントロピーが増大)、
充電のときには、外部から無理矢理電流を流すことで仕事を蓄えている(電池内のエントロピーを下げている、外部とのトータルでは増大)
ということでないか?
506ご冗談でしょう?名無しさん:2009/01/10(土) 22:29:54 ID:???
>>499
力が働き続けたら何かを消費していると考えるのは自然なこと
実際その通りだし
507ご冗談でしょう?名無しさん:2009/01/10(土) 22:37:56 ID:???
>>506
もういいから帰れよ
508420:2009/01/10(土) 22:50:39 ID:???
>>506
その「自然なこと」と思えることが必ずしも正しくないってこった。
アリストテレスの時代からそれが間違いだと気付くのに数千年を要したのだから無理もない。
509ご冗談でしょう?名無しさん:2009/01/10(土) 23:04:35 ID:???
>>506
>力が働き続けたら何かを消費していると考えるのは自然なこと
>実際その通りだし

反例1:
物と物とのスキマにバネを押し込める。
摩擦のために両物体が動かないとき、バネの力は物に働き続ける。
だが、何かを消費し続けているわけではない。

反例2:
テーブルの上に物が置いてある。
物体には、重力とテーブルからの垂直抗力が働き続ける。
だが、何かを消費し続けているわけではない。

筋肉の場合は、投入したエネルギーを仕事と熱と化学反応にして放出している。
外部に仕事してなくても、エネルギーを消費してしまう個別の理由がある。
510ご冗談でしょう?名無しさん:2009/01/10(土) 23:07:12 ID:???
磁石で永久機関発明しようとしたりしそうだなw
511ご冗談でしょう?名無しさん:2009/01/10(土) 23:10:20 ID:???
>>501
495=493だけど、まあいいか

地球表面での些事は兎も角、マントル対流とかしてるんで、その分重力波の放出とか有るんじゃねえの?
地球-月系だって、E放出してるっしょ。
512ご冗談でしょう?名無しさん:2009/01/10(土) 23:15:30 ID:SddVoyjR
大学の物理学の授業でレポート
を書かなくてはいけないんですけど

・ディープインパクト計画
・ハッブル宇宙望遠鏡      について書こうと思います

参考になるページがあったら教えてください。
よろしくお願いします。
                

513ご冗談でしょう?名無しさん:2009/01/10(土) 23:17:41 ID:???
>>512
あー、一昨日来てくれてたらその質問に答えられたんだけどね。残念!
514ご冗談でしょう?名無しさん:2009/01/10(土) 23:48:45 ID:???
ラグランジアン密度L[x]って、よく一般化座標φ(x)とその不均一性∂φの関数としてるけど
単に、高次の∂∂φとかは無視できる近似っていうこと?
515ご冗談でしょう?名無しさん:2009/01/11(日) 00:15:23 ID:???
>>511
じゃ、お前自体馬鹿だ
516ご冗談でしょう?名無しさん:2009/01/11(日) 00:22:30 ID:???
>>509
全然理解してなくてクソワロタw

1も2も疲労破壊という現象が起きるだろ
517ご冗談でしょう?名無しさん:2009/01/11(日) 00:30:02 ID:???
だとして、なに?
518ご冗談でしょう?名無しさん:2009/01/11(日) 00:35:26 ID:???
>>517
お前自体馬鹿だってこと
519ご冗談でしょう?名無しさん:2009/01/11(日) 00:42:38 ID:U2JGqo4s
すみません、教えてください。

常温で水素原子ってどれくらい存在していられるのでしょうか?
数十億年くらい持つのでしょうか?
もたないとしたら、どういったエネルギーが放出されているのでしょうか?
520ご冗談でしょう?名無しさん:2009/01/11(日) 00:45:18 ID:???
>>517
お前はやっぱりきちんと理解してないな
抗力が起きる要因とか考えたこともないんだろう
521ご冗談でしょう?名無しさん:2009/01/11(日) 00:47:34 ID:???
>>516
>1も2も疲労破壊という現象が起きるだろ

繰り返し運動もなく、ある程度低温なら、疲労破壊なんて起きねえよ。

第一、もし疲労破壊が起きているなら、
物質の状態の変化にエネルギーを使われただけで、
実質的に仕事をしているわけだ。

>509は、たとえば、仕事(エネルギー)= 力×距離
だから、力が有限でも距離がゼロの定常状態ならエネルギーは消費されない
という指摘だろ。それはどう見ても熱力学や力学の基本。
熱力学とか自由エネルギーの概念を全く無視して
疲労破壊とか2次的な話にすり替えるのは、
物理をある程度理解していると主張する人間としては恥ずかしいと思う。
522ご冗談でしょう?名無しさん:2009/01/11(日) 00:56:12 ID:???
重力理論の話で「高校物理」とか言ってる時点で
お前ら釣りだって気づいてやれよ・・・
523ご冗談でしょう?名無しさん:2009/01/11(日) 01:28:13 ID:???
521の勝ちだな
524ご冗談でしょう?名無しさん:2009/01/11(日) 01:28:30 ID:???
>>521
筋道だった反論。勉強になった。
525ご冗談でしょう?名無しさん:2009/01/11(日) 01:33:45 ID:yCjoYbXp
>>415
朝永
526ご冗談でしょう?名無しさん:2009/01/11(日) 01:55:06 ID:???
疲労破壊の知識ゼロの>>516に呆れた。
まぁ高校物理も知らないようだから仕方ないのか。
527353:2009/01/11(日) 02:26:32 ID:???
すいません、どなたか>>353お願い致します。
528ご冗談でしょう?名無しさん:2009/01/11(日) 02:37:28 ID:3c5/IrvJ
古典線形群について質問します。

複素一般線形群GL(n,C)が連結であるのに実一般線形群GL(n,R)が
連結でないのは、究極的にはどういう事実によるのでしょうか。

大雑把な指針とかヒントで構わないので、お願いします。
529ご冗談でしょう?名無しさん:2009/01/11(日) 02:38:32 ID:???
>>524
自演ワロタ
530ご冗談でしょう?名無しさん :2009/01/11(日) 02:45:57 ID:kOBoH2V2
軽量鉄骨の家に住んでいます。
寒い日の朝などには、外壁の外側表面に結露が発生します。
中に鉄骨が入っている部分に沿って結露ができるのですが、そのメカニズムがいまいちよくわかりません。
鉄骨がある部分は室温を外壁に伝えやすく、周りより暖かくなるはずですが、なぜそこで結露が起きるのかというのが疑問です。
531ご冗談でしょう?名無しさん:2009/01/11(日) 02:47:56 ID:???
地面へ熱が逃げるからじゃね?
532ご冗談でしょう?名無しさん:2009/01/11(日) 02:49:20 ID:???
>>528
GL(1,R)=R-{0}は連結でない
GL(1,C)=C-{0}は連結
の一般化、じゃダメ?
533528:2009/01/11(日) 02:59:22 ID:???
>>532
>GL(1,R)=R-{0}は連結でない
>GL(1,C)=C-{0}は連結

そうでした。位相空間に不慣れなもんで「実数は連結」とばかり
思い込んでいたので、混乱してたみたいです。

まじすっきりしました。どうもありがとうございます
534ご冗談でしょう?名無しさん:2009/01/11(日) 03:28:52 ID:???
>>529
いい加減にしろよ、見苦しい。
535ご冗談でしょう?名無しさん:2009/01/11(日) 04:50:33 ID:???
>>514
単に通常、一次微分までの汎関数に限定してるだけだな。
一般には高次微分項がある場合も考えられるよ。
536ご冗談でしょう?名無しさん:2009/01/11(日) 09:01:06 ID:???
541 すずめちゃん(アラバマ州) 2009/01/11(日) 07:03:53.32 ID:4L4ovtWu
工作しまくってた石川ってIPA職員か知らんが
あそこひでえよ。
ようするにITセミナー会社なんだが、中身がHPに書いてない。
じゃあどうやって営業してんだと。
毎年同じ売り上げ。決算はぎりぎり赤字な
つまり関係会社の社員をよこしてマッチポンプ営業だろ。
そんで補助金もらう分だけ人件費やら設備購入で
みんなで山分けなわけだ。
ようするに泥棒してんじゃねーか。泥棒石川IPA

553 すずめちゃん(大阪府) sage 2009/01/11(日) 07:08:52.85 ID:rCGQMeB6
>>541
マジかよ
官僚 政治家 企業 で税金泥棒しとる訳やな
あぁぁぁぁぁぁぁぁ 

アホクサ 真面目に働いて税金キチンと納めとるのが馬鹿らしぃ
537ご冗談でしょう?名無しさん:2009/01/11(日) 10:43:23 ID:UrFmjfMK
星が光ることができるのは内部での核融合のエネルギーのためですか?
詳しく教えていただきたいです
538ご冗談でしょう?名無しさん:2009/01/11(日) 12:03:10 ID:???
ブラックホールに万有引力の法則が通用しないと聞きましたが本当でしょうか?
539ご冗談でしょう?名無しさん:2009/01/11(日) 12:05:44 ID:???
本当です。
540ご冗談でしょう?名無しさん:2009/01/11(日) 12:12:39 ID:???
>>539
なんでですか?
541ご冗談でしょう?名無しさん:2009/01/11(日) 12:13:27 ID:???
どうしても。
542ご冗談でしょう?名無しさん:2009/01/11(日) 12:13:57 ID:???
>>541
なんでなのか教えてください
543ご冗談でしょう?名無しさん:2009/01/11(日) 13:00:33 ID:???
>>542
物理学はHowを追究する学問です。Whyを追究するなら哲学です
544ご冗談でしょう?名無しさん:2009/01/11(日) 13:02:00 ID:???
>>543
答えられないんですね?わかりました。
545ご冗談でしょう?名無しさん:2009/01/11(日) 13:14:41 ID:???
答えられなくても問題ない。
546ご冗談でしょう?名無しさん:2009/01/11(日) 13:24:17 ID:???
>>545
ここは質問スレじゃないんですか?
547ご冗談でしょう?名無しさん:2009/01/11(日) 13:26:38 ID:???
議論が微妙にかみ合っていないな

質問する方は、
ブラックホールのどういうところが、万有引力の法則で説明できないのですか?
その部分については、どういう説明がなされていますか?
ってことがききたそうだ。

だが、ブラックホールスレか相対論スレで訊くのがいいんじゃないか?
548ご冗談でしょう?名無しさん:2009/01/11(日) 13:31:46 ID:???
>>547
ブラックホールスレで質問しましたが適応できないと言われてこっちにきました。
549ご冗談でしょう?名無しさん:2009/01/11(日) 13:38:21 ID:???
万有引力って、要するにニュートン力学(笑)のレベルでBHを語れってか?
550ご冗談でしょう?名無しさん:2009/01/11(日) 13:40:29 ID:???
>>549
いや、自分の質量以上の物吸い込めるかな?と思って質問してみましたがスレチのようなので消えます。
551ご冗談でしょう?名無しさん:2009/01/11(日) 13:46:55 ID:???
>>550
すごいな 後出しジャンケンか
552ご冗談でしょう?名無しさん:2009/01/11(日) 13:48:01 ID:???
>>550
ニュートン力学以前の問題のようだな
553ご冗談でしょう?名無しさん:2009/01/11(日) 13:48:57 ID:???
>>537にも誰か答えてー
554ご冗談でしょう?名無しさん:2009/01/11(日) 13:50:42 ID:UrFmjfMK
>>550
万有引力すら理解してないわけか 
よくもまあ偉そうに質問しやがって
555ご冗談でしょう?名無しさん:2009/01/11(日) 13:53:26 ID:???
>>553
恒星でググって死ね
556ご冗談でしょう?名無しさん:2009/01/11(日) 13:55:17 ID:???
>>554
お前は説明できんのか?w
557ご冗談でしょう?名無しさん:2009/01/11(日) 13:58:41 ID:UrFmjfMK
なんで死ななきゃいけないの!?なんで死ななきゃいけないの!?なんで死ななきゃいけないの!?なんで死ななきゃいけないの!?なんで死ななきゃいけないの!?なんで死ななきゃいけないの
558ご冗談でしょう?名無しさん:2009/01/11(日) 13:59:20 ID:UrFmjfMK
なんでなんでなんでなんでなんでなんでなんでなんでなんでなんでなんでなんでなんでなんでなんでなんでなんでなんでなんでなんで
559ご冗談でしょう?名無しさん:2009/01/11(日) 14:06:48 ID:???
>>556
ニュートン力学とは無関係な話だからな
560ご冗談でしょう?名無しさん:2009/01/11(日) 14:07:31 ID:???
>>553
そのとおり
561ご冗談でしょう?名無しさん:2009/01/11(日) 14:10:18 ID:???
>>559
>ニュートン力学とは無関係な話だからな
ちがうな。遠くではニュートン力学でもOK
近くなると、ものすごく重力が強くなって、
周りのものを何でも吸い込むようになる。
それが、ブラックホールてやつ。
562ご冗談でしょう?名無しさん:2009/01/11(日) 14:11:20 ID:???
>>561
シュヴァルツシルト半径に入ったらの話でしょ?
563ご冗談でしょう?名無しさん:2009/01/11(日) 14:13:22 ID:???
否、ブラックホールの近くには、

    近づくな危険

と、看板がかかっているよ。見るだけでも危ないかもしれない。
564ご冗談でしょう?名無しさん:2009/01/11(日) 17:30:56 ID:???
>>561
良く嫁。質問は

>自分の質量以上の物吸い込めるかな?と思って質問してみましたが

だ。これのどこがニュートン力学か?
565ご冗談でしょう?名無しさん:2009/01/11(日) 18:16:14 ID:???
横レスだが
そもそも吸い込めるとか吸い込めないとかって
考え方自体がどうかと思う
ブラックホールって言う言葉がいかんのだろうね
566ご冗談でしょう?名無しさん:2009/01/11(日) 18:22:57 ID:uLB5Mtvp
あ、>>487がスルーされてる。
もう一度。


「素粒子」って質量を持たないの?
567ご冗談でしょう?名無しさん:2009/01/11(日) 18:34:14 ID:???
持つものもあるし0のものもあるけど
知ってる素粒子のデータを数個調べたらわかるだろ

試しに知ってる素粒子の名前並べてみ
568ご冗談でしょう?名無しさん:2009/01/11(日) 18:39:22 ID:???
>>566
電子ってなんだと思ってる?
569ご冗談でしょう?名無しさん:2009/01/11(日) 18:40:37 ID:???
LHCでもしブラックホールができて蒸発しなかったらどんな被害が予想されますか?
570ご冗談でしょう?名無しさん:2009/01/11(日) 18:43:20 ID:???
ホーキングが喜び驚き悲しむ。
571ご冗談でしょう?名無しさん:2009/01/11(日) 18:46:32 ID:???
>>570
地球にはどのような被害がありますか?
572ご冗談でしょう?名無しさん:2009/01/11(日) 18:49:33 ID:uLB5Mtvp
>>567
定義が曖昧ですか。
ボソン粒子、クウォーク、ニュートリノ、ゲージ粒子、ヒッグス粒子、バリオン、スレプトン、ゲジーノ、イオン、
ぱっと思いつくのはコレくらいです。(+電荷量による、素粒子の違いもあるが)

>>568
語りきれないので、超大幅にはしょって「素粒子を構成する中の一つ」。
573ご冗談でしょう?名無しさん:2009/01/11(日) 18:50:29 ID:???
>>571
たぶん、誰も気づかない。

ってはなしを何回繰り返せばいいんだろうね。
574ご冗談でしょう?名無しさん:2009/01/11(日) 18:51:25 ID:???
>>573
なんでですか?一瞬で吸い込まれるから?
575ご冗談でしょう?名無しさん:2009/01/11(日) 18:52:47 ID:???
>>573
データにはちゃんと現れるっしょ。
576ご冗談でしょう?名無しさん:2009/01/11(日) 18:55:14 ID:???
俺は説明にあきるほど説明したから、過去スレ探せ。>>574

あるいはあきてない誰かを死ぬまで待ってろ。

577ご冗談でしょう?名無しさん:2009/01/11(日) 19:15:55 ID:uLB5Mtvp
>>576
別人だけど、追加の問い。
>>576=>>573として。
その、「LHC」の定義、細かく何なのかっちゅう話になると相違があるかも知れんが。

仮説として、ブラックホールに第三者的立場から見たら共有する時間及び空間は一定のものだから。
ゆえに、時空を共有している我々はその変動に淘汰される事は無い。
って事?(>>573は)

もう少し突っ込むと、シュヴァルツチャイルドが唱えた重力半径が及ばない程度にこの実験が行われた場合。
物理屋の想像を絶する(後に検証してみれば、至極当然かもしれないが)結果もありえるのでは。

この仮定による、例はいくらでも挙げられる。
現在の物理法則に則った手法でも、それは可能だ。
578ご冗談でしょう?名無しさん:2009/01/11(日) 19:18:10 ID:???
> シュヴァルツチャイルド
579ご冗談でしょう?名無しさん:2009/01/11(日) 19:35:43 ID:???
ロートチャイルド/ロスシルト
580ご冗談でしょう?名無しさん:2009/01/11(日) 19:51:48 ID:???
LHCでブラックホールを作るには陽子同士の衝突じゃないとだめなんだよね?
581ご冗談でしょう?名無しさん:2009/01/11(日) 19:55:31 ID:???
なんで? 説明して。
582ご冗談でしょう?名無しさん:2009/01/11(日) 19:57:01 ID:???
>>581
あれ?違ったっけ?
583ご冗談でしょう?名無しさん:2009/01/11(日) 20:01:38 ID:???
だから何でそう考えるの?
584ご冗談でしょう?名無しさん:2009/01/11(日) 20:02:23 ID:???
>>583
いや、宇宙線とLHCじゃ条件が違うって聞いたから…
585ご冗談でしょう?名無しさん:2009/01/11(日) 20:05:06 ID:???
>>584
でどうして580になるの?
586ご冗談でしょう?名無しさん:2009/01/11(日) 20:06:48 ID:???
>>585
高エネルギー宇宙線だと原子核が主だからLHCとは違うって言われたから…なんとなく陽子同士なのかなーって思っただけ。
587ご冗談でしょう?名無しさん:2009/01/11(日) 20:08:16 ID:???
なんとなく(笑)
588ご冗談でしょう?名無しさん:2009/01/11(日) 20:13:53 ID:???
>>587
原子核でもできるの?
589ご冗談でしょう?名無しさん:2009/01/11(日) 20:47:45 ID:???
なにこのギスギスした問答は?
590ご冗談でしょう?名無しさん:2009/01/11(日) 21:34:02 ID:???
>>572
データ調べろつってんだから
個別の粒子の名前を挙げてそれ調べればいいだろ
そんな分類たくさん並べなくていいから
具体的に1個2個調べればいいんだよ
無駄に頭でっかちだね
591ご冗談でしょう?名無しさん:2009/01/11(日) 22:03:56 ID:uLB5Mtvp
あ、やっとキタ。

>>590
別人かもしれないが、
>>567
>試しに知ってる素粒子の名前並べてみ
って言ってるじゃん。。

怒られちった。

あと、調べてもわからない。
わからないというのは、小生の問いの答えが載ってなかったというわけでもない。
その真偽に疑義が生じる。
君の考えを教えてくれ。
592ご冗談でしょう?名無しさん:2009/01/11(日) 22:08:23 ID:???
答えをもらう側なのになんでそんなに挑戦的なの?
そんな態度じゃ誰も教えてくれないぞ。
593ご冗談でしょう?名無しさん:2009/01/11(日) 22:08:32 ID:???
素直になれって。分類名は知ってても光子や電子が
素粒子だってことは知らないのか。
知ってたらそれ調べれば済む話だからな。

この感じだと対称性の破れ云々を知りたいわけじゃないだろうし。
594ご冗談でしょう?名無しさん:2009/01/11(日) 22:10:00 ID:???
>>591
http://ccwww.kek.jp/pdg/
ここに載っていることで疑義がある点を一つ挙げてくれ。
595ご冗談でしょう?名無しさん:2009/01/11(日) 22:22:15 ID:uLB5Mtvp
>>322
「音」って質量を持たないの?

>>373
「光」って質量を持たないの?

>>487
「素粒子」って質量を持たないの?


一日一題、皆の考え方を聞きたかっただけじゃ。
音と光では、いい議論を聞けたんだけどな。
週末は、人も変わるんだね。
596ご冗談でしょう?名無しさん:2009/01/11(日) 22:25:04 ID:???
たとえば、電子という素粒子には質量があります。光子という素粒子には質量はありません。
597ご冗談でしょう?名無しさん:2009/01/11(日) 22:30:25 ID:uLB5Mtvp
コンプトン効果ですね。
エネルギー(そっからの定義も難しいが)を持つ粒子に質量(ここの定義も難しい)がない。
そこにも疑問があります。
598ご冗談でしょう?名無しさん:2009/01/11(日) 22:32:55 ID:???
光って質量を持たないんですよね?
ぜも全部の粒子がクォークから出来てることから考えて矛盾しません?
599ご冗談でしょう?名無しさん:2009/01/11(日) 22:35:04 ID:???
まるでどちて坊やだな
600ご冗談でしょう?名無しさん:2009/01/11(日) 22:35:25 ID:???
>>598
> 全部の粒子がクォークから出来てる
わけではないが、もしそうだとしたらどうして
> 光って質量を持たない
と矛盾するのだ?
601ご冗談でしょう?名無しさん:2009/01/11(日) 22:36:45 ID:uLB5Mtvp
>>598
全ての粒子ではない。
クウォークは、素粒子学上数ある分類の中の一つ。

一般的?には、クウォークといったら全ての物質を司る素粒子の元凶と誤認識されるかもしれないが、当該学問上は一粒子。

ですよね?
602ご冗談でしょう?名無しさん:2009/01/11(日) 22:47:16 ID:???
横着しないで本読めよ
603ご冗談でしょう?名無しさん:2009/01/11(日) 23:40:31 ID:LNARZGaK
3000おく使いたいな…

一番ちかみちビルクリントンかビル持ち逃げかビルボードとか頼むよ。
604ご冗談でしょう?名無しさん:2009/01/12(月) 00:25:42 ID:???
>>598

お前結局分類名「並べただけ」で結局こっちの言うことひとつもしなかったな
605ご冗談でしょう?名無しさん:2009/01/12(月) 00:27:50 ID:???
古典力学を学ぶのはなぜですか。
現代物理を学ぶのに得になりますか?
606ご冗談でしょう?名無しさん:2009/01/12(月) 00:35:50 ID:???
・古典力学には物理的思考のエッセンスが詰まってる
・日常の範囲なら古典力学で十分
・古典力学で記述し得ない現象の理論も全て古典力学の拡張として作られる
・古典力学で記述し得ない領域でも古典力学のアナロジーとして理解できることがある
(Lorentz振動子や有効質量など)
607ご冗談でしょう?名無しさん:2009/01/12(月) 00:59:40 ID:???
エネルギーを持つ粒子は、光子であろうと、重力子であろうと、質量を持つ。
これが、唯一の正しいアインシュタインの教えだ。
608ご冗談でしょう?名無しさん:2009/01/12(月) 02:02:41 ID:???
"It is not good to introduce the concept of the mass M = m/(1-v2/c2)^(1/2)
of a body for which no clear definition can be given. It is better to introduce no
other mass than 'the rest mass' m. Instead of introducing M, it is better to
mention the expression for the momentum and energy of a body in motion."
--- A. Einstein (1948)
609ご冗談でしょう?名無しさん:2009/01/12(月) 04:17:29 ID:???
なんて書いてるの?
610ご冗談でしょう?名無しさん:2009/01/12(月) 04:55:43 ID:RjhiLwPa
>>609、アインシュタインが>>607のような考えは良くない、と言っている。
>>607のような謂いは否定はしてないが。で、>>608の立場からは光子の
質量は0。
 存在する粒子の4元運動量の時間成分は必ず0でないので、それを
もって「質量」のあるなしを議論する事は不毛。(この意味での質量0は
あり得ないから)ところがローレンツ不変量E^2-p^2c^2は0のことも
(光子)0でないことも(電子等)あるので、この区別は重要。理論的にも。
>>608のrest massとはE^^2-p^2c^2の平方根をc^2で割ったもの、静止質量。
611ご冗談でしょう?名無しさん:2009/01/12(月) 07:09:34 ID:???
単に質量というと、どういう意味の質量を言ってるか分からんから
よく混乱の元になってるな。
まぁ、素粒子論では粒子の質量と言えばまずpole massの事を指すのが普通で
粒子単体の静止質量でもあるわけだが、それは光子ならゼロ。
612ご冗談でしょう?名無しさん:2009/01/12(月) 10:27:42 ID:???
もしかして時間は2つ以上あるんじゃないでしょうか。
613ご冗談でしょう?名無しさん:2009/01/12(月) 10:36:03 ID:???
ディラックは時間は複数あると言っていたよ。
朝永博士は時間は無限にあると言った。
で、それって、実験的に証明されたの?
614ご冗談でしょう?名無しさん:2009/01/12(月) 11:31:20 ID:???
>>613自身の時間の定義が曖昧みたいだから、そっちを先に確認してみてから
再度質問しにきてみたらどうだろうか。
615ご冗談でしょう?名無しさん:2009/01/12(月) 11:43:22 ID:???
pole massってなに?
616ご冗談でしょう?名無しさん:2009/01/12(月) 11:48:21 ID:???
円柱状にして測定した重さ。
617ご冗談でしょう?名無しさん:2009/01/12(月) 12:13:58 ID:DdJD2QL4
こんにちは。
一般相対論的宇宙論における
銀河の後退速度について質問があります。

「銀河の後退速度が光速を超える場合がある」

ということを主張する人がいますが、
私の理解では、大域的な(100Mpc以上の広さ)領域における
宇宙膨張の効果による銀河の後退速度を考える際の前提である
一様等方時空(RW時空)がミンコフスキー時空でない為、
大域的には、我々の銀河から見た他の銀河の後退速度(=相対速度)を
定義することができず、よって、銀河の後退速度が光速を超えると
主張することは、大域的には、仮定に反する為ナンセンスであると
思うのです。

また、これはハッブル則がZ<<1(そこでは特殊相対論における
ドップラー偏移が厳密に成り立つと考えてよい領域)でのみ
成立することを考えると、定義ではなく、物理的な意味を持つ
後退速度はやはり光速を超えることはないのではないかとも
思うのです。

いろんな相対論のスレを見ても肝心な理由が詳細に議論されて
おらず、しっくりきていません。
もちろん、私がまだ不勉強な為、何かを誤解しているのかも
しれません。

上記、私の理解に間違いがあれば、専門の方のご意見をお聞かせ
願いないでしょうか。
618ご冗談でしょう?名無しさん:2009/01/12(月) 12:33:23 ID:???
>>617
後退速度を定義できないのは間違い。星と星の距離が定義できて、一定時間
後の距離が定義できれば、速度は定義できます。RW時空なら特に簡単。

そして、その定義のもとで速度が光速を越えることもある。RW時空でも。

619ご冗談でしょう?名無しさん:2009/01/12(月) 12:42:52 ID:???
なるほど。よく分かりました。有り難う。
620ご冗談でしょう?名無しさん:2009/01/12(月) 12:51:48 ID:UvD3vRdW
กะรัน
กะรัน
621KingMind ◆HPxPvj7m.s :2009/01/12(月) 13:04:14 ID:rZsoZk7u
King王国の住人です。よろしく!
622ご冗談でしょう?名無しさん:2009/01/12(月) 13:12:04 ID:???
>>613
やっぱり、あるんですね。
623ご冗談でしょう?名無しさん:2009/01/12(月) 13:12:57 ID:???
あるんです。
624ご冗談でしょう?名無しさん:2009/01/12(月) 13:14:21 ID:???
時間は時間平面といって、連続なようだけど微少なすきまがあって不連続だよ
625ご冗談でしょう?名無しさん:2009/01/12(月) 13:16:16 ID:???
やっぱりね。
626ご冗談でしょう?名無しさん:2009/01/12(月) 13:39:06 ID:???
ど素人からの質問です。

摩耗がなくなるのは不可能なのでしょうか?
車をいじったりして油まみれになってる人をみるとよくそう思ってしまいます。
627ご冗談でしょう?名無しさん:2009/01/12(月) 14:05:37 ID:???
磁気軸受が普及すれば…
628ご冗談でしょう?名無しさん:2009/01/12(月) 14:13:13 ID:???
時間は意志を持ちますか?
629ご冗談でしょう?名無しさん:2009/01/12(月) 14:13:40 ID:h/QBMvsN
>>618
返答に本当に感謝します!
素人ですいません。
もう少し、議論にお付き合い願います。

RW時空でも銀河の後退速度(=我々の銀河と十分遠方の銀河との
相対速度)を定義できるとのことでした。
しかしながら、RW時空全域を1つの慣性系で覆うことはできない為、
多様体論における異なる接空間に属するベクトルの差の計算が
計算できないのと同じ理由により、相対速度は計算できない、
と私は思っておりました。
(最初の段落では、上記の意味で、後退速度が定義できないのでは
ないかと私は申しました。)
この認識が間違っているということでしょうか?

追記:
私が前コメントで書いたように、ハッブル則では
便宜上、z>>1でもv=czを後退速度と「定義」することは
ありますよね。でもこれは、便宜上であって、光速を超えて
遠ざかる銀河の存在を意味するわけではありませんよね。
(参考文献 須藤さん著 一般相対論入門 P122)
よって、やはり、物理的に意味をもつ後退速度は光速を
超えないと私は思っていましたが・・・・。
630ご冗談でしょう?名無しさん:2009/01/12(月) 14:54:27 ID:???
>>627
ありがとうございます。リニアモーターカーに使われてる技術ですね。
じゃあ弱点は磁力が弱くなることか。。
ふむふむ。
631ご冗談でしょう?名無しさん:2009/01/12(月) 15:01:00 ID:???
光速に上限があり、それを超えるか否かに拘り、思考するのはなぜだ?
「因果律」を破らない様にするためだろう。
後退速度が光速を超えて、「因果律」が破られるのか?
破られないなら、拘る必要なし。
632ご冗談でしょう?名無しさん:2009/01/12(月) 15:11:04 ID:???
>>629
物理的に意味をもつ後退速度って何?

長い定規置いといて、今と1秒後の銀河の位置測って
その差が速度と定義したらいいんじゃない?
633ご冗談でしょう?名無しさん:2009/01/12(月) 16:26:47 ID:???
>>629ちょっと見だけで横レスするが、>>630の言う通りでいいんじゃね?
>多様体論における異なる接空間に属するベクトルの差の計算が
>計算できないのと同じ理由により、相対速度は計算できない、
>と私は思っておりました。
なんて意味での「相対速度」なんてこの場合端から問題にしないで
>>632のいうような速度を考えているのでは?(物理的に意味があるかは
さておいて数学的には定義可能)
634ご冗談でしょう?名無しさん:2009/01/12(月) 16:48:37 ID:???
自然座標系c=h/2π=1でエントロピーSと温度Tの次元ってどうなりますか?
それぞれ質量の0次、1次っぽいけど
これでは、松原理論の座標の次元:質量^-1と合わないんだがどうなの?
635ご冗談でしょう?名無しさん:2009/01/12(月) 17:03:38 ID:ojh1zU9t
返答ありがとうございます!

>>631
因果律との関連のご指摘、本当にありがとうございました。
たしかに光速を超える後退速度の存在は、因果律とは、
矛盾しないと思います。
しかしながら、後退速度が光速を超えるような十分遠ざかった
銀河間において、時空はRW時空で記述する必要があります。
そのような大域的な領域において後退速度を考えることは、
多様体論と矛盾しないのでしょうか?
(私が誤解していたらすみません。)


>>632
ここで、私が考える物理的に意味を持つ後退速度とは、
ハッブル則(遠方の銀河が距離に応じて
赤方偏移する)のv=cz=Hd (ただしz<<1 v<<c)のv
であり、
我々の銀河から遠ざかる銀河からのスペクトル線の
ドップラー偏移から観測可能な、銀河の後退速度vのことです。
一方、須藤さんの著作にあるような便宜上の後退速度v=cz (z>>1)は、
ドップラー偏移の仮定v<<cを無視して、z>>1でもv=cz と定義した点において、
「便宜上」であると私は考えています。


・・・・素人でホントすいません(^^;
(続く)
636ご冗談でしょう?名無しさん:2009/01/12(月) 17:05:24 ID:???
S=k log Wで、[T]=[U/S]=[U/k]=[mc^2/k]だからBoltzmann定数の次元によるでしょ
c=ħ=1に加えてk=1なら当然[S]=[1], [T]=[m]で、
[x]=[h/p]=[h/mc]=[1/m]と矛盾することも特にないと思うんだが
637ご冗談でしょう?名無しさん:2009/01/12(月) 17:26:56 ID:ojh1zU9t
返答ありがとうございました!
素人にたびたびお付き合い頂き、感謝します。

>>632

> 長い定規置いといて、今と1秒後の銀河の位置測って
> その差が速度と定義したらいいんじゃない?

この方法は、実際に光速を越えるような後退速度の存在を
観測で検証する際に用いることができますかね?(^^;

また、理論的に考えても、後退速度が光速を超える場合、
いったいどうやって、1秒後の銀河の位置(=定規の目盛り)を
読むことができるのでしょうか?

>>633
>なんて意味での「相対速度」なんてこの場合端から問題にしない

そうなんですか?何故でしょう。。。
638ご冗談でしょう?名無しさん:2009/01/12(月) 17:30:47 ID:???
>>618 が定義する「後退速度」と >>629 が定義する「後退速度」が異なる量だというだけのことのような。
639ご冗談でしょう?名無しさん:2009/01/12(月) 18:21:06 ID:???
今ひじょーに物理が苦手なんですが数学の様にやればできるようになりますかね?
640ご冗談でしょう?名無しさん:2009/01/12(月) 18:35:50 ID:UUxsEjMa
<問いままだ完全に解決していない為、消えないよう、再書き込みします。>

こんにちは。
一般相対論的宇宙論における
銀河の後退速度について質問があります。

「銀河の後退速度が光速を超える場合がある」

ということを主張する人がいますが、
私の理解では、大域的な(100Mpc以上の広さ)領域における
宇宙膨張の効果による銀河の後退速度を考える際の前提である
一様等方時空(RW時空)がミンコフスキー時空でない為、
大域的には、我々の銀河から見た他の銀河の後退速度(=相対速度)を
定義することができず、よって、銀河の後退速度が光速を超えると
主張することは、大域的には、仮定に反する為ナンセンスであると
思うのです。

また、これはハッブル則がZ<<1(そこでは特殊相対論における
ドップラー偏移が厳密に成り立つと考えてよい領域)でのみ
成立することを考えると、定義ではなく、物理的な意味を持つ
後退速度はやはり光速を超えることはないのではないかとも
思うのです。

いろんな相対論のスレを見ても肝心な理由が詳細に議論されて
おらず、しっくりきていません。
もちろん、私がまだ不勉強な為、何かを誤解しているのかも
しれません。

上記、私の理解に間違いがあれば、専門の方のご意見をお聞かせ
願いないでしょうか。
641ご冗談でしょう?名無しさん:2009/01/12(月) 18:38:40 ID:UUxsEjMa
>>638

そうなんですかねー(^^;

探していたら、私の意見に近い事
主張しているように見えるHPありました。

ttp://www.sci-museum.kita.osaka.jp/~ishizaka/cosmology4.html



642ご冗談でしょう?名無しさん:2009/01/12(月) 19:02:27 ID:M0su7Pgz
「ヒッグス粒子」って質量を持たないの?
643ご冗談でしょう?名無しさん:2009/01/12(月) 19:38:00 ID:???
持つよ
644蔵本天外大先生:2009/01/12(月) 19:55:21 ID:SK2U+s19
蔵本天外尊師〔シンビインターナショナル所属〕
http://www.sinvi-international.com/
が新たな悟を開かれております。
量子力学についてやハーバードの学長の話を持ち出してます。
http://jp.youtube.com/watch?v=smAWm3g9Wns&feature=related
http://academy6.2ch.net/test/read.cgi/psycho/1208076147/l50
これからは物理性心理学の時代です。
645ご冗談でしょう?名無しさん:2009/01/12(月) 20:20:13 ID:???
熱伝導方程式の解はsin cosの重ね合わせと指数関数の積で
表せると書いてあったんですが
そうすると長いスパンでは温度が低くなっていっても
小さなスパンでは距離が伸びているのに
温度が高くなったり低くなったりするん
でしょうか?
646ご冗談でしょう?名無しさん:2009/01/12(月) 20:24:48 ID:???
そりゃそういう条件ならそうなるだろう。
647ご冗談でしょう?名無しさん:2009/01/12(月) 20:28:49 ID:LX3M7sdI
高校物理の基礎的な重力計算ででてくるy_maxって何のことですか?
648ご冗談でしょう?名無しさん:2009/01/12(月) 20:33:48 ID:???
> 高校物理の基礎的な重力計算ででてくるy_maxって何のことですか?

「基礎的な重力計算」とかいうのが具体的になんなのか書いたほうがいいんじゃねーの?
649ご冗談でしょう?名無しさん:2009/01/12(月) 20:37:23 ID:???
多分放物線の頂点の高さ、中学物理になるけど
650ご冗談でしょう?名無しさん:2009/01/12(月) 20:49:35 ID:???
たぶんそれです
ありがとうございました
651ご冗談でしょう?名無しさん:2009/01/12(月) 23:30:49 ID:???
>>645
どんな関数もサインコサインで書けるよってフーリエが言ってた
652ご冗談でしょう?名無しさん:2009/01/12(月) 23:30:57 ID:???
>>646
645への答えでしょうか?
了解しました。ありがとうございます。
653ご冗談でしょう?名無しさん:2009/01/13(火) 03:29:25 ID:???
ttp://www.tuat.ac.jp/~katsuaki/kzb040423.html
上記のリンクのキュリーの法則から
ランジェバンの理論の<μj>を求める計算過程がわかりません。
計算過程なので板違いかもしれませんが、解説お願いします。
654ご冗談でしょう?名無しさん:2009/01/13(火) 08:47:06 ID:???
速度がマイナスという状態は存在しますか?
655ご冗談でしょう?名無しさん:2009/01/13(火) 08:50:35 ID:???
します
656ご冗談でしょう?名無しさん:2009/01/13(火) 08:53:13 ID:???
交流回路の場合、電流や電圧が複素数になるらしいのですが、
複素数の電流とは存在するのでしょうか?
数学では複素数は現実に存在しないと習いました
657ご冗談でしょう?名無しさん:2009/01/13(火) 08:58:54 ID:???
>>655
どんな状態ですか?圧縮されるんですか?
658ご冗談でしょう?名無しさん:2009/01/13(火) 09:01:23 ID:???
>>656
実数は現実に存在するのか、実数の電流とは存在するのか、
それに自分なりに論理的一貫した答えを出してからまた質問にきて下さい
659ご冗談でしょう?名無しさん:2009/01/13(火) 09:20:56 ID:???
>>658
>実数は現実に存在するのか
存在します。
660ご冗談でしょう?名無しさん:2009/01/13(火) 10:22:09 ID:???
こちらとしては>>653が計算過程の何が分からないかが分からないので
答えようがありません。
661ご冗談でしょう?名無しさん:2009/01/13(火) 10:35:59 ID:???
>>659
なぜそういえるのですか?
662ご冗談でしょう?名無しさん:2009/01/13(火) 11:11:04 ID:???
つ 数直線
663ご冗談でしょう?名無しさん:2009/01/13(火) 12:15:08 ID:???
>>662
つ 複素平面
664ご冗談でしょう?名無しさん:2009/01/13(火) 16:24:42 ID:???
>>661
数学の「集合論」系の本に載ってる実数の定義を見てこい。
665656:2009/01/13(火) 17:00:29 ID:???
結局複素数の値の電流・電圧とはなんなのでしょうか?
666ご冗談でしょう?名無しさん:2009/01/13(火) 17:09:28 ID:???
数なんてのは、所詮、計算の道具に過ぎん。
667ご冗談でしょう?名無しさん:2009/01/13(火) 17:15:46 ID:???
>>664
有理数からなるCauchy列の商集合の元が存在すると主張できる論拠は?

>>665
V(t)をFourier変換したV(ω)とか
668ご冗談でしょう?名無しさん:2009/01/13(火) 17:18:24 ID:???
陽子はp、中性子はnで表しますよね。
もしかしてクォークを最初に考えた人は、pを点対象に引っくり返してd、nを点対称に引っくり返してuという記号を考えて
dとuを頭文字に持って正反対の意味を持つ言葉を考えてから、dをダウン、uをアップという風に名前を後付けで考えたのですか?
669ご冗談でしょう?名無しさん:2009/01/13(火) 17:38:03 ID:???
昔トップクォークってのが最小単位だと思っていたよ
670ご冗談でしょう?名無しさん:2009/01/13(火) 17:49:55 ID:???
>>667
そもそもCauchy列って何か分かってる?
671ご冗談でしょう?名無しさん:2009/01/13(火) 18:10:59 ID:???
>>670
教えて。
672ご冗談でしょう?名無しさん:2009/01/13(火) 18:43:02 ID:???
>>668
おもしろいこと言うやつだな

マジレスすると、スピンと似た数学的性質を持つアイソスピンという物理量があって、
スピンと同じように上向き・下向きの状態がある。で、アイソスピンが上向きと下向きの
クォークにそれぞれupとdownという名がついた。それだけ。
673ご冗談でしょう?名無しさん:2009/01/13(火) 19:40:35 ID:???
strange,charmまではまだ命名の理由がわからんでもないが、bottom,topに至ってはどういう理由で
その語が選ばれたのかさっぱりわからんな。
674ご冗談でしょう?名無しさん:2009/01/13(火) 19:55:54 ID:???
>>670
有理数の場合は通常の意味で定義される距離をd(x,y)として、
∀ε>0 ∃N s.t. n,m>N ⇒ d(x_n, x_m)<ε
を満たす数列{x_n}とでも言っておく?

それにしても、自分が質問に答えられないから逆質問を繰り返すとは見苦しい
675ご冗談でしょう?名無しさん:2009/01/13(火) 20:12:06 ID:???
>>673確か南部さんの「クオーク」(ブルーバックス)にはおっぱいとおしりのことだ、って
出てたような。
676ご冗談でしょう?名無しさん:2009/01/13(火) 20:42:27 ID:???
なんで日本の科学者はネーミングセンスが無いんですか
677ご冗談でしょう?名無しさん:2009/01/13(火) 21:04:19 ID:???
bottomとtopにはそんな意味もあるってだけでは。そもそも南部さんの命名なのか?>bとt

野球の1回表とか9回裏とかいうときの表裏が英語ではtopとbottom。
クォークを世代ごとに
dct
usb
と並べてみたら野球のスコアボードを連想した香具師がいたのかな、と
678ご冗談でしょう?名無しさん:2009/01/13(火) 21:47:06 ID:???
あ、個人的想像です>>677
本当のところは知らない

ファインマンは他のことに関しては抜群だけどネーミングセンスだけはねぇ。パートンて・・・
堅物だけどネーミングセンスはイカしてたゲルマンとここでも好対照な2人
679ご冗談でしょう?名無しさん:2009/01/13(火) 21:49:06 ID:???
>>677マジレスすんな!南部氏の当該文章中だってほのめかしなのに。>>676
アメリカ人に文句言って。alminumだとか。まあ、日本人の場合でも生物系
以外ではラテン知らないし、かといって今では漢籍疎いから東洋的になんか重々しい
用語作れそうにないけど。(現代中国人も古典に親しんでいないかぎり出来なさそう)
680ご冗談でしょう?名無しさん:2009/01/13(火) 21:51:47 ID:???
おいらは、
 b=beauty
 t=truth
の方がすきやけどな。
681ご冗談でしょう?名無しさん:2009/01/13(火) 21:58:36 ID:???
状態方程式PV=NKTというのはどう導出すればいいのでしょうか?
p=KT(d/dV・ln(Z))からどうなるのか分かりません
682ご冗談でしょう?名無しさん:2009/01/13(火) 22:00:03 ID:???
マルチ乙
683ご冗談でしょう?名無しさん:2009/01/13(火) 22:01:27 ID:???
誤爆って書いたじゃん;;
684ご冗談でしょう?名無しさん:2009/01/14(水) 02:49:14 ID:???
>>654
「速度」ならベクトルだから、当然マイナスの状態は存在する。
「速さ」なら、マイナスの状態はないけどね。

>>656
>交流回路の場合、電流や電圧が複素数になるらしいのですが、
>複素数の電流とは存在するのでしょうか?
物理的実体として複素数の電流や電圧が存在するわけではなく、
単なる数学的技法と考えるのがいい。

交流回路でコンデンサーやらコイルやらがついていたりすると、
交流電流の位相が進んだり遅れたり色々な現象が起きて、
直流回路での法則が使えないわけだが、交流の電流や電圧を
複素数で表わしてみると上手い具合に既存の法則が使えたりする。
で、これは便利なので複素数で表わすようになった。

>数学では複素数は現実に存在しないと習いました
先生が無知か、お前の聞き間違いだな。
量子力学では複素数は現実に存在する。
685ご冗談でしょう?名無しさん:2009/01/14(水) 03:09:24 ID:???
>>684
>量子力学では複素数は現実に存在する。
て、そう断言できりゃ科学哲学者の仕事はなくなるわな
ある対象が実在するか否かは物理で扱う範疇にないということ
(無論数学でもない)

>>667にある通り、複素表示は単にFourier変換しただけなんだから、
(だから解析的操作が代数的操作に置き換わって御利益が多いわけだけど)
V(t)の実在認めるならV(ω)の実在認めてあげてもいいような気はするけどね
686ご冗談でしょう?名無しさん:2009/01/14(水) 07:29:49 ID:???
>>684
複素数っていうのは、単に二つの実数の組に対して演算を定義しただけで、
複素数という形を使わずとも、実数で扱う事ができる。
簡単な例としては線型代数的に扱う、ってのがあるな。
量子力学の演算は線型変換しかないから、線型代数で扱う事ができる。
むしろ、「量子力学では複素数は現実に存在する」という根拠を教えて欲しいのだが。
687ご冗談でしょう?名無しさん:2009/01/14(水) 08:16:04 ID:???
>量子力学の演算は線型変換しかない
クソワロタwwwwwwwwwwwwwwwwwwwwwwww
688ご冗談でしょう?名無しさん:2009/01/14(水) 08:24:10 ID:???
>>687????
>>686交流回路なんかの場合は元々実数で扱うべきものを複素数に
拡張すると便利、ということで複素数が出て来た。量子力学の場合は
いきなり複素数がでてくる。このあたりのことを>>684は言っているんだろう。
勿論いきなりでてくるものを2次元平面上の複素構造として捉えてあくまで
「実2次元」と称するのは人の勝手だが、では何故そんな構造が量子力学に
おいて必要だったか?という話。
689ご冗談でしょう?名無しさん:2009/01/14(水) 10:11:35 ID:???
なぜかはしらんが複素数の代数構造を利用するとうまくいった。物理の話としてはそれだけ。
690ご冗談でしょう?名無しさん:2009/01/14(水) 11:48:34 ID:c2EYFtHV
こんにしは。
640,641を書き込んだものです。
みなさん、
何か適切な反論はおありでしょうか(^^;
なければ、別のスレッドに移るなり
立てるなりを検討します。
691ご冗談でしょう?名無しさん:2009/01/14(水) 11:58:48 ID:???
そもそも「後退速度」は定義できないお! って言ってる訳で、光速関係ないな。
692曲率てんそる:2009/01/14(水) 14:25:10 ID:q2tpqjut
<<691さん

コメントありがとうございます!

>そもそも「後退速度」は定義できないお!って言ってる訳で、光速関係ないな。

いや、それは少し違うと思っています。
理由は、少なくともz<<1の領域では、ハッブル則 v=cz=Hd (v<<c)から、
物理的に意味のある銀河の後退速度を観測することが可能だからです。
私の理解では、z<<1の領域では、宇宙がハッブル則に従って膨張していても、
宇宙がひとつの慣性系で覆われると「仮定して」、
(∵z<<1では宇宙半径〜数十Mpcであり、この領域では
宇宙は一様等方ではない。つまり、RW時空で記述すべきでない。)
銀河からのスペクトル線のドップラー偏移から銀河の後退速度(=相対速度)を
観測する事は、「十分よい近似で」行なえると考えているからです。
つまり、z<<1では、銀河の後退速度は、
物理的に意味をもつ量として定義できる思っています。

よって、私が、後退速度が
(1) 大域的(宇宙半径〜cH^{-1}〜4300Mpc)には
  宇宙は完全に一様等方なので、RW時空で宇宙膨張を記述すべきであり、
  そこでは多様体論より、曲がっている時空でのベクトルの差は定義できない。
(2) v=cz (z>1)は便宜上の定義で、光速を超えて遠ざかる銀河の存在を
  意味するものではない。(須藤さん 一般相対論P122より)
の理由から、定義することがナンセンスであると思っている領域は、
後退速度が光速を超えるz>1の場合についてです。


素人です。間違っていたらすいません(^^;
693ご冗談でしょう?名無しさん:2009/01/14(水) 15:41:55 ID:???
相対論的には W = 1+z とおくとき、交代速度は
v = (W^2 - 1 ) / (W^2 + 1 ) c
だよ。特に、|z| << 1 のときは、
v= zc
と書ける。
694ご冗談でしょう?名無しさん:2009/01/14(水) 15:46:52 ID:???
>>692
結局 >>638 であって、>>618 が定義する量を後退速度とは呼びたくないってことでしょ
695ご冗談でしょう?名無しさん:2009/01/14(水) 15:58:24 ID:???
> 我々の銀河から見た他の銀河の後退速度(=相対速度)を定義することができないお
> 後退速度が光速超はナンセンスだお
696ご冗談でしょう?名無しさん:2009/01/14(水) 15:59:28 ID:???
後退速度は光速まで限りなくちかづくお!
光速を越えたら後退速度とはいわないお!
697ご冗談でしょう?名無しさん:2009/01/14(水) 16:02:48 ID:???
宮崎県知事の額の後退速度は光速を超える
698ご冗談でしょう?名無しさん:2009/01/14(水) 16:13:05 ID:???
なんか、必死の人が入るから、もぅひとこと。
後退速度は、宇宙論では使ってないよ。それは、宇宙モデルが確定しないと
zの値から速度へ換算ができないし、わざわざ速度に換算することに理論的
意味はないから。観測的宇宙論では、遠方の天体の距離は、zの観測値その
ものを使って表すし、宇宙論的には、1+zが、膨張の比率を表す量として重
要なのだ。例えば、z=3 なら、その天体が光が放たれてから宇宙は 1+z = 4倍
に膨張したことを意味する。

後退速度が重要なのは、Hubbleの法則の歴史的経過においてかな。
あと、銀河系近傍の銀河の運動の測定において、宇宙膨張の影響を分離する
ときにも重要だが、z<<1 の領域に限られる。
699曲率てんそる:2009/01/14(水) 16:28:40 ID:TGi0s6AP
<<695,696,698

コメントありがとうございます。
その意見に同意します。

すると、やはり
「銀河の後退速度が光速を超える場合がある」
と主張することはナンセンスですよね??
700ご冗談でしょう?名無しさん:2009/01/14(水) 16:38:16 ID:???
>>698
「後退速度」がもはや古語/死語ということですか?
701ご冗談でしょう?名無しさん:2009/01/14(水) 16:42:32 ID:???
>>699
>「銀河の後退速度が光速を超える場合がある」
>と主張することはナンセンスですよね??
693の式をつかわなだめだよ。けっして光速度はこえないが。
702ご冗談でしょう?名無しさん:2009/01/14(水) 16:45:38 ID:???
>>700
>「後退速度」がもはや古語/死語ということですか?
近傍の銀河の運動の観測ではいまも重要。
遠ざかっているように見えるの中に、天体の固有運動と宇宙膨張の両方が
入っているからね。
703曲率てんそる:2009/01/14(水) 17:02:30 ID:TGi0s6AP
<<693さん

え!?
そこでのzって宇宙論的赤方偏移のzじゃないでしょう。(^^;
それは光のドップラー偏移のzじゃないんですか?
704ご冗談でしょう?名無しさん:2009/01/14(水) 17:05:09 ID:???
おなじだよ。
宇宙論的赤方偏移は宇宙膨張をドップラー偏移(遠ざかっている)と解釈したのさ。
705曲率てんそる:2009/01/14(水) 17:32:51 ID:TGi0s6AP
>>704さん

いや、「一般に」同じではないでしょう。
698さんもおっしゃっていますが、
宇宙論的赤方偏移zと
特殊相対論でのドップラー赤方偏移での
後退速度vとを関連づけていいのは
z<<1のときだけですよ。

何故ならば、以前私がコメントしたように、
z>1となるような大域的領域(〜4300Mpc)では
特殊相対論は成立しませんから。(^^;

みなさんはどう思われますか?
706曲率てんそる:2009/01/14(水) 17:36:12 ID:TGi0s6AP
私の意見に近い事を
主張しているように見えるHPもご覧下さい。

ttp://www.sci-museum.kita.osaka.jp/~ishizaka/cosmology4.html

このHPの意見に私は同意します。
707ご冗談でしょう?名無しさん:2009/01/14(水) 17:44:59 ID:???
HPの宣伝かよw
708ご冗談でしょう?名無しさん:2009/01/14(水) 17:52:34 ID:???
>>705
>特殊相対論は成立しませんから。(^^;
宇宙膨張があれば、成立しないよ。
しかし、歴史的にはそんなの関係ないんだな。
ハッブルはzの値から「後退速度」vを求めたのだ。
「後退速度」は「遠ざかっているように見える」という見かけの話なの。
(宇宙が実際にどうなっているかは別の話。)
光なので、ドップラー公式は特殊相対論的なものを使うのが当然。
たとえば、こんな風にね。
http://www.nao.ac.jp/nao_news/mails/000398.txt

 しかし、z<<1では、v=czで観測誤差の範囲でOKだから、v=czが
よく使われだけ。
709曲率てんそる:2009/01/14(水) 17:54:27 ID:TGi0s6AP
>>707さん

ちがいますよ〜(^^;
(私はこのHPの管理者ではありません。)
私は、素人ながらも真剣に

「銀河の後退速度が光速を超える場合がある」
と主張することはナンセンスである

かどうかをハッキリさせたいだけです。
710ご冗談でしょう?名無しさん:2009/01/14(水) 18:00:53 ID:???
上限に光速を持ってくるお前がナンセンスだよ
711曲率てんそる:2009/01/14(水) 18:01:11 ID:TGi0s6AP
>>707さん

ちがいますよ〜(^^;
(私はこのHPの管理者ではありません。)
私は、素人ながらも真剣に

「銀河の後退速度が光速を超える場合がある」
と主張することはナンセンスである

かどうかをハッキリさせたいだけです。
712ご冗談でしょう?名無しさん:2009/01/14(水) 18:04:23 ID:???
>>709

>>http://www.sci-museum.kita.osaka.jp/~ishizaka/cosmology4.html

>「後退速度」は相対論的宇宙論の誕生(1917年)より70年以上前から
> 慣れ親しんだドップラー効果(1842年)の用語を使って、銀河の赤方
> 偏移量を書き表しただけの便宜的なものである(なにごとも言葉そのも
> のの意味にとらわれてはいけない)。
に同意しているとは思えん。
713ご冗談でしょう?名無しさん:2009/01/14(水) 18:07:59 ID:???
荒れてるとこすまんが質問

単極誘導の原理って解明済みだっけ?
まだわからんことあるなら学士論文のネタの一部分に使おうと思うんだが…
714ご冗談でしょう?名無しさん:2009/01/14(水) 18:22:36 ID:???
>>707さん

ちがいますよ〜(^^;
(私はこのHPの管理者ではありません。)
私は、素人ながらも真剣に

「銀河の後退速度が光速を超える場合がある」
と主張することはナンセンスである

かどうかをハッキリさせたいだけです。
715曲率てんそる:2009/01/14(水) 18:25:39 ID:TGi0s6AP
>>708さん

国立天文台の記事読みました。(^^

>この銀河は光速の96.7パーセントの速
>さで遠ざかっていることに対応し、
>ざっと130億光年もの遠方にあることになります。

「歴史的に」、「便宜上」、宇宙論的赤方偏移とドップラー赤方偏移とを
z>1でも関連づけをするようになったということですかね?
なるほど。
仮に、そういう便宜上の定義を、
宇宙のことを全く勉強していない人に伝える為に使うことは否定しません。
(例え物理的に意味がないとわかっていても)

しかしかなが、以前の私のコメント
>大域的(宇宙半径〜cH^{-1}〜4300Mpc)には
>宇宙は完全に一様等方なので、RW時空で宇宙膨張を記述すべきであり、
>そこでは多様体論より、曲がっている時空でのベクトルの差は定義できない。
はどう思われますか?
もし、仮に私の主張が正しければ、
その便宜上の定義も数学的にはおかしいわけですから、
「厳密には」、その便宜上の定義も理論的にナンセンスとは思いませんか?
716ご冗談でしょう?名無しさん:2009/01/14(水) 18:27:56 ID:???
>>715
>その便宜上の定義も数学的にはおかしいわけですから、
全然おかしくない。余計な妄想は禁物なだけ。
717曲率てんそる:2009/01/14(水) 18:31:41 ID:TGi0s6AP
>>716さん
ですから、その理由は何故ですか??(^^;
教えてください。
718ご冗談でしょう?名無しさん:2009/01/14(水) 18:36:05 ID:???
「様に見える」ってこと。そう解釈しましょう=歴史的=便宜的
それを実態的なものと直に結びつけようとする=余計な妄想
宇宙論をやるならzそのものを使えばいいだけのはなし。
719ご冗談でしょう?名無しさん:2009/01/14(水) 19:08:31 ID:???
ちなみに、現在観測可能な最遠の天体はCMBで、z=1088で、<=観測事実
これは光速度の0.999983倍で遠ざかっていることに対応する。<=見かけ
その間に宇宙は、z+1=1089倍に膨張したんだね。<=宇宙論的解釈
720曲率てんそる:2009/01/14(水) 19:18:06 ID:TGi0s6AP
>>718さん

>宇宙論をやるならzそのものを使えばいいだけのはなし。

近傍の銀河の運動の観測ではいまも重要なわけですから、
宇宙論において、「常に」zだけを使っていればいいって話ではないでしょう。

>それを実態的なものと直に結びつけようとする=余計な妄想

うーん。718さんは、少なくとも、
z>1で銀河の後退速度を考えることは便宜上の意味しかなく、
理論的には全く無意味であるということには同意されているんですよね?
(その理由として、私があげた多様体論からの問題点に同意するかは
別として。)
私は最初に申しましたとおり、「便宜上の定義」を使うことには
同意しています。
ただ、その便宜上の定義が理論的にはナンセンスであろうことを
主張しているだけです。
(便宜上なので、これは当然かもしれません。)
そして、「銀河の後退速度が光速を超える場合がある」に至っては
理論的にはナンセンスであろう主張しているだけです。

みなさんはどう思っているんですか?
よく聞きませんか?銀河の後退速度が光速超えるって意見。
実際、最初は「超えていい」って主張していた人もいたし。

z>1では、便宜上ならともかく、あたかも後退速度が光速を超える
ような銀河の存在を物理的に認めるような意見は間違いだと
思うんですよね。(^^;
間違っていたらすいません。
721ご冗談でしょう?名無しさん:2009/01/14(水) 19:20:36 ID:???
>>720
>理論的には全く無意味であるということには同意されているんですよね?
いんや。
722曲率てんそる:2009/01/14(水) 19:22:26 ID:TGi0s6AP
>>719さん

なるほど!
情報ありがとうございます!

「便宜上」、z>1でも宇宙論的赤方偏移とドップラー赤方偏移とを
関連づけることはある。
とうことですね!(^^
そらは、知らなかったです。ありがとうございました!
723ご冗談でしょう?名無しさん:2009/01/14(水) 19:23:51 ID:TGi0s6AP

>>721
では、意味があるという理論をお聞かせください。
724ご冗談でしょう?名無しさん:2009/01/14(水) 19:25:52 ID:???
>後退速度が光速を超える
>ような銀河の存在を物理的に認めるような意見は間違いだと
>思うんですよね。(^^;
インフレーション宇宙論は間違いだと言うのですね
725ご冗談でしょう?名無しさん:2009/01/14(水) 19:30:12 ID:???
時空多様体上で、光の通った経路にそってうまく座標系をとれば、
その経路にそった細い近傍全体は、ほとんどMinkowski空間と同じ
にできその場合の後退速度はドップラー効果として厳密に解釈できる。
しかし、宇宙の膨張の詳細が分からないと、具体的にそのような
座標系をはることはできないが。宇宙論のモデルが具体的に与えら
れれば、数学的には構成可能。
726曲率てんそる:2009/01/14(水) 19:42:19 ID:TGi0s6AP
>>724

今の議論では、「直接的には」
インフレーションは関係ないでしょう。(^^;

それに、z〜1000程度において、以前の私のコメント
>大域的(宇宙半径〜cH^{-1}〜4300Mpc)には
>宇宙は完全に一様等方なので、RW時空で宇宙膨張を記述すべきであり、
>そこでは多様体論より、曲がっている時空でのベクトルの差は定義できない。
は反証できますか?
(間違いであれば、ホントはバチッと反証して欲しいです。
その為に必死で書き込んでいます(^^;)

もっとも、インフレーション時のドジッター時空も曲がっているんで、
私の上記のコメントと無関係ではありませんが。

私がまず知りたいのは、あくまで
「銀河の後退速度が光速を超える場合がある」
と主張することはナンセンスであるかどうかです。
727ご冗談でしょう?名無しさん:2009/01/14(水) 19:43:41 ID:???
全文イミフ
728曲率てんそる:2009/01/14(水) 19:49:52 ID:TGi0s6AP
>>725

コメントありがとうございます!
なるほど。
BHに落下する粒子の近傍でフェルミ座標系を作るのと
同じ理屈ですかね。
729ご冗談でしょう?名無しさん:2009/01/14(水) 19:52:30 ID:???
>>726
>今の議論では、「直接的には」
>インフレーションは関係ないでしょう。(^^;
もろに関係あります。インフレーション宇宙論では宇宙誕生直後に超光速の膨張をしたと
いうことが肝になってますんで。あなたの論ではインフレーション説は理論的にナンセンスで
間違ってると断じてることになります。
730曲率てんそる:2009/01/14(水) 19:58:41 ID:TGi0s6AP
>>725

>時空多様体上で、光の通った経路にそってうまく座標系をとれば、

その座標系は、光速で遠ざかる銀河を中心にして張られた
座標系ですか?
731ご冗談でしょう?名無しさん:2009/01/14(水) 20:03:43 ID:???
>>730
>中心にして張られた座標系
意味不明。原点は何処でもよいが、光源または観測者を原点に取るのが便利かな。
要は、その座標系では観測された光の経路に沿って、計量がMinkowskiになってる
こと。(測地線にそってそのような計量をとれることは有名な定理)
732曲率てんそる:2009/01/14(水) 20:14:01 ID:TGi0s6AP
>>731

銀河の中心を原点として張られた座標系です。
失礼しました。
素人ですからミスることもあります。スイマセン(^^;
733曲率てんそる:2009/01/14(水) 20:25:14 ID:TGi0s6AP
>>731

>その座標系では観測された光の経路に沿って、計量がMinkowskiになってる
>こと。(測地線にそってそのような計量をとれることは有名な定理)

なるほど。
そのような座標系が局所的には常に取れるので
z>1でも銀河の後退速度(=相対速度)を特殊相対論のドップラー赤方偏移
の式を使って計算できるということですか?
734ご冗談でしょう?名無しさん:2009/01/14(水) 21:14:32 ID:???
遠方銀河の赤方偏移が特殊相対論の?
735ご冗談でしょう?名無しさん:2009/01/14(水) 21:17:07 ID:???
>>726
例えば、RW型の計量 ds^2 = dt^2 - a(t)^2 dx^2 の時空(a(t)>0、単調増加)に
点O(t,0)と点A(t,x)があったとき、a(t) |x| を「OA間の『空間的距離』」と呼び、
その時間微分を「Oを基準にしたAの『後退速度』」と呼ぶ、というのは数学的
にはwell-defined だと思うがどうか。そしてこの用語法はどれほどナンセンス
なのだろうか。

正確には知らないが、「銀河の後退速度が光速を超える場合がある」というときの
「後退速度」は上記と似た定義のものだと思えるのだが。
736ご冗談でしょう?名無しさん:2009/01/14(水) 21:27:21 ID:???
観測可能量じゃないからだろ。
737曲率てんそる:2009/01/14(水) 22:13:45 ID:gUgblrQq
だれか725さんのコメントを読んで
私の733での問いに答えられる方
いらっしゃいませんか?(>_<)

いろんな意見が出てきましたね。
結局
「銀河の後退速度は光速を超える場合がある」
のかどうか、まだ私はハッキリしません...。
738ご冗談でしょう?名無しさん:2009/01/14(水) 22:22:09 ID:???
いい加減余所でやれよおまえ
739曲率てんそる:2009/01/14(水) 22:26:19 ID:gUgblrQq
>>738
そうしまーす(^^)
740ご冗談でしょう?名無しさん:2009/01/14(水) 22:32:10 ID:???
>>737
>>692>>712>>735 は無視か
741ご冗談でしょう?名無しさん:2009/01/14(水) 22:33:29 ID:???
× >>692
>>694
742ご冗談でしょう?名無しさん:2009/01/14(水) 22:36:33 ID:NsLtLEKf
波で回折しやすいのは波長が長いほうですか?短い方ですか?
743ご冗談でしょう?名無しさん:2009/01/14(水) 22:40:52 ID:???
長いほう
744ご冗談でしょう?名無しさん:2009/01/14(水) 22:58:18 ID:???
>>742
ホイヘンスの原理で考えると分かり易い。
745ご冗談でしょう?名無しさん:2009/01/14(水) 23:21:33 ID:???
スリット幅をドンドン狭くしていったとき、回折が起きなくなる限界ってあるのかな。
仮に、原子一列分の隙間があったとして光は回折するのか?
746ご冗談でしょう?名無しさん:2009/01/14(水) 23:34:13 ID:???
近接場光とかは?
747ご冗談でしょう?名無しさん:2009/01/14(水) 23:34:36 ID:???
実験で確かめるのは難しい。
スリット板の厚さも原子スケールにしにゃならんから。
748ご冗談でしょう?名無しさん:2009/01/15(木) 00:03:12 ID:???
X線回折とか
749ご冗談でしょう?名無しさん:2009/01/15(木) 02:49:39 ID:r9NMfgEE
2つの配向がとれる分子(CO分子,固体)がN個あるとき,
系の区別出来る状態の数Wは幾つになりますか?

始め単純にW=2Nと思ったのですが,W=Σw=N!/Σn!
という式があって,混乱してしまいました
750ご冗談でしょう?名無しさん:2009/01/15(木) 07:27:38 ID:Lbf6ZI40
間違っているけど、ヒントになる?
http://jp.youtube.com/watch?v=V3Gswf8DkIU
751ご冗談でしょう?名無しさん:2009/01/15(木) 07:30:58 ID:Lbf6ZI40
トップシークレットだけど、小型のエクセリオンできてるぞ。企業でな。
752ご冗談でしょう?名無しさん:2009/01/15(木) 14:50:44 ID:???
ちゅっどーーーーーーーーーーーーん!
753ご冗談でしょう?名無しさん:2009/01/15(木) 16:35:14 ID:???
シュレディンガーの猫にはメコスジがあるかないか悩んでます
754ご冗談でしょう?名無しさん:2009/01/15(木) 17:31:42 ID:???
物理学にあたるかどうか分からないのですが、天文・気象板で訊くのもなんか違うのでここで質問させてください。
小説かなにかで「限界まで空気を圧縮したら鉄と同じくらい硬くなって銃弾を跳ね返すこともできる」みたいなことが書いてあったのですが、
本当にそんなことが可能なのでしょうか?
要は空気抵抗を物凄く大きくするということなのでしょうが、銃弾って空気抵抗の影響が少ないようにあんな形をしているのですよね?
それを止めることなんて出来るのかな、と疑問に思ったもので。
755ご冗談でしょう?名無しさん:2009/01/15(木) 17:39:17 ID:???
>>748
X線回折はいわゆる回折じゃないと思うんですよね。
干渉、もしくは散乱と呼ぶべきではないかと。なんとなくですが。
といってもまあ、30%ぐらいです。


756ご冗談でしょう?名無しさん:2009/01/15(木) 17:54:50 ID:???
>>754
当然だが原理的には可能。
空気だって所詮分子の集合なんだから集めまくれば硬くなるに決まっておる、その程度の意味かと

ただ実際にできるかどうかはわからんってか仮に実際にやったら硬くなる前に液化しちゃう気がして仕方ないがw
757ご冗談でしょう?名無しさん:2009/01/15(木) 18:03:51 ID:???
大丈夫だ。室温なら臨界点は越えている。
758ご冗談でしょう?名無しさん:2009/01/15(木) 18:07:33 ID:???
>>754
単に、ガスの圧力の上限は電子縮退圧(10^10Paオーダー)まで上げられるので、
鉄の圧縮強度(10^8Paオーダー)よりは上じゃねって事いってんでは。
あまり現実的(実用的)な話じゃないと思うが。
759ご冗談でしょう?名無しさん:2009/01/15(木) 18:16:49 ID:???
ありがとうございます。
もっとも「空気を自由に操る」という超能力を持った人の話だったので、その時点で物理なんてとっくに無視しちゃってるわけですが。
仮にそんな能力があったとしたら本当に可能なのかな、と。
でも考えてみたら空気を操れる=音や天候を自由に操れる なんですよね。
凄すぎる……
760ご冗談でしょう?名無しさん:2009/01/15(木) 18:33:14 ID:CPLCPnFH
単位長さあたりnの巻き数の半径がaで長さlのソレノイドに
定常電流Iをながすとき
ソレノイドに蓄えられた磁場のエネルギーは?

というもんだいで

IはI(t)とおいたらまずいでしょうか?
ここから誘導起電力をだして
エネルギーを求めようという魂胆なのですが、
I(t)でないと起電力がなくなってしまいます
761ご冗談でしょう?名無しさん:2009/01/15(木) 18:37:48 ID:???
やってみてから、再び報告よろ。
762ご冗談でしょう?名無しさん:2009/01/15(木) 19:02:38 ID:CPLCPnFH
>>761
やってみたらうまくいきます
やはりtの関数なんですかね
763ご冗談でしょう?名無しさん:2009/01/15(木) 19:29:42 ID:5nksQ5ii
過去スレ ブラックホールでは、物質は崩壊するよ。それで、考えてみてね。
764ご冗談でしょう?名無しさん:2009/01/15(木) 19:47:02 ID:???
素粒子は固体?固体じゃない?
765ご冗談でしょう?名無しさん:2009/01/15(木) 20:03:56 ID:???
気体(電子)の場合と、液体(陽子・中性子)の場合がある。
766ご冗談でしょう?名無しさん:2009/01/15(木) 20:05:46 ID:???
陽子と中性子は素粒子じゃないけどな
767ご冗談でしょう?名無しさん:2009/01/15(木) 20:10:47 ID:???
よく理解できませんがとにかくありがとうございました。
素粒子が固体とは限らないと分かっただけでも収穫でした
768ご冗談でしょう?名無しさん:2009/01/15(木) 20:22:07 ID:???
素粒子の群なら兎も角、素粒子に三態とか関係ないけどな
769ご冗談でしょう?名無しさん:2009/01/15(木) 20:26:28 ID:???
固体とか液体とかはある程度まとまった数の集団の状態をあらわすものであって
素粒子単体で固体だ液体だというのはナンセンス

陽子・中性子の集団である原子核が液体か気体かという議論は可能だけど
陽子・中性子だけ取りあげて液体だ気体だというのもナンセンス。
770ご冗談でしょう?名無しさん:2009/01/15(木) 20:36:08 ID:???
>>768-769
そこがさー、物性論と素粒子論の仲が悪いところじゃないの?
ここのスレは両方の多少はレベルの高そうな人もいるんみたいだから、
ROMのお勉強ために、突っ込んだ議論をやってもらえないかしらん?
771ご冗談でしょう?名無しさん:2009/01/15(木) 20:43:16 ID:???
原子の内部は巨大な空洞。そこにはエネルギーと情報が波のように揺らいでいる、らしい。
772ご冗談でしょう?名無しさん:2009/01/15(木) 20:45:04 ID:???
空洞っていうか、真空だろ。
773ご冗談でしょう?名無しさん:2009/01/15(木) 20:49:22 ID:???
学部三回生が研究室を選ぶのに相談したいっていうようなスレ建てたらこわいおにいさんたちにおこられますか?
774ご冗談でしょう?名無しさん:2009/01/15(木) 20:55:16 ID:???
素粒子→幽子→霊子→素子→神子

素粒子以下からは次元がちがうのでその物質の解明は大変困難。
775ご冗談でしょう?名無しさん:2009/01/15(木) 20:59:52 ID:???
では私も2ch的に(笑)

素粒子論はヒモ暮らし
776ご冗談でしょう?名無しさん:2009/01/15(木) 21:03:54 ID:???
>>755
思われても困る
777ご冗談でしょう?名無しさん:2009/01/15(木) 21:05:05 ID:???
>>764
おまいのせいで、スレが壊れちまったじゃねぇ蚊。w
778ご冗談でしょう?名無しさん:2009/01/15(木) 21:25:54 ID:nPMJ6tKb
水槽の下にホースを付けて
そのホースを上に伸ばした状態で水を抜いたら
水はホースの口まで上がって来ますか?
779ご冗談でしょう?名無しさん:2009/01/15(木) 21:30:07 ID:???
>>770
>そこがさー、物性論と素粒子論の仲が悪いところじゃないの?
意味不明
何か言いたいからって無理やり作らないように
780ご冗談でしょう?名無しさん:2009/01/15(木) 21:30:55 ID:???
ホースの口の高さやら太さ、圧力によるだろ
781ご冗談でしょう?名無しさん:2009/01/15(木) 21:40:30 ID:???
口を細くすれば水圧が口に集中するから非常に高く上がるよ。
火山の噴水噴火みたいにね。
782ご冗談でしょう?名無しさん:2009/01/15(木) 21:45:02 ID:???
>>778
ホースの口を押さえていれば10mちょっとまで水は上がる。
開放されていれば水槽の水面の高さまで。
中学理科の問題。
783ご冗談でしょう?名無しさん:2009/01/15(木) 21:52:16 ID:???
>>779
はーい。(笑)
784ご冗談でしょう?名無しさん:2009/01/15(木) 23:17:05 ID:5nksQ5ii
今年は、スペースシップいっぱい飛ぶよ
785ご冗談でしょう?名無しさん:2009/01/16(金) 00:11:48 ID:dYc/oqsK
質問です。
ファンデルワールスの方程式(p+N^2a/V^2)(V-Nb)=NkTを考える。
これに臨界点近傍で無次元化された微小変数φ=V/Vc-1を導入したとき、上式を次のように展開せよ
h=A(T)φ+Bφ+O(φ)

という問題なのですが、φ二乗の項が落ちなくて困っています。
自分がやったとき方としては、ファンデルワールスの式をp/pc,V/Vc,T/Tcで無次元化して式を簡単にしてからφを代入し、臨界点でp/pcをT/Tcとφの2変数についてのテイラー展開をやってみましたがうまくいきませんでした。

この問題が解ける方、できればアドバイスをいただけないでしょうか?
786785:2009/01/16(金) 00:30:28 ID:???
すみません、式書き間違えました

h=A(T)φ+Bφ^3+O(φ^4)です
787ご冗談でしょう?名無しさん:2009/01/16(金) 00:32:15 ID:???
>>782
>ホースの口を押さえていれば10mちょっとまで水は上がる。
うんこのおつり思い出した。ww
788ご冗談でしょう?名無しさん:2009/01/16(金) 00:48:34 ID:???
>>781-782 まじで言っているのか?
789ご冗談でしょう?名無しさん:2009/01/16(金) 00:50:51 ID:???
>>788
782は何か間違ったこと言ってるか?
790ご冗談でしょう?名無しさん:2009/01/16(金) 01:01:43 ID:dlzCyi99
ナデシコ型エクセリオンだろ
791ご冗談でしょう?名無しさん:2009/01/16(金) 01:10:54 ID:???
>>789
どんなに口を押さえても、連続した水の流れでは水槽の水面以上に上がらないよ。
それ以上に上げ様と思えば、水激ポンプみたいな仕組みが必要。
792ご冗談でしょう?名無しさん:2009/01/16(金) 01:18:52 ID:???
海に波があるのは何の影響ですか?
793ご冗談でしょう?名無しさん:2009/01/16(金) 01:25:43 ID:???
>>791
トリチェリの真空って中学理科じゃないのかな?
794ご冗談でしょう?名無しさん:2009/01/16(金) 01:33:12 ID:???
>>781は明らかに間違いだけど、>>782は当たり前の事書いてるだけだろ。
795773:2009/01/16(金) 01:39:48 ID:???
俺涙
796ご冗談でしょう?名無しさん:2009/01/16(金) 01:49:22 ID:???
>>791
? 気圧はご存じ?
797ご冗談でしょう?名無しさん:2009/01/16(金) 02:00:19 ID:dlzCyi99
バナナとか、果物を加工して、チューブんび入れて、食べる商品
798ご冗談でしょう?名無しさん:2009/01/16(金) 02:00:43 ID:???

もし隣り合っていて同じ時速の電車同士なら、お互いに止まって見えるだろう。

では・・・
もし飛んでゆく光の矢を、光速で追いかけたら、光の矢はどう見えるだろうか?
799ご冗談でしょう?名無しさん:2009/01/16(金) 02:00:50 ID:dlzCyi99
バナナとか、果物を加工して、チューブに入れて、食べる商品
800ご冗談でしょう?名無しさん:2009/01/16(金) 02:01:24 ID:???

もし隣り合っていて同じ時速の電車同士なら、お互いに止まって見えるだろう。

では・・・
もし飛んでゆく光の矢を、光速で追いかけたら、光の矢はどう見えるだろうか?
801ご冗談でしょう?名無しさん:2009/01/16(金) 02:07:54 ID:ToCbRryM
>>800
特殊相対性理論(光速度不変の原理)
802ご冗談でしょう?名無しさん:2009/01/16(金) 02:08:11 ID:zhPbiTAi
ヘリウムガスを吸った人の声は変わるのに、楽器の音が変わらないのは
どうしてですか?
教えてください
803ご冗談でしょう?名無しさん:2009/01/16(金) 02:16:48 ID:???
>>802
楽器でも、気柱の共鳴を利用してる物は音が変わるんじゃないかな。
804ご冗談でしょう?名無しさん:2009/01/16(金) 02:18:40 ID:???
>>801
で、どう見えるんですか?(という質問なんです)
805ご冗談でしょう?名無しさん:2009/01/16(金) 02:19:46 ID:zhPbiTAi
ありがとうございます。楽器がピアノだとしたら伝わりませんか?
高校の知識しかないので、ググってもわかりませんでした
806ご冗談でしょう?名無しさん:2009/01/16(金) 02:20:06 ID:???
>>802
楽器はガスを吸えないからだよ。
807ご冗談でしょう?名無しさん:2009/01/16(金) 02:34:15 ID:???
>>804
>もし飛んでゆく光の矢を、光速で追いかけたら
実現不可能なことを前提とした質問なので答えようがない
まあ、Einsteinは学生時代そういう疑問を持ってたらしく、
それが光速度不変の原理の発見に繋がったと言われているが

>>805
Heを吸って声を出す(あるいはHeの中で管楽器を演奏する)のは、
弦楽器でいえば弦の材質を変えることに相当する
808ご冗談でしょう?名無しさん:2009/01/16(金) 02:34:56 ID:???
ピアノはピアノ線の張力とかで振動数が決まるから音は変わらないのかな、と想像。
809ご冗談でしょう?名無しさん:2009/01/16(金) 02:45:34 ID:???
>>807
>>実現不可能なことを前提とした質問なので答えようがない

もし月がなかったら満ち潮と引き潮は起きると思う?
810ご冗談でしょう?名無しさん:2009/01/16(金) 02:57:52 ID:???
>>809
月を無くすことは実現可能(無論、現実的には無理だが)なので、
簡単に答えられる。>>800とは違う。
811ご冗談でしょう?名無しさん:2009/01/16(金) 06:13:14 ID:N4xKG5pG
宇宙の最初は、水素やヘリウムばかりで、
他の原子は、巨大恒星の核融合の繰り返しで作られ、
最後に爆発して宇宙に飛び散るそうですが、

疑問があります。巨大恒星の中心は、
爆発時に中性子星(またはブラックホール)として残るそうですが、
中心にできる鉄原子は、中性子星とかに取り込まれ、宇宙に飛び散れないのでは?

そうすると、地球とかにも鉄が存在しないことになりますね?
812ご冗談でしょう?名無しさん:2009/01/16(金) 06:50:32 ID:???
てすと
>>225->>228
813ご冗談でしょう?名無しさん:2009/01/16(金) 06:51:21 ID:???
>>794
ホースの口を押さえるって意味を勘違いしてた、>>782さんにはすまなかった。
814ご冗談でしょう?名無しさん:2009/01/16(金) 07:23:00 ID:???
>>811
全部が全部、中性子星orBHに残らないことなんてガキにも分かりそうなもんだが。
815ご冗談でしょう?名無しさん:2009/01/16(金) 07:50:42 ID:jchOFuxR
FIRフィルタをフーリエ級数法で設計するという問題がありまして
そのことについて質問したい事があるのですが
信号処理の質問はここの板で宜しいのですか?
816ご冗談でしょう?名無しさん:2009/01/16(金) 07:51:09 ID:???
良くない
817ご冗談でしょう?名無しさん:2009/01/16(金) 08:04:46 ID:???
了解
818ご冗談でしょう?名無しさん:2009/01/16(金) 08:25:03 ID:dlzCyi99
819ご冗談でしょう?名無しさん:2009/01/16(金) 11:13:40 ID:41GInX5H
相対論的に考えて、光に乗る事が出来たならば
その系では時間が流れないって事で正しいですか?
820ご冗談でしょう?名無しさん:2009/01/16(金) 11:15:17 ID:???
相対論的に考えて、光に乗る事はできない。
821ご冗談でしょう?名無しさん:2009/01/16(金) 11:23:33 ID:???
光の99.99パーセントの速度で移動出来たら
その系では時間はほとんど流れないと思うのか?
822ご冗談でしょう?名無しさん:2009/01/16(金) 11:35:51 ID:41GInX5H
>>821
それで違いないですよね?
823ご冗談でしょう?名無しさん:2009/01/16(金) 11:40:20 ID:???
反語って知ってる?
824ご冗談でしょう?名無しさん:2009/01/16(金) 11:40:51 ID:???
光の99.99パーセントの速度のロケットの中でも時間は普通に流れるぞ。
825ご冗談でしょう?名無しさん:2009/01/16(金) 11:43:09 ID:41GInX5H
内部の人間の固有時間は地球にいる人間と比べ、微小なものになるのではないのですか?
826ご冗談でしょう?名無しさん:2009/01/16(金) 11:43:20 ID:???
y = (v0 * t - 0.500 * g * t * t ) * 480.000 / y_max

落下運動に関するもので↑のような式があったんですけど
この式はyを求めてるんですが何でこの式でyが求まるのかがわかりません
ちなみに落下して跳ね返ったあとの物体の変位です
()の中は重力の影響を表してるんだと思いますがそのうしろの480.000 / y_maxがなんなのかさっぱりです
480 っていうのは地面から天井までの高さだと思ってください
827ご冗談でしょう?名無しさん:2009/01/16(金) 11:49:07 ID:???
>>825
固有時ってそもそも何だと思ってる?
828ご冗談でしょう?名無しさん:2009/01/16(金) 11:51:41 ID:???
たぶんそれは

地球にいる人間から見れば

が抜けてるだけだろ
それを指摘せずに無駄にレス数のばしてるアホはなんなんだ
829ご冗談でしょう?名無しさん:2009/01/16(金) 11:53:50 ID:41GInX5H
>>827
各々の系で、静止している時計の時間ですよね?
830811:2009/01/16(金) 15:59:04 ID:N4xKG5pG
>>814
つまり中性子星orBHとして残るのは、ごく僅かな部分だけですね?
中心部にある鉄原子も、大半は宇宙に飛び散るのですね?
831ご冗談でしょう?名無しさん:2009/01/16(金) 17:23:12 ID:???
自分の思ったとおりの言い回しじゃないからといって
しつこく同じことを聞くのはうざい
832ご冗談でしょう?名無しさん:2009/01/16(金) 17:25:38 ID:p6hREtKw
V=V1
833ご冗談でしょう?名無しさん:2009/01/16(金) 18:02:21 ID:???
>>830
いやどっちかつうと、観測事実(鉄の存在比率等)に合わせて理論を構築してるわけで、
最近までシミュレーションでは超新星爆発を起こせず(外周部を吹き飛ばすはずの衝撃波を飲み込み)そのまま重力崩壊しちまってたのよ。
だから核近くの物質の輸送機構なんかはまだまだ確定した話ではないのよ。
834ご冗談でしょう?名無しさん:2009/01/16(金) 19:47:46 ID:???
速さに最小の速さ(これ以上速さを遅くしようと思っても0にしかならない速さ)というのはあるのでしょうか?
835ご冗談でしょう?名無しさん:2009/01/16(金) 19:58:26 ID:???
あるよ。
836ご冗談でしょう?名無しさん:2009/01/16(金) 20:03:29 ID:???
>>833
そっち方面詳しくないけど、重力崩壊のシミュレーションなんてパラメーターがたくさん必要な段階なんだろうなぁ
837ご冗談でしょう?名無しさん:2009/01/16(金) 20:10:09 ID:???
x方向に速度が虚数ってことは、y方向に進むってことですか?
838ご冗談でしょう?名無しさん:2009/01/16(金) 20:11:55 ID:???
z方向かもしれない。
839ご冗談でしょう?名無しさん:2009/01/16(金) 22:39:23 ID:8mcgvB6+
唐突な質問をお許し下さい。

んじゃー逆に、光ってその速度は超えないの?
具体的に秒速約30万キロメートル(c)とすると、cを超える速度で移動する事はないの?
840ご冗談でしょう?名無しさん:2009/01/16(金) 22:42:15 ID:???
何の逆だよ
841ご冗談でしょう?名無しさん:2009/01/16(金) 22:42:18 ID:???
唐突な質問をお許し下さい。

どうして>>2を読まないの?
842ご冗談でしょう?名無しさん:2009/01/16(金) 22:42:43 ID:???
相対論相対論言ってるやつは一回ちゃんとした本を一回読め
843ご冗談でしょう?名無しさん:2009/01/16(金) 22:48:59 ID:???
読んでも理解できないから
844ご冗談でしょう?名無しさん:2009/01/16(金) 23:38:32 ID:???
変分と全微分って何が違うんでしょうか?
845ご冗談でしょう?名無しさん:2009/01/16(金) 23:58:18 ID:???
作用する相手が汎関数と関数の違い?
846ご冗談でしょう?名無しさん:2009/01/17(土) 00:42:29 ID:???
汎関数と合成関数って何が違うんでしょうか?
847ご冗談でしょう?名無しさん:2009/01/17(土) 00:47:29 ID:???
逆になぜ同じに思えるのか、君の考えを書いてみたら?
848ご冗談でしょう?名無しさん:2009/01/17(土) 00:52:06 ID:???
合成関数ってのはそれ自体はただの関数だよ。
849ご冗談でしょう?名無しさん:2009/01/17(土) 00:55:37 ID:???
>>839
まずね、「光の速さ」っていうのは物体が移動する時の「物が動く速度」とは違うんだよ。
物じゃないから、移動するわけじゃないんだよ。
そういう初歩的なところから理解してくれ。
850ご冗談でしょう?名無しさん:2009/01/17(土) 01:08:52 ID:???
>>807
なんで実現不可能なの?
月は消せるんだろ?
851ご冗談でしょう?名無しさん:2009/01/17(土) 01:13:10 ID:???
>>850
質量0では時間経過しないから観測不可、が答え。
852ご冗談でしょう?名無しさん:2009/01/17(土) 01:22:28 ID:???
>>804
見えない
853ご冗談でしょう?名無しさん:2009/01/17(土) 01:48:16 ID:???
>>850
質量のある物体が光速になるためには、無限のエネルギーが必要だから。
854ご冗談でしょう?名無しさん:2009/01/17(土) 02:18:00 ID:???
マクスウェル方程式の4式について質問
それぞれの式の基礎となった実験的事実を簡単に述べよ、という問題が出されたのですがいい資料がうまく見つかりませんでした
知ってる方教えてください
参考になるURLとか教えてもらえるとうれしいです。お願いします。
↓マクスウェル方程式
ttp://ja.wikipedia.org/wiki/%E3%83%9E%E3%82%AF%E3%82%B9%E3%82%A6%E3%82%A7%E3%83%AB%E3%81%AE%E6%96%B9%E7%A8%8B%E5%BC%8F
855ご冗談でしょう?名無しさん:2009/01/17(土) 02:36:38 ID:nK7AG7UN
申し訳ありません初歩的な質問です。
ディラック量子力学の本のどのあたりに、経路積分の元に
なる記述が書いてありますか?
あとブラベクトルってヒルベルト空間のベクトルと
思うのですが、どうして実部と虚部に分ける事が
出来ないのですか?
すみませんがどなたか教えて下さいますよう宜しくお願いします。
856ご冗談でしょう?名無しさん:2009/01/17(土) 03:46:48 ID:???
>>851
「もし生まれ変わるとしたら何になりたい?」とか聞いても
「生まれ変るということはあり得ないから答えられない」とか言う奴いるよなぁ。

だいたいそういう奴っていじめられて育つんだよな。
857ご冗談でしょう?名無しさん:2009/01/17(土) 03:49:42 ID:???
>>856
物理板でお前何言ってんの? いじめられたトラウマでも蘇ったのかい?w
858ご冗談でしょう?名無しさん:2009/01/17(土) 03:50:07 ID:???
>>856
>>851は答えられないから逃げてるだけだろ。
どう考えても。
859ご冗談でしょう?名無しさん:2009/01/17(土) 03:53:28 ID:???
>>851
じゃぁ光は見えないの?
860ご冗談でしょう?名無しさん:2009/01/17(土) 03:54:56 ID:???
>>858
>>858は答えられないから煽ってるだけだろ。
どう考えても。
861ご冗談でしょう?名無しさん:2009/01/17(土) 08:00:26 ID:???
>>855
Diracは持ってないからわからん
ベクトルの実部虚部ってのは聞いたこと無い

例えば複素数zから実部と虚部をとりだす操作は
re(z) = z+z^*/2, im(z) = z-z^*/2i
と書けるけど、これをHilbert空間Hのベクトルv in Hに対してやろうと思うと
に対するv^* in Hみたいなのを定義する必要があると思う
この*をとる操作はブラをケットみたいな双対空間の元に移すのとは全く別物な

H上の作用素ならadjoint使ってT+T^*とかT-T^*みたいにして組みなおすことはできると思うが
862ご冗談でしょう?名無しさん:2009/01/17(土) 08:01:52 ID:???
>>861訂正
>に対するv^* in Hみたいなのを定義する必要があると思う
vに対するv^* in Hみたいなのを定義する必要があると思う
863ご冗談でしょう?名無しさん:2009/01/17(土) 08:24:45 ID:???
>>855
気になったから検索した
これのp3の最初の言葉で悩んでる?
http://hb3.seikyou.ne.jp/home/E-Yama/Dirac_1.PDF

>>861で書いたが、|a>に対して|a>^*が定義されてないからだと思われる
|a>に<a|を一意的に対応させる意味での共役は複素共役とは全く別物
本か↓のページみたいなところでRieszの表現定理というのを見れば感覚つかめるはず
http://www.math.meiji.ac.jp/~mk/labo/text/functional-analysis-3/node43.html
864ご冗談でしょう?名無しさん:2009/01/17(土) 11:44:55 ID:F5wu6k4r
>>479
遅レスですが、意味不明です。

 そもそも、反粒子の存在は何某(名前忘れた)が予想して、それが
実験で確かめられたため存在が確認されたという類の物のはず。

 対消滅でγ線が発生したからと言って、γ線が発生した=対消滅
とは為らないのではなかろうか。

 同じ理屈で、ブラックホールの予言もありましたが、自然観測で
実証されていますし。
865ご冗談でしょう?名無しさん:2009/01/17(土) 11:57:07 ID:???
導体の静電気学の問題なんだが、接地した導体と点電荷から成る系で、
導体の接地を切って、点電荷消してもキャパシタンスが不変っていうのは何故?
ラグランジアンにこの操作しても時間微分出ないし・・

それとも、本質的な操作は、ポテンシャルの基準変更で、
点電荷消えるのは、接地切ったところで不安定になるから?
866ご冗談でしょう?名無しさん:2009/01/17(土) 11:57:46 ID:q83YQbAe

だれか>>854も知りたい答えてやってよ。
867ご冗談でしょう?名無しさん:2009/01/17(土) 11:59:26 ID:???
>>854
ブルーバックスのマクスウェル方程式とか?
868ご冗談でしょう?名無しさん:2009/01/17(土) 12:03:27 ID:???
>>854
4本の式に対して一つもわからないというのであれば、単なる怠慢。
869854:2009/01/17(土) 12:28:27 ID:???
divD=ρ、rotE+dB/dt=0はそれぞれクーロンの法則、ファラデーの法則をより一般化した形であるというのはわかりましした。
クーロンの法則とファラデーの法則の実験事実をある程度書けたのですが、残りの2式の情報が足りません。
divB=0とrotH-dD/dt=iの基礎となった実験事実を簡単にお教えください。
870ご冗談でしょう?名無しさん:2009/01/17(土) 12:29:42 ID:Nz+KeHoB
マクスウェル方程式
でググっても、たぶん実験に関する情報は出てこないぞ。
それらは数学的に洗練された形の式だから。
たとえば、∇・D=ρなら、ガウスの法則、またはクーロンの法則でググるといい。
他のやつも、ファラデーの法則とか、名前ついてるから。
871ご冗談でしょう?名無しさん:2009/01/17(土) 12:36:30 ID:???
>>869
あとの二つを直接表すような実験はない。
エルステッドやアンペールの実験に基づいて頭のいい人たちが色々考えてその二式を導いた。
872ご冗談でしょう?名無しさん:2009/01/17(土) 14:25:05 ID:???
>>864
そりゃγ線捕まえただけじゃ証拠にはならんけど、そのエネルギーが
ちょうど対消滅した粒子の静止質量エネルギーに等しいとか、
出てきたγ線の運動量が全体でちょうど0になってるとか、
そういった情報を総合した結果、対消滅したと考えるのが
最も妥当な解釈という結論になる
873ご冗談でしょう?名無しさん:2009/01/17(土) 14:25:18 ID:???
古典力学でハミルトニアンってどういうときに使うと便利なんですか?
いまいち古典力学でのハミルトニアンの有効性がわからない
874ご冗談でしょう?名無しさん:2009/01/17(土) 14:26:10 ID:vkjnDBQN BE:690501964-2BP(0)
http://uproda11.2ch-library.com/src/11150198.bmp
このグラフの近似式を教えてください
エクセルでA1〜A141に0〜70まで0.5置きに数字を入れて
B1のセルに=EXP(-33/1+(15*COS(0.127/0.49*(A1+5))+1)/2)*1000000000000-39
を打ち込んで、B141までドラッグすると底が平らな近似グラフができるんですが、
これを上に貼ったグラフのようにおわん型にする方法はないですか?
教えてください。お願いします
875ご冗談でしょう?名無しさん:2009/01/17(土) 14:31:50 ID:F5wu6k4r
>>872
 だから、自然観測で対消滅が起こったという事実は観測されたか?
と言う単純な疑問ですよ。

 ビッグバンで、粒子と反粒子が生成されたと言う事実が確認されて
いない限り、反粒子は生成されていないと言う反論も可能ではなかろ
うか。
876ご冗談でしょう?名無しさん:2009/01/17(土) 15:05:06 ID:???
反粒子観測されてるだろ。
霧箱実験読んでろ、カスが。
877ご冗談でしょう?名無しさん:2009/01/17(土) 15:12:12 ID:???
最近のことですか?
878ご冗談でしょう?名無しさん:2009/01/17(土) 15:20:09 ID:???
>>873
実際の系の具体的計算はLagrangianで十分だと思う
ただ、Lagrangianの一般化座標よりHamiltonianの正準変数のほうが自由に選べるし、
何といっても非相対論であればHamilton力学のほうが数学的にきれいな形してるから、
古典力学の方程式系の性質知りたければHamilton力学を使うべき

>>875
つ Andersonのノーベル賞受賞
879ご冗談でしょう?名無しさん:2009/01/17(土) 15:26:52 ID:???
>>878
>Anderson
これ? なんかよう分からん。
http://nobelprize.org/nobel_prizes/physics/laureates/1977/anderson-lecture.pdf
880ご冗談でしょう?名無しさん:2009/01/17(土) 15:28:45 ID:OiJu7ddX
ホールは反粒子とちゃいます。 w
それとも、Dirac流の解釈もあり?
881ご冗談でしょう?名無しさん:2009/01/17(土) 15:34:38 ID:???
ハイゼンベルクあたり?がノーベル賞講演で
ディラック理論とシュテルン・ゲルラッハの実験が一致しないとか言ってるけど
完成版QED使えば合うの?
882ご冗談でしょう?名無しさん:2009/01/17(土) 16:06:45 ID:???
>>881
> Stem’s experiments seem, furthermore, to indicate that the behaviour
> of the heavy elementary particles cannot be represented by the formalism
> of Dirac’s theory of the electron.

heavy elementary particles =陽子じゃろ。g因子が合わないって問題ね。
883ご冗談でしょう?名無しさん:2009/01/17(土) 17:21:29 ID:???
>>874
近似曲線の追加→多項式近似
で次数を適当に高いものに選べばいい。
884ご冗談でしょう?名無しさん:2009/01/17(土) 18:07:42 ID:???
>>874
x^2を並べとけよ
885ご冗談でしょう?名無しさん:2009/01/17(土) 19:23:08 ID:M9NwU6F4 BE:517877036-2BP(0)
>>884
よく意味が解らないです
詳説願います
886ご冗談でしょう?名無しさん:2009/01/17(土) 19:47:13 ID:???
特殊相対論では線素dsの自乗が不変ですが、一般相対論でも同じく不変ですか?
887ご冗談でしょう?名無しさん:2009/01/17(土) 19:58:47 ID:???
普遍とは何ぞや?
888ご冗談でしょう?名無しさん:2009/01/17(土) 20:17:47 ID:???
>>886
スカラーですか?ってことでしょ。
うん。そうだよ。
ただし、一般相対論では、ds^2=ηdxdxではなく、ds^2=gdxdxであることに注意。
889ご冗談でしょう?名無しさん:2009/01/17(土) 20:23:32 ID:???
4次元測度d4xって常に時間要素と空間要素の積dVdtに分離できるの?
890ご冗談でしょう?名無しさん:2009/01/17(土) 20:30:38 ID:???
>>888
スカラーとは何ぞや?
考えている変換によって変わるのだぞ。
891ご冗談でしょう?名無しさん:2009/01/17(土) 20:44:36 ID:???
>>890
gdxdx = g'dx'dx'って意味で不変ってことじゃない?
892ご冗談でしょう?名無しさん:2009/01/17(土) 20:54:06 ID:???
>>858
851はめちゃくちゃ正論だろ

どのみち光速になったらローレンツ短縮で進行方向の距離はゼロになるんだから
光速になった瞬間に光速でなくなる地点まで到達し、光速でなくなる。
893ご冗談でしょう?名無しさん:2009/01/17(土) 20:55:37 ID:???
>>864
意味不明なのはお前じゃねえか…
894ご冗談でしょう?名無しさん:2009/01/17(土) 20:58:08 ID:???
だれか856のために、夢の有る答えを出してやってください
生まれ変わりでもなんでもアリです
895ご冗談でしょう?名無しさん:2009/01/17(土) 21:04:58 ID:???
>>891
>gdxdx = g'dx'dx'って意味で不変ってことじゃない?
どんな種類の変換を考えてるのだ? それを指定しなけりゃ、無意味だ。
896ご冗談でしょう?名無しさん:2009/01/17(土) 21:13:46 ID:pWAjTOUa
重力は固定できますか?
897ご冗談でしょう?名無しさん:2009/01/17(土) 21:31:15 ID:W49kIuLP
光の速度とは常に一定だと言うのは本当だと思いますか?


以下ヒント。
特殊相対性理論、ブラックホール、光速度不変、ローレンツ変換、タキオン、宇宙空間、エーテル。
898ご冗談でしょう?名無しさん:2009/01/17(土) 21:39:52 ID:???
>>895
擬リーマン多様体上の座標変換
899ご冗談でしょう?名無しさん:2009/01/17(土) 21:40:52 ID:???
>>896
日本語の勉強を。
900ご冗談でしょう?名無しさん:2009/01/17(土) 22:01:44 ID:???
特殊相対論の世界ではds^2がどの慣性系からも一定不変であるということは
すべての物体は4次元時空の中を常に光速で運動し続けているという解釈が
できますが、一般相対論まで拡張するとds^2が不変であるということはどのよう
な意味に解釈されるのでしょうか。
901ご冗談でしょう?名無しさん:2009/01/17(土) 22:03:13 ID:???
> すべての物体は4次元時空の中を常に光速で運動し続けているという解釈ができますが

できねぇ
902ご冗談でしょう?名無しさん:2009/01/17(土) 22:11:41 ID:???
どうしてても量子力学が正しいとは信じられないんですが、
どうしたらいいですか。
903ご冗談でしょう?名無しさん:2009/01/17(土) 22:12:29 ID:???
科学理論は信じるものじゃないので無問題。
904ご冗談でしょう?名無しさん:2009/01/17(土) 22:14:50 ID:???
まあ、科学者でも飛行機なんて鉄の塊が空飛べるなんて「信じない」人もいるだろう
905ご冗談でしょう?名無しさん:2009/01/17(土) 22:15:22 ID:???
>>902
君が信じるか信じないかはどうでも良い。
906ご冗談でしょう?名無しさん:2009/01/17(土) 22:22:45 ID:???
既成概念で全て解決するなら物理はいらない
907ご冗談でしょう?名無しさん:2009/01/17(土) 22:24:05 ID:???
量子力学が既に既成概念になってしまっていて、
その枠から逃れられないだけだとは思わないのだろうか?
908ご冗談でしょう?名無しさん:2009/01/17(土) 22:25:27 ID:???
>>907
科学ってそもそも何か分かってる?
909ご冗談でしょう?名無しさん:2009/01/17(土) 22:33:44 ID:???
>>907
哲学板にでも行け
910ご冗談でしょう?名無しさん:2009/01/17(土) 22:35:07 ID:???
>>901
ds^2の式の両辺をdt^2(dτ^2じゃない)で割って整理してみてごらん。
慣性系から見たらすべてのものは4次元時空を光速で移動していると
解釈できるから。

慣性系から見たら運動している物体は時空を常に一定の速度で進んで
るから、空間成分の速度が速ければ速いほど時間成分の速度がおそく
見える。
つまり、その物体の固有時間が遅く見える。
911ご冗談でしょう?名無しさん:2009/01/17(土) 22:36:46 ID:???
だめだこりゃ(>>910)
912ご冗談でしょう?名無しさん:2009/01/17(土) 22:41:17 ID:???
相対論の専門書にはそういう解説を書いてあるものもあるんだけど…。

見たことない?(>>911
913ご冗談でしょう?名無しさん:2009/01/17(土) 22:41:32 ID:???
>>910
-ds^2 = (dx/dt)^2 + (dy/dt)^2 + (dz/dt)^2 - c^2
ds^2≠0 なら速さ≠c にしか見えん。
914ご冗談でしょう?名無しさん:2009/01/17(土) 22:49:02 ID:???
>>913
ちがうちがう。「式を整理して」って言ったでしょ。
(dτ/dt)^2 + (dx/dt)^2 + (dy/dt)^2 + (dz/dt)^2 = c^2
だよ。

たとえば2次元空間(x-y座標系)を一定の速さVで進んでる物体が
あったとき、その速度のx成分Vxが大きいほどその物体の速度の
y成分Vyは小さいでしょ。
三平方の定理からV^2は常に一定だから。

特殊相対論では、物体は上式より4次元時空を常に一定速度cで進んでる
から空間の速度が速い物体は時間の進みが遅い、と解釈できるんだよ。
まあ、「運動する物体の時間は遅れる」という事実を線素の式で言い換えた
だけなんだけどね。
915ご冗談でしょう?名無しさん:2009/01/17(土) 22:51:43 ID:???
>>913じゃないけど
>4次元時空を
ってところを見逃してた
916ご冗談でしょう?名無しさん:2009/01/17(土) 22:51:58 ID:???
ごめん
(c*dτ/dt)^2 + (dx/dt)^2 + (dy/dt)^2 + (dz/dt)^2 = c^2
だ。
917ご冗談でしょう?名無しさん:2009/01/17(土) 22:53:50 ID:???
>>914
>(dτ/dt)^2 + (dx/dt)^2 + (dy/dt)^2 + (dz/dt)^2 = c^2
左辺にτが入ってきていますが、
τというのは固有時間(つまり、運動している物体に固定した座標系での時間)なので
この座標系では、dx、dy、dzはいずれも0になると思います。
よって、上の式は(dτ/dt)^2 = c^2 という式の間違いだと思うのですが、いかがでしょう?
918ご冗談でしょう?名無しさん:2009/01/17(土) 22:56:51 ID:???
>dx、dy、dzはいずれも0になる
それは物体が空間的に静止してる場合だな。
その場合はdτ = dtだから時間は同じ速さで進んでいる。
919ご冗談でしょう?名無しさん:2009/01/17(土) 23:00:22 ID:???
> ちがうちがう。「式を整理して」って言ったでしょ。
> (dτ/dt)^2 + (dx/dt)^2 + (dy/dt)^2 + (dz/dt)^2 = c^2
> だよ。

で? t は何? (t, x, y, z) じゃなくて (τ,x,y,z) が4次元時空の点で t はその4次元時空に流れる時間なわけ? 滅茶苦茶だろそれ。
920ご冗談でしょう?名無しさん:2009/01/17(土) 23:04:00 ID:???
ぽかーん
921ご冗談でしょう?名無しさん:2009/01/17(土) 23:04:15 ID:???
>>919
tで微分してるから慣性系が観測している物体の速さと、慣性系が観測する
その物体の時間の流れ(固有時)の速さを意味するんだよ。

運動方程式をtで微分した形に直したフォーム知らない?
どの教科書にも書いてあると思うんだけど。
922ご冗談でしょう?名無しさん:2009/01/17(土) 23:04:17 ID:???
「4次元時空を進む速さ」がそもそも未定義。
>>914 は「dτ=dτ」あるいは「静止系では dτ=dt」以上のことは言ってないと思われる。
923ご冗談でしょう?名無しさん:2009/01/17(土) 23:06:33 ID:???
>>921
その教科書の名前と、書いてある数式をそっくり写しなよ。
特殊相対論の教科書なら、ほとんどの場合
tじゃなくてτで微分してるだろ。
924ご冗談でしょう?名無しさん:2009/01/17(土) 23:08:56 ID:???
4次元云々はテクニカルな問題じゃね?
単に>>914はv≠0ならば(dτ/dt) < 1ってことを言いたいだけだと思うんだが
925ご冗談でしょう?名無しさん:2009/01/17(土) 23:09:20 ID:???
> 運動方程式をtで微分した形に直したフォーム知らない?
> どの教科書にも書いてあると思うんだけど。

知らないなぁ。
926ご冗談でしょう?名無しさん:2009/01/17(土) 23:10:01 ID:???
もちろんτで微分しているわけじゃないからローレンツ変換に対して不変な形式
ではないけど、特定の慣性系にいる観測者からは物体の運動はtで微分した形式
の運動方程式を解くことで得られるでしょ。
927ご冗談でしょう?名無しさん:2009/01/17(土) 23:10:34 ID:???
単に 1/γ = √(1 - v^2) (c = 1 として) を言い直しただけにしか見えん。
928ご冗談でしょう?名無しさん:2009/01/17(土) 23:11:49 ID:???
>>925
http://homepage2.nifty.com/eman/relativity/uni_accel.html
の「一定加速の軌道はどう表されるか」の段落にある運動方程式。
929ご冗談でしょう?名無しさん:2009/01/17(土) 23:12:52 ID:???
…教科書?
930ご冗談でしょう?名無しさん:2009/01/17(土) 23:14:18 ID:???
>>927
そうだね。
線素がスカラーであることを表す式からのダイレクトな変形だからね。
でもdt^2で割ることで特定の慣性系から運動する物体がどう見えるかという
視点に変えて議論できる。
931ご冗談でしょう?名無しさん:2009/01/17(土) 23:14:48 ID:???
>>928
どこに、「全ての物体は光速で移動している」なんて書いてあるの?

ていうか、きみの質問したいことが分からない。
紛らわしいから、まず名前をつけて
質問をもう一度まとめてください。
932ご冗談でしょう?名無しさん:2009/01/17(土) 23:15:05 ID:???
>>929
教科書をネットで公表してるとこ知らないからふつーのサイトで許してねw
933ご冗談でしょう?名無しさん:2009/01/17(土) 23:15:52 ID:???
>>931
> どこに、「全ての物体は光速で移動している」なんて書いてあるの?

あのー。
話の流れを理解してレスしてもらえます?
934ご冗談でしょう?名無しさん:2009/01/17(土) 23:20:55 ID:???
>>930
それを「すべての物体は4次元時空の中を常に光速で運動し続けていると解釈」している教科書が
あるのかい? それが「どの教科書にも載っている」のかい?

> 教科書をネットで公表してるとこ知らないからふつーのサイトで許してねw

誰もURL示せなんて言ってない訳だけどね。
教科書名挙げることくらい出来るだろ。

そのサイトにすらそういう「解釈」は載ってないようだけどね。
935ご冗談でしょう?名無しさん:2009/01/17(土) 23:22:50 ID:???
>>933
あなたの流れなんて知りません。
名前をつけて、ちゃんと質問したいことをまとめてください。
さっきは線素がなんちゃら、って話をしてたのに
なぜ急に運動方程式の話になるのか理解できません。
936ご冗談でしょう?名無しさん:2009/01/17(土) 23:24:39 ID:???
>>910
とにかく「4次元時空を進む速さ」を定義してくれ。
937ご冗談でしょう?名無しさん:2009/01/17(土) 23:25:08 ID:???
>>935
dtで割るというのがめちゃくちゃといった人がいたから、そんなことはないよ
という意味で運動方程式の例を出しただけ。
938ご冗談でしょう?名無しさん:2009/01/17(土) 23:26:24 ID:???
>>936
(c*dτ/dt)^2 + (dx/dt)^2 + (dy/dt)^2 + (dz/dt)^2
939ご冗談でしょう?名無しさん:2009/01/17(土) 23:27:45 ID:???
> dtで割るというのがめちゃくちゃといった人

誰? いねーよそんな奴。お前の脳内にしか。
940ご冗談でしょう?名無しさん:2009/01/17(土) 23:29:07 ID:???
一つの世界点に対してτは一意に定義できないので、(τ, x, y, z) を「4次元時空」とみなすことは不可能でそ
941ご冗談でしょう?名無しさん:2009/01/17(土) 23:30:44 ID:???
>>939
そう?ならいいんだけど。
ということは線素の自乗をdt^2で割ったってナンセンスでないでしょ。

だから
(c*dτ/dt)^2 + (dx/dt)^2 + (dy/dt)^2 + (dz/dt)^2 = c^2
はある慣性系から見た物体は時空を光速で進んでいるように見えると
解釈できる、と言っただけ。
942ご冗談でしょう?名無しさん:2009/01/17(土) 23:31:55 ID:???
>>940
微分したら時間の遅れそのものだからメイクセンスするかと。
943ご冗談でしょう?名無しさん:2009/01/17(土) 23:32:12 ID:???
ウィキペなら
http://ja.wikipedia.org/wiki/%E6%99%82%E9%96%93#.E3.82.A2.E3.82.A4.E3.83.B3.E3.82.B7.E3.83.A5.E3.82.BF.E3.82.A4.E3.83.B3.E7.9A.84.E6.99.82.E9.96.93.E8.A7.A3.E9.87.88
現在、未来(時空連続体)の中を私達が光速度秒速約30万キロで走っている、とされる。
私達が光の速度を秒速約30万キロと観測するのもこの為である、とされる。
もし、私たちが時空連続体内を秒速 10万キロで走っていると、光速度は秒速10万キロとなる、とされる。
アインシュタインは記者から「その様な事が本当にあるのか?」と聞かれ、
「信じては貰えないと思うが、過去、現在、未来がすでに同時に私の数学方程式上には存在しているのです」と答えている。
とあるね。でも要出典となっている。
944ご冗談でしょう?名無しさん:2009/01/17(土) 23:33:05 ID:???
正確に言うと慣性系から見たある物体に時間の「流れる速さ」か…。
945ご冗談でしょう?名無しさん:2009/01/17(土) 23:34:39 ID:???
> ということは線素の自乗をdt^2で割ったってナンセンスでないでしょ。

そんなこと誰が言ったの? 誰も言ってないことに必死に反論してたの?

> (c*dτ/dt)^2 + (dx/dt)^2 + (dy/dt)^2 + (dz/dt)^2 = c^2
> はある慣性系から見た物体は時空を光速で進んでいるように見えると
> 解釈できる、と言っただけ。

その解釈が載ってる教科書名を挙げてくれ。キミの独自解釈ならはっきりそう言えばいい。
念のために言っておくが、どうやってもそうは解釈できないと言うほかない。
946ご冗談でしょう?名無しさん:2009/01/17(土) 23:37:03 ID:???
> >>940
> 微分したら時間の遅れそのものだからメイクセンスするかと。

それは単に、

  dτ = dt・√(1-v^2/c^2


式がまともに解釈できることと、特定の解釈の是非は無関係だが。(τ, x, y, z) が時空上の位置を表すのでないのなら、
どうしてその t での微分が時空上の速度と解釈できるのだ?
947ご冗談でしょう?名無しさん:2009/01/17(土) 23:37:27 ID:???
>>945
必死に食らいついてるのはお前だろ。

しばらくこのスレから離れて頭ひやせ。
948ご冗談でしょう?名無しさん:2009/01/17(土) 23:38:41 ID:???
>>946は途中で書き込んじまった

> それは単に、
>
>   dτ = dt・√(1-v^2/c^2

この部分は

 それは単に
 
   dτ = dt・√(1-v^2/c^2)
 
 は「時間の遅れ」として普通に解釈できる、と言ってるだけだろう。

と書くつもりだった。

949ご冗談でしょう?名無しさん:2009/01/17(土) 23:39:14 ID:???
>>943
>信じて貰えないと思うが
このスレの多くの香具師からも未だ信じて貰えてないみたいだなww
950ご冗談でしょう?名無しさん:2009/01/17(土) 23:41:10 ID:???
>>947

で、

> ということは線素の自乗をdt^2で割ったってナンセンスでないでしょ。

「線素の自乗をdt^2で割ったらナンセンス」と言ったのは誰?
951ご冗談でしょう?名無しさん:2009/01/17(土) 23:42:34 ID:???
まーたこの流れか・・・
相対論が絡むと必ずこうなるな
952ご冗談でしょう?名無しさん:2009/01/17(土) 23:42:55 ID:???
> 「信じては貰えないと思うが、過去、現在、未来がすでに同時に私の数学方程式上には存在しているのです」と答えている。

これは>>900の言ってることとは別の話だな。
953ご冗談でしょう?名無しさん:2009/01/17(土) 23:43:14 ID:???
>>950
物理の前に日本語の文章の流れを把握できるようにしましょうね♪
954ご冗談でしょう?名無しさん:2009/01/17(土) 23:44:19 ID:???
>>952
>現在、未来(時空連続体)の中を私達が光速度秒速約30万キロで走っている
これとは同じことだな。
955ご冗談でしょう?名無しさん:2009/01/17(土) 23:45:21 ID:???
煽ってるやつも思いのほかレベル低いな
956ご冗談でしょう?名無しさん:2009/01/17(土) 23:50:04 ID:???
>>952
記者のどういう質問に対してそう答えたのかというくだりをちゃんと読んだら?
957ご冗談でしょう?名無しさん:2009/01/17(土) 23:50:05 ID:???
>>953
つまり「線素を dt で割るのはおかしくない」という話を、「物体が時空上を光速で動いている、
という解釈もおかしくない」という話にすりかえようとしているわけね。

>>954
その部分はそれを書いた奴が言ってることであって、アインシュタインの考えとは別問題だな。
もっとはっきり言えば、その後のアインシュタインの発言とされるものの引用jによってあたかも
それがアインシュタインの考えと一致するように見せかけようとしている (が失敗している) ということだな。
958ご冗談でしょう?名無しさん:2009/01/17(土) 23:51:25 ID:???
顔真っ赤にしてるやつとは話してもしょうがないや。
959ご冗談でしょう?名無しさん:2009/01/17(土) 23:53:17 ID:???
>>957
そう読み取れないから要出典になってるんだよ。
頭悪いの?
960ご冗談でしょう?名無しさん:2009/01/17(土) 23:54:49 ID:???
流れを把握、ということであれば、今の流れは

  一方の側が、急に具体的な論拠を何一つ挙げられなくなって

    「流れ読め」「頭冷やせ」「顔真っ赤にして」

  としか言えなくなった流れ

だな。
961ご冗談でしょう?名無しさん:2009/01/17(土) 23:55:50 ID:???
わかったわかったw
962ご冗談でしょう?名無しさん:2009/01/17(土) 23:55:55 ID:???
どっちもどっちだろ正直w。
963ご冗談でしょう?名無しさん:2009/01/17(土) 23:59:46 ID:???
>>900の解釈が載っている「教科書」は何?
・(τ, x, y, z) が時空上の位置を表さないのに、どうしてその t での微分が時空上の速度と解釈できるのか?

という疑問に>>900は答えてくれないのか?
964ご冗談でしょう?名無しさん:2009/01/18(日) 00:02:34 ID:???
Wikiに書いてある
>現在、未来(時空連続体)の中を私達が光速度秒速約30万キロで走っている
の文章は
(c*dτ/dt)^2 + (dx/dt)^2 + (dy/dt)^2 + (dz/dt)^2 = c^2
の式を言葉にしたといってもいいんだろうな。おそらく。
アインシュタイン本人がどういったかは別として。
965ご冗談でしょう?名無しさん:2009/01/18(日) 00:08:50 ID:???
>>964
何も注釈の無い文章に「時空連続体」という言葉が出てきたのなら、通常の「4次元時空」の意味を
表すと思うべきじゃないのか?
966ご冗談でしょう?名無しさん:2009/01/18(日) 00:10:12 ID:???
>>965
かもね。

じゃあ
>現在、未来(時空連続体)の中を私達が光速度秒速約30万キロで走っている
は何の数式のことを言ってるんだろう。
967ご冗談でしょう?名無しさん:2009/01/18(日) 00:16:53 ID:???
単に時間軸が ct だってことを言ってるだけだと思うけど。
1秒前の自分と今の自分は 30万km の隔たりがあるということを、そう解釈してるんだろ。

いずれにしても推測でしかない。
968ご冗談でしょう?名無しさん:2009/01/18(日) 00:18:31 ID:???
なるほど。
969ご冗談でしょう?名無しさん:2009/01/18(日) 00:22:57 ID:???
なんでわざわざ言い回しを曲解するのかね。
出典自体が曲解なのかのほうが問題だろ。
970ご冗談でしょう?名無しさん:2009/01/18(日) 00:25:41 ID:???
ただ、
(c*dτ/dt)^2 + (dx/dt)^2 + (dy/dt)^2 + (dz/dt)^2 = c^2
の式は慣性系では常に成立しているわけだから、運動するあらゆるものに時計を
つけてやって針の進む速さを観察すると、そのもの自体が空間を進む速さとの
自乗の和が光速の自乗に等しくなるよう制限されてるかのように見えるのも確かだ罠。
971970:2009/01/18(日) 00:26:31 ID:???
あ、>>968より上にいる人たちへのレスね。
972ご冗談でしょう?名無しさん:2009/01/18(日) 00:47:03 ID:???
我々が時間と空間の融合を感じることができるのは>>970のような
事象の観察ということなんだろうな。
973ご冗談でしょう?名無しさん:2009/01/18(日) 00:52:58 ID:???
平等に帯電した無限に広い平面からaなる距離だけ離れた点Pにおける電場のうち
その半分の寄与はPから2aの距離以内にある電荷によるものであることを証明せよ


とりあえずポテンシャルを計算してそこから電場を求めようと思いました

無限に広い平面をz=0において円筒座標を考えてz=aでのポテンシャルを考えると

∫[0、∞]∫[0、2π]ρrdrdθ/(4πε_0sqrt(r^2+a^2))
=ρ/4πε_0∫[0、2π]dθ∫[0、∞]rdr/sqrt(r^2+a^2)
=ρ/4πε_0*2π*[sqrt(r^2+a^2)]^∞_0

最後にrに∞を代入すると発散してしまいました
どこが間違っていますか?
974ご冗談でしょう?名無しさん:2009/01/18(日) 01:12:07 ID:???
電荷の総量が無限大だと無限遠基準のポテンシャルは発散するのでは
975ご冗談でしょう?名無しさん:2009/01/18(日) 01:21:09 ID:???
そうするとどうやって電場を求めればいいですか?
976ご冗談でしょう?名無しさん:2009/01/18(日) 01:32:47 ID:???
>>973
この場合ポテンシャルじゃなく直接クーロンの法則で計算するとものすごく楽

  ・P からの距離が r と r + dr の間にある電荷の量 Q を計算する
  ・P から距離 r 離れた面上の一点に Q が集中していると考えたときの、Qがつくる P での電場の面に垂直な成分を考える。
  ・r = a 〜 2a までと、2a〜∞の積分を比較

単に比較するだけなので係数は省略できるし、ほとんど一瞬で解ける
977ご冗談でしょう?名無しさん:2009/01/18(日) 01:34:56 ID:???
二番目がわかりにくいかもしれないが、Pから半径 r の位置にある電荷は面の上の円周上に分布するので、
面に垂直な方向以外の成分は打ち消されるわけだ。垂直な方向だけを考えるなら、Qが円周上の一点に
集中していると考えても値は同じになる。
978855:2009/01/18(日) 01:38:14 ID:ys4tbjjZ
>>861
非常に懇切なご回答ありがとうございました。
冷静に考えると、「共役」をとるとは転置して各成分の数の複素共役
をとる事なので正方サイズのみ閉じていて
全ての正方行列は対称と反対称の和で書ける
という事ですよね。

ディラックの本は全部webにアップされてるのだったら便利ですね。
本当にありがとうございました。
979ご冗談でしょう?名無しさん:2009/01/18(日) 01:57:34 ID:???
>>976
距離rからの電場は
Q/r^2
で垂直成分は
aQ/r^3
これをa-2aで積分すると
[-aQ/(2r^2)]^2a_a
=3Q/(8a)
2a-∞で積分すると
[-aQ/(2r^2)]^∞_2a
=Q/(8a)

うーん・・・
980ご冗談でしょう?名無しさん:2009/01/18(日) 02:06:12 ID:???
>>979
垂直成分はa/√(r^2+a^2)をかけたもの
あとQはrの関数になってるからな

Q=ρ2πrdr
981980:2009/01/18(日) 02:09:34 ID:???
>>979
もしかして円筒座標は捨てた?
すまんが
そうだったら>>980は間違い
982ご冗談でしょう?名無しさん:2009/01/18(日) 02:11:14 ID:???
>>979

>  ・P からの距離が r と r + dr の間にある電荷の量 Q を計算する

と書いてあるのだから Q は定数じゃないことはわかってほしいものだ。

>>980
垂直成分は a/r をかけたものでいい。
983ご冗談でしょう?名無しさん:2009/01/18(日) 02:17:32 ID:???
>>982
俺は円筒座標でやってて、
>>979の電場の式が1/r^2というのを見て球座標だったと気が付いた
と言い訳させてくだせえ
984ご冗談でしょう?名無しさん:2009/01/18(日) 02:25:44 ID:???
>>874
マルチすんなチンカス

もの凄い勢いで誰かが質問に答えるスレ@理系板@40
http://science6.2ch.net/test/read.cgi/rikei/1230430950/339
985973:2009/01/18(日) 02:26:41 ID:???
ヒントをもとに計算をやり直してる途中なんだけど

>P からの距離が r と r + dr
>r = a 〜 2a
ってので円筒座標じゃなくなってるじゃないの?
986ご冗談でしょう?名無しさん:2009/01/18(日) 02:29:30 ID:???
>>985
それでいいと思う
出題者の言う距離はその意味での距離
円筒半径とは違う

円筒半径だと√3aになることがわかる
極座標の半径だとちょうど2aになるところだ
987ご冗談でしょう?名無しさん:2009/01/18(日) 02:31:51 ID:???
こちらでやった計算では、z軸 (Pから面に降ろした垂線) からの距離 R=√(r^2-a^2) も併用してる。なので
円筒座標か球面座標かとかは別にどうでもいい。

 dR = r/R・dr

であることがわかればあとは計算するだけだろう。しかし Q は dQ と表記するべきだったな。今にして思えば。
988ご冗談でしょう?名無しさん:2009/01/18(日) 02:48:46 ID:???
ありがとうございます
ちゃんと計算合いました

989ご冗談でしょう?名無しさん:2009/01/18(日) 07:03:09 ID:???
なんでvdt=dxなの?
990ご冗談でしょう?名無しさん:2009/01/18(日) 07:49:37 ID:??? BE:56803032-PLT(47777)
次スレ立てました
■ちょっとした疑問や質問はここに書いてね105■
http://science6.2ch.net/test/read.cgi/sci/1232232548/
991ご冗談でしょう?名無しさん:2009/01/18(日) 09:51:33 ID:???
>>875
対消滅した粒子の静止質量エネルギーにちょうど等しいエネルギーをもち、
全運動量が0になるような、対消滅の結果と解釈されるγ線が出るイベントが
観測されてると読めないのか
992ご冗談でしょう?名無しさん:2009/01/18(日) 10:57:15 ID:???
君、想像力逞しいな。www
993ご冗談でしょう?名無しさん:2009/01/18(日) 11:22:11 ID:???
>>875
γ線の「測定」っつーのが、実際にどう行われているかを
知った方がいいな。

ガイガー計数計みたいのを使って「あ、今γ線が出てきた!」
みたいな測定してる光景しか思い浮かばないのであれば、
それは悲しすぎる。
994ご冗談でしょう?名無しさん:2009/01/18(日) 11:44:52 ID:???
つ 霧箱
995ご冗談でしょう?名無しさん:2009/01/18(日) 11:49:01 ID:???
>>994
お前はいつの時代の人間なんだ?
996ご冗談でしょう?名無しさん:2009/01/18(日) 12:52:44 ID:???
加速度も時間的に変化することがあると思いますが、
加速度の微分はなんというのでしょうか?
997ご冗談でしょう?名無しさん:2009/01/18(日) 12:56:30 ID:???
加加速度
998ご冗談でしょう?名無しさん:2009/01/18(日) 13:12:57 ID:???
ありがとうございます
加加速度は物理的にはどういう意味を持ってくるのですか?
999ご冗談でしょう?名無しさん:2009/01/18(日) 13:14:00 ID:???
加える力の変化。
特に意味はない。
1000ご冗談でしょう?名無しさん:2009/01/18(日) 13:16:42 ID:???
>>998
>加加速度は物理的にはどういう意味を持ってくるのですか?
酔いやすさの指数。加加速度が大きいと感覚が以上を来たし酔います。
10011001
このスレッドは1000を超えました。
もう書けないので、新しいスレッドを立ててくださいです。。。